SlideShare una empresa de Scribd logo
1 de 180
Descargar para leer sin conexión
1
PROF.MENDONÇA
PARA APOSTILA COMPLETA VENHA
FAZER UM DE MEUS CURSOS:
Segunda feira 18:30 até 21:45
Curso e colégio integral (Específica semestral)
Fone: (44) 3028-5280 Maringá
Terça feira 17:30 até 19:30
Curso Prime (Física básica semestral)
Fone: (43) 3025-1020 Londrina
Quarta feira 14.00 até 17:00
Curso Sigma ( Específica modular)
Fone: (43) 3321-2652 Londrina
Quinta feira 19:15 até 22:30
Curso Saber (Específica aprofundada)
Fone: (43) 3326-1212 Londrina
Sexta feira 16:00 até 18:00
Curso e colégio integral (Física básica semestral)
Fone: (44) 3028-5280 Maringá
2017
2
Sumário
LISTA 1 - MOVIMENTO UNIFORME...................................................................................................................................3
LISTA 2 - MOVIMENTO UNIFORMEMENTE VARIADO.......................................................................................................7
LISTA 3 - LANÇAMENTO OBLÍQUO..................................................................................................................................12
LISTA 4 - MOVIMENTO CIRCULAR...................................................................................................................................17
LISTA 5 - VETORES E CINEMÁTICA VETORIAL..................................................................................................................21
LISTA 6 - LEIS DE NEWTON..............................................................................................................................................26
LISTA 7 - APLICAÇÕES DAS LEIS DE NEWTON .................................................................................................................31
LISTA 8 - TRABALHO E ENERGIA......................................................................................................................................36
LISTA 9 - DINÂMICA IMPULSIVA E COLISÕES ..................................................................................................................41
LISTA 10 - ESTÁTICA E HIDROSTÁTICA ............................................................................................................................45
LISTA 11 – LEIS DE KEPLER E GRAVITAÇÃO.....................................................................................................................51
LISTA 12 - TERMOMETRIA E DILATAÇÃO ........................................................................................................................57
LISTA 13 - CALORIMETRIA...............................................................................................................................................61
LISTA 14 - GASES E TERMODINÂMICA ............................................................................................................................66
LISTA 15 - TERMODINÂMICA ..........................................................................................................................................72
LISTA 16 - ELETROSTÁTICA 1 – LEI DE COULOMB E CAMPO ELÉTRICO...........................................................................77
LISTA 17 – ELETROSTÁTICA 2 – POTENCIAL, ENERGIA E TRABALHO ..............................................................................85
LISTA 18 – ELETRODINÂMICA 1 – LEIS DE ÔHM..............................................................................................................90
LISTA 19 - ELETRODINÂMICA 2 - GERADORES E RECEPTORES........................................................................................97
LISTA 20 – ELETRODINÂMICA 3 – LEIS DE KIRCHHOFF .................................................................................................105
LISTA 21 – ÓPTICA 1 – FUNDAMENTOS E REFLEXÃO DA LUZ .......................................................................................111
LISTA 22 – REFLEXÃO DA LUZ ESPELHOS ESFÉRICOS ....................................................................................................118
LISTA 23 – REFRAÇÃO DA LUZ LEI DE SNELL-DESCARTES..............................................................................................124
LISTA 24 – ESTUDO DAS LENTES ESFÉRICAS .................................................................................................................130
LISTA 25 – MOVIMENTO HARMÔNICO SIMPLES E ONDULATÓRIA ..............................................................................136
LISTA 26 – INTERFERÊNCIA E ACÚSTICA........................................................................................................................145
LISTA 27 – MAGNETISMO E FONTES DE CAMPO MAGNÉTICO.....................................................................................150
LISTA 28 – FORÇA MAGNÉTICA E INDUÇÃO MAGNÉTICA ............................................................................................157
LISTA 29 – FÍSICA MODERNA ........................................................................................................................................164
PROVAS DE VESTIBULARES .........................................................................................................................176
UEL 1ª FASE 2017..............................................................................................................................................176
UEL 2ª FASE 2017..............................................................................................................................................177
UEM 1º DIA 2016 ...............................................................................................................................................178
3
LISTA 1 - MOVIMENTO UNIFORME
RESUMO DE CONTEÚDO
Características do Movimento Uniforme
O movimento uniforme é o movimento que possui módulo
da velocidade constante, ou seja, ela não varia com o
passar do tempo. Entretanto, essa velocidade, apesar de
ser constante, é diferente de zero, ou seja, ela pode
assumir qualquer outro valor que não seja o zero.
Sendo a aceleração definida da seguinte forma:
E sabendo que no movimento uniforme a variação do
módulo da velocidade é igual a zero, pois a velocidade final
é igual à velocidade inicial, concluímos que a aceleração
escalar é constante e igual a zero.
A função Horária do Movimento Uniforme
No movimento uniforme temos que a velocidade escalar é
constante e coincide com a velocidade escalar média em
qualquer instante ou intervalo de tempo. Matematicamente,
a velocidade escalar média pode ser expressa da seguinte
forma:
Onde:
• ΔS é a variação de posição do móvel, ΔS = S – So;
• Δt é a variação do tempo, Δt = t – to.
Substituído ΔS e Δt na equação da velocidade descrita
acima, temos:
Fazendo tempo inicial igual a zero, to= 0, temos a função
horária do movimento uniforme.
S = So + Vt
EXERCÍCIOS
Questão 01 - (UEG GO/2012)
A órbita do planeta Terra, em torno do Sol, possui uma
distância aproximada de 930 milhões de quilômetros.
Sabendo-se que o ano possui 365 dias e 5 horas, a
velocidade média exercida pela Terra para executar
essa órbita é, aproximadamente, de
a) 106.103 km/h
b) 1.061 km/h
c) 106 km/h
d) 10,6 km/h
Questão 02 - (FMABC/2012)
Duas esferas de dimensões desprezíveis dirigem-se
uma ao encontro da outra, executando movimentos
retilíneos e uniformes (veja a figura). As esferas
possuem velocidades cujos módulos valem 4m/s e
7m/s. A distância entre elas nos 4 segundos que
antecedem a colisão é de
a) 50
b) 44
c) 28
d) 16
e) 12
Questão 03 - (UEL PR)
Sabe-se que o cabelo de uma pessoa cresce em média
3cm a cada dois meses. Supondo que o cabelo não seja
cortado e nem caia, o comprimento total, após terem se
passado 10 anos será:
a) 800mm
b) 1200mm
c) 1000mm
d) 1800mm
e) 150mm
Questão 04 - (UFLA MG)
Considerando uma partícula em movimento retilíneo
com velocidade constante, as seguintes afirmações são
CORRETAS, exceto:
a) O momento linear da partícula mantém-se
constante durante o movimento.
b) A força resultante não-nula que atua na partícula é
constante em módulo, direção e sentido.
c) A posição da partícula dependerá linearmente do
tempo.
d) A energia cinética da partícula será conservada.
e) A aceleração do movimento da partícula será nula.
Questão 05 - (FURG RS)
O gráfico representa o módulo das velocidades de dois
automóveis como função do tempo. Com relação à área
hachurada, podemos dizer que ela representa:
a) a diferença entre as acelerações dos dois
automóveis.
b) a diferença entre as distâncias percorridas pelos
dois automóveis.
c) a aceleração do automóvel A em relação ao
automóvel B.
d) a diferença entre as velocidades dos dois
automóveis.
e) uma grandeza sem qualquer significado físico.
Questão 06 - (FUVEST SP/2006)
Um automóvel e um ônibus trafegam em uma estrada
plana, mantendo velocidades constantes em torno de
100 km/h e 75 km/h, respectivamente. Os dois veículos
passam lado a lado em um posto de pedágio. Quarenta
minutos (2/3 de hora) depois, nessa mesma estrada, o
motorista do ônibus vê o automóvel ultrapassá-lo. Ele
4
supõe, então, que o automóvel deve ter realizado, nesse
período, uma parada com duração aproximada de
a) 4 minutos
b) 7 minutos
c) 10 minutos
d) 15 minutos
e) 25 minutos
Questão 07 - (UFLA MG)
O gráfico abaixo representa a variação das posições de
um móvel em função do tempo (S = f(t)).
10
0
-10
1 2 3 4 5 6 7 86
S(m)
t(s)
O gráfico de v x t que melhor representa o movimento
acima é:
5
-5
10
0
V(m/s)
2 4 6 8
| | | |
t(s)
a.
5
-5
10
0
V(m/s)
2 4 6 8
| | | |
t(s)
b.
5
-5
10
0
V(m/s)
2 4 6 8
| | | |
t(s)
c.
5
-5
10
0
V(m/s)
2 4 6 8
| | | |
t(s)
d.
Questão 08 - (UFPE)
A equação horária para o movimento de uma partícula é
x(t) = 15 – 2 t, onde x é dado em metros e t em
segundos. Calcule o tempo, em s, para que a partícula
percorra uma distância que é o dobro da distância da
partícula à origem no instante t = 0 s.
Questão 09 - (UERJ/2011)
Uma partícula se afasta de um ponto de referência O, a
partir de uma posição inicial A, no instante t = 0 s,
deslocando-se em movimento retilíneo e uniforme,
sempre no mesmo sentido.
A distância da partícula em relação ao ponto O, no
instante t = 3,0 s, é igual a 28,0 m e, no instante t =
8,0 s, é igual a 58,0 m.
Determine a distância, em metros, da posição inicial A
em relação ao ponto de referência O.
Questão 10 - (PUC MG/2006)
O gráfico mostra a velocidade (v) de um objeto em
movimento retilíneo, em função do tempo t.
Sobre o movimento do objeto, é CORRETO afirmar:
a) Analisando-se o gráfico como um todo, pode-se
afirmar que o objeto tende a parar.
b) Entre os instantes 0 e 0,8 s , o objeto está em
movimento retilíneo uniformemente variado.
c) Até 1,2 s , a distância percorrida pelo móvel foi de
2,4 m.
d) A partir do instante 1,2 s , o objeto passa a se se
mover em movimento retilíneo uniforme.
Questão 11 - (UEL PR)
Em grandes cidades, a rota das ambulâncias leva em
consideração fatores como proximidade do local do
chamado e rapidez no deslocamento.
Considere um chamado proveniente da região central
de uma cidade, às 19h, conforme ilustra a figura, e que
para atendê-lo, estão disponíveis quatro bases de
ambulâncias, X, Y, W e Z.
Para se definir a melhor rota, foram consideradas as
velocidades médias desenvolvidas pelas ambulâncias
em alguns intervalos de horários:
5
Assim, o chamado comunicado às 19h será atendido
mais rapidamente pela ambulância da base
a) X, seguindo pela rota 1.
b) Z, seguindo pela rota 2.
c) W, seguindo pela rota 3.
d) Y, seguindo pela rota 4.
e) Z, seguindo pela rota 5.
Questão 12 - (UEL PR)
Um pequeno animal desloca-se com velocidade média
igual a 0,5 m/s. A velocidade desse animal em km/dia
é:
a) 13,8
b) 48,3
c) 43,2
d) 1,80
e) 4,30
Questão 13 - (UEL PR)
Nos edifícios, os números que identificam os
apartamentos representam vetorialmente suas
posições, isto é, esses números são compostos por
dígitos que representam o andar (posição na vertical) e
a localização do apartamento no andar (posição na
horizontal). Em um edifício de 10 andares, que tem um
apartamento por andar, cada apartamento é identificado
por um número que varia de 1 a 10. Se nesse edifício,
cada andar tem altura de 5 metros, qual é a distância
percorrida na direção vertical por alguém que sai do
apartamento 3 e vai para o apartamento 9?
a) 6 m
b) 15 m
c) 30 m
d) 45 m
e) 60 m
Questão 14 - (UEL PR)
O raio médio da órbita elíptica da Terra em torno do Sol
é considerado para definir 1 Unidade Astronômica
(U.A.): 1 U.A.  1,49108
km = 1,491011
m. A U.A. é
utilizada para medir os raios das órbitas dos planetas do
sistema solar, entretanto é uma unidade muito pequena
para ser utilizada como parâmetro de medida para as
distâncias das estrelas. Para essas distâncias, é utilizado
o Ano-Luz (A. L.) que é a distância percorrida pela luz
em um ano. Por exemplo, a estrela -Centauri está a 4,3
A. L. de distância da Terra. Se a velocidade de
propagação da luz é igual a c = 3  108
m/s, é correto
afirmar que a distância média entre o Sol e a Terra é de:
a) 150 Segundos-Luz.
b) 300 Segundos-Luz.
c) 430 Segundos-Luz.
d) 500 Segundos-Luz.
e) 600 Segundos-Luz.
Questão 15 - (UEM PR)
Um trem se move com velocidade constante.
Dentro dele estão o observador A e um garoto. Na
estação, parado sobre a plataforma, está o observador
B. Quando o trem passa pela plataforma, o garoto joga
uma bola verticalmente para cima. Desprezando-se a
resistência do ar, podemos afirmar que:
01. o observador A vê a bola se mover
verticalmente para cima e cair nas mãos do garoto.
02. o observador B vê a bola descrever uma parábola e
cair nas mãos do garoto.
04. os dois observadores vêem a bola se mover
com a mesma aceleração.
08. o observador B vê a bola se mover
verticalmente para cima e cair atrás do garoto.
16. o observador A vê a bola descrever uma
parábola e cair atrás do garoto.
Questão 16 - (UEM PR)
Com velocidade constante, um caminhão se move num
trecho retilíneo horizontal, sem atrito. Ele transporta,
sobre a carroceria, pedras e um garoto. Se o garoto
começa a arremessar pedras, pode–se concluir que a
velocidade do caminhão, na direção inicial do
movimento,
01. aumenta, se as pedras forem arremessadas para
trás.
02. diminui, se as pedras forem arremessadas para
frente.
04. diminui, se as pedras forem arremessadas
verticalmente para cima.
08. aumenta, se as pedras forem arremessadas
lateralmente, perpendicularmente à direção do
movimento do caminhão.
16. permanece constante, qualquer que seja a direção
em que o garoto arremessar as pedras.
Questão 17 - (UEM PR)
Os diagramas abaixo (a, b, c, d) mostram uma esfera
movendo-se em quatro situações diferentes.
Considerando que, em todas as situações, não existem
forças dissipativas atuando, que, em a e em b, é dado
um pequeno empurrão na esfera para que ela comece a
se mover e que, em d, a colisão entre a esfera e a
superfície é perfeitamente elástica, podese afirmar que,
01. em a, a esfera está em movimento retilíneo
uniforme (MRU).
02. em b, a esfera está em MRU.
04. em c, a esfera está em movimento harmônico
simples (MHS).
08. em d, a esfera está em MHS.
16. tanto em c como em d, decorrido algum tempo, a
esfera pára.
32. tanto em a como em b, a força que a superfície faz
sobre a esfera é constante.
Questão 18 - (PUC MG)
6
A tabela abaixo contém as velocidades, consideradas
constantes, em metros por segundo, que quatro
nadadoras apresentaram na ida e na volta nadando
estilo livre em uma piscina de 50 metros de
comprimento.
nadadora A B C D E
ida 1,00 1,25 0,50 0,60 0,80
volta 1,00 0,80 1,60 0,90 0,70
Qual delas fez a virada em primeiro lugar?
a) A
b) B
c) C
d) D
e) E
Questão 19
Um candidato sai de sua residência para prestar
vestibular pretendendo percorrer a distância total até o
local da prova em uma hora, conduzindo seu automóvel
com velocidade média de 60 km/h. Após percorrer os
primeiros 10 km do percurso em 10 minutos, percebe
que esqueceu o documento de identificação e retorna
para apanhá-lo. Sua mãe o espera no portão com o
documento.
Desprezando-se o tempo para receber o documento e
manobrar o carro, para que esse candidato consiga
chegar ao local da prova no horário previsto
anteriormente, ele deverá desenvolver no percurso de
retorno à sua casa e ida até o local da prova uma
velocidade média, em km/h, igual a
a) 78.
b) 84.
c) 90.
d) 98.
e) 72.
Questão 20 - (UnB DF)
Qual é o tempo gasto para que um metrô de 200m a
uma velocidade de 180km/h atravesse um túnel de
150m? Dê sua resposta em segundos.
GABARITO:
1) Gab: A 2) Gab: B 3) Gab: D
4) Gab: B 5) Gab: B 6) Gab: C
7) Gab: B 8) Gab: 15s
9) Gab: 10,0m 10) Gab: D
11) Gab: E 12) Gab: C
13) Gab: C 14) Gab: D
15) Gab: 01-02-04 16) Gab: 01-02
17) Gab: 02-04-08 18) Gab: B
19) Gab: B 20) Gab: 07
21) Gab: 04 litros 22) Gab: B
23) Gab: A 24) Gab: C
25) Gab: C 26) Gab: A
27) Gab: B 28) Gab: E 29) Gab: D
30) Gab: D 31) Gab: D 32) Gab: B
33) Gab:
a) vm  17,5 km/s
b) 't = 16667 s  4,63 h
c) 2181.
34) Gab: C 35) Gab: D 36) Gab: D
37) Gab: C 38) Gab: B 39) Gab: C
40) Gab: A
7
LISTA 2 - MOVIMENTO UNIFORMEMENTE VARIADO
RESUMO DE CONTEÚDO
O movimento de uma partícula só é
considerado uniformemente variado quando o módulo de
sua velocidade varia, aumentando ou diminuindo ao longo
do tempo. Quando observamos que a velocidade aumenta ou
diminui de maneira uniforme, independentemente da
trajetória descrita por essa partícula, quer dizer que a
partícula possui aceleração constante.
Exemplo:
Se a aceleração é 10m/s2
, a velocidade varia 10m/s a cada
segundo, ou seja, significa que o valor da velocidade da
partícula aumenta numa taxa constante.
Classificação do Movimento uniformemente variado
(MUV)
Para descrevermos o movimento de uma partícula devemos
adotar um referencial. Podemos classificar esse movimento
como acelerado ou retardado:
Se a variação do módulo da velocidade escalar é positivo, ou
seja, o valor da velocidade escalar em módulo aumenta
classificamos o movimento como acelerado. ( Movimento
acelerado progressivo ou movimento acelerado regressivo).
Se a variação do módulo da velocidade escalar é negativo,
ou seja, o valor da velocidade escalar em módulo diminui,
classificamos o movimento como retardado. ( Movimento
retardado progressivo ou movimento retardado regressivo).
Resumindo:
Movimento acelerado
- |v|=> aumenta.
- a e v possuem o mesmo sinal.
Movimento retardado
- |v|=> diminui.
- a e v possuem sinais contrários.
Fonte:
http://www.infoescola.com/fisica/movimento-
uniformemente-variado-muv/
EXERCÍCIOS
TEXTO: 1 -
Leia o texto e analise o gráfico.
Um objeto que não pode ser considerado uma partícula
é solto de uma dada altura sobre um lago. O gráfico
abaixo apresenta a velocidade desse objeto em função
do tempo. No tempo t = 1, 0s, o objeto toca a superfície
da água. Despreze somente a resistência no ar.
Questão 01 - (UEL PR/2011)
Qual a profundidade do lago?
a) 1 m
b) 5 m
c) 7 m
d) 100 m
e) 1000 m
Questão 02 - (ESCS DF/2009)
A velocidade de um corpo em função do tempo é dada
pelo gráfico:
O espaço percorrido pelo corpo entre 0 e 4s é:
a) 30m;
b) 35m;
c) 40m;
d) 45m;
e) 50m.
Questão 03 - (UFF RJ)
No gráfico, v2 indica o quadrado da velocidade e x, a
posição. O movimento foi realizado em trajetória
retilínea, partindo de posição inicial nula.
V
x0
2
O gráfico corresponde a um movimento:
a) uniforme
b) uniformemente acelerado com velocidade inicial não
nula
c) uniformemente retardado com velocidade inicial
nula.
d) uniformemente acelerado com velocidade inicial
nula.
e) uniformemente retardado com velocidade inicial
não nula.
Questão 04 - (FEEVALE RS)
O gráfico abaixo representa a velocidade de um
automóvel que se movimenta em uma avenida retilínea,
partindo de um semáforo que abriu, até parar em um
outro semáforo fechado.
8
t(s)604020
20
v(m/s)
A distância entre os dois semáforos vale
a) 200 m
b) 400 m
c) 600 m
d) 800 m
e) 1000 m
Questão 05 - (CESJF MG)
O gráfico abaixo representa a variação da velocidade de
dois carros , A e B , em função do tempo . Os carros
partem de uma mesma posição no mesmo instante ,
seguindo uma mesma trajetória retilínea e rumo a uma
mesma cidade.
0 0,1 0,2 0,3 0,4 t(h)
V(km/h)
60
80 B
A
Após 6,0 minutos de movimento a distância entre os
carros é de :
a) 1,0 km
b) 3,0 km
c) 6,0 km
d) 5,0 km
e) N. R. A .
Questão 06 - (UNESP)
Um atleta de corridas de curto alcance, partindo do
repouso, consegue imprimir a si próprio uma aceleração
constante de 5,0m/s2
durante 2,0s e, depois, percorre o
resto do percurso com a mesma velocidade adquirida no
final do período de aceleração.
a) Esboce o gráfico da velocidade do atleta em função
do tempo, numa corrida de 5s.
b) Qual é a distância total que ele percorre nessa
corrida de 5s?
Questão 07 - (UNIRIO RJ)
Analisando o gráfico abaixo, que relaciona a posição dos
móveis A e B com o tempo t, assinale a opção correta.
s
A
B
t
c
a) VA = VB
b) VA > VB
c) VA < VB
d) aA > aB
e) aA < aB
Questão 08 - (UNIRIO RJ)
A velocidade de uma partícula varia com o passar do
tempo conforme o gráfico abaixo.
v(m/s)
t(s)0 1 2 3 4
O seu deslocamento do instante 0 s até o instante 1 s
foi de 1,5 m. Através da observação do gráfico podemos
concluir que seu deslocamento entre os instantes 2 s e
3 s, em m, foi de:
a) 2,0
b) 2,05
c) 3,0
d) 3,5
e) 4,0
Questão 09 - (UNIFOR CE)
O gráfico fornece a velocidade de um corpo, que se move
em linha reta, em função do tempo. Sabe-se que, no
instante t  0, o corpo se encontra na posição 20 m.
No instante t  8,0 s, o corpo estará na posição
a) 40 m
b) 20 m
c) 20 m
d) 40 m
e) 60 m
Questão 10 - (UNIFOR CE)
O gráfico abaixo representa, em função do tempo, a
velocidade escalar de uma partícula que está em
movimento retilíneo.
9
O movimento é acelerado SOMENTE no trecho
a) I
b) II
c) III
d) IV
e) V
Questão 11 - (UNIOESTE PR/2007)
Em uma competição esportiva entre escolas, dois
alunos, João e Pedro, disputam quem vai chegar
primeiro à posição C. João está inicialmente em repouso
na posição A, conforme a figura abaixo, e precisa se
deslocar 144 m para alcançar o ponto C, seguindo a
trajetória retilínea AC. No mesmo instante em que João
inicia seu movimento com uma aceleração uniforme de
0,5 m/s2 mantida em todo o trajeto, Pedro está
passando pela posição B com uma velocidade de 2 m/s.
Pedro segue a trajetória retilínea BC de comprimento
igual a 145 m e alcança o ponto C no mesmo instante
que João. Assinale a alternativa que fornece o valor da
aceleração de Pedro, em metros por segundo ao
quadrado (com arredondamento na segunda casa
decimal), no trecho BC.
a) 0,50
b) 0,01
c) 0,11
d) 0,17
e) 0,34.
Questão 12 - (UNIOESTE PR)
Um feixe de íons negativos é acelerado no interior de um
tubo de raios catódicos. Cada íon é acelerado desde o
repouso até a velocidade final de 2,0107
m/s em uma
distância de 10 mm. Determine, em fN (femto newtons),
o módulo da força resultante que atua sobre cada íon
nesse tubo de raios catódicos. Use, para a massa do íon,
o valor de 10,01031
kg e lembre-se de que o
multiplicador f (femto) vale 1015
.
Questão 13 - (UNIOESTE PR)
Uma pedra é atirada verticalmente para cima, a partir
de uma janela de um edifício, situada a 5 m de altura
em relação ao solo, com uma velocidade inicial igual a
20 m/s. No mesmo instante, de uma janela situada a 25
m de altura em relação ao solo, exatamente acima da
primeira janela, é atirada verticalmente para cima uma
outra pedra com uma velocidade inicial de 10 m/s.
Desprezando a resistência do ar, determine após
quantos segundos, a partir do instante do arremesso, as
duas pedras estarão à mesma altura.
Questão 14 - (UNIOESTE PR/2006)
Considere a queda livre de um corpo a partir do repouso.
Tome g = 10,0 m/s2
. Com relação a esse movimento,
pode-se afirmar que
00. a aceleração do corpo é constante e igual a
10,0m/s2
, independente de considerar o atrito com
o ar.
01. a distância que o corpo cai após 3,0s é de 50,0m,
quando não há atrito.
02. se não há atrito com o ar, a sua velocidade, após
ter caído 20,0m, é de 20,0m/s.
03. caso não haja atrito com o ar, o tempo necessário
para o corpo atingir uma velocidade de 40,0m/s é
de 6,0 s.
04. não havendo atrito com o ar, o tempo gasto para o
corpo cair 500,0m é de 10,0s.
Questão 15 - (UNIOESTE PR/2009)
Em uma pista de testes um automóvel, partindo do
repouso e com aceleração constante de 3 m/s2
, percorre
certa distância em 20 s. Para fazer o mesmo trajeto no
mesmo intervalo de tempo, porém com aceleração nula,
um segundo automóvel deve desenvolver velocidade de
a) 20 m/s
b) 25 m/s
c) 80 km/h
d) 100 km/h
e) 108 km/h
Questão 16 - (UEL PR/2011)
No circuito automobilístico de Spa Francorchamps, na
Bélgica, um carro de Fórmula 1 sai da curva Raidillion
e, depois de uma longa reta, chega à curva Les Combes.
Figura: Circuito automobilístico de Spa Francorchamps
A telemetria da velocidade versus tempo do carro foi
registrada e é apresentada no gráfico a seguir.
Qual das alternativas a seguir contém o gráfico que
melhor representa a aceleração do carro de F-1 em
função deste mesmo intervalo de tempo?
10
a)
b)
c)
d)
e)
Questão 17
Analise as alternativas abaixo e assinale o que for
correto.
01. O gráfico da velocidade em função do tempo, para
um móvel descrevendo um Movimento Retilíneo e
Uniforme, é uma reta paralela ao eixo dos tempos.
02. O gráfico da posição em função do tempo, para um
móvel descrevendo um movimento Retilíneo e
Uniforme, é uma reta, e o coeficiente angular
dessa reta fornece a velocidade do móvel.
04. O gráfico do espaço percorrido em função do
tempo é uma reta para um móvel que realiza um
Movimento Uniforme qualquer.
08. O espaço percorrido por um móvel, em um dado
intervalo de tempo, pode ser obtido calculando-se
a “área sob a curva” do gráfico da velocidade em
função do tempo, para aquele dado intervalo de
tempo.
16. O gráfico da velocidade em função do tempo, para
um móvel descrevendo um Movimento Retilíneo
Uniformemente Variado, é uma parábola.
Questão 18 - (UEL PR)
O que acontece com o movimento de dois corpos, de
massas diferentes, ao serem lançados horizontalmente
com a mesma velocidade, de uma mesma altura e ao
mesmo tempo, quando a resistência do ar é desprezada?
a) O objeto de maior massa atingirá o solo primeiro.
b) O objeto de menor massa atingirá o solo primeiro.
c) Os dois atingirão o solo simultaneamente.
d) O objeto mais leve percorrerá distância maior.
e) As acelerações de cada objeto serão diferentes.
Questão 19 - (UEL PR)
Um motorista dirige um automóvel a 72 km/h quando
percebe que o semáforo a sua frente está fechado. Ele
pisa, então, no pedal do freio e a velocidade do
automóvel diminui como mostra o gráfico abaixo.
0 5,0 t(s)
72
v(km/h)
A menor distância que o automóvel deve estar do
semáforo, no instante em que o motorista pisa no pedal
do freio, para que não avance o semáforo é, em metros,
a) 144
b) 72
c) 50
d) 30
e) 18
Questão 20 - (UEL PR/2008)
No departamento de Física da UEL, foi realizado um
experimento de queda livre cuja equação de movimento
foi obtida com auxílio de um computador. O experimento
consistiu na aquisição de um sinal elétrico cada vez que
um objeto, em queda, interrompia um feixe de luz laser
que era direcionado por espelhos (separados por 2 cm)
até ser coletado numa fotocélula (ver figura). O objeto
de estudo foi uma pena, para qual a resistência do ar
não pode ser desprezada.
A fotocélula, quando recebia luz, produzia uma tensão
elétrica, e o tempo entre as interrupções de luz eram
registradas pelo computador. Ao final da queda, obteve-
se um gráfico de espaço percorrido versus tempo (S x t)
cujos dados são mostrados no gráfico abaixo. Pelo
arranjo experimental, conseguiu-se simplificar a
equação que descreve o movimento, uma vez que o
espaço inicial, bem como a velocidade inicial, puderam
ser considerados zero.
11
Sabendo que a função polinomial que descreve o
movimento é do tipo 2
CxBxAxF )( , e que
cm0SA  e cm/s 0vB , qual o valor aproximado da
aceleração “a” da pena em cm/s2
?
a) 9,8 cm/s2
.
b) 12,2 cm/s2
.
c) 5,3 cm/s2
.
d) 3,6 cm/s2
.
e) 7,1 cm/s2
.
GABARITO:
1) Gab: C 2) Gab: C 3) Gab: B
4) Gab: D 5) Gab: A 6) Gab:
a)
V(m/s)
t(s)
10
5
0 1 2 3 4 5 ;
b) d = 40m
7) Gab: A 8) Gab: D 9) Gab: E
10) Gab: D 11) Gab: E 12) Gab: 20
13) Gab: 14) Gab: FFVFV 15) Gab: E
16) Gab: D 17) Gab: 15 18) Gab: C
19) Gab: C 20) Gab: E 21) Gab: 03
22) Gab: 07 23) Gab: 25 24) Gab: 30
25) Gab: 15 26) Gab: B 27) Gab: D
28) Gab: D 29) Gab: a) a = 6,0m/s2
;
b) v = 11m/s; c) vH = 6,2m/s
30) Gab: 01-F; 02-V; 04-F; 08-V; 16-F; 32-F.
31) Gab: 05 32) Gab: B 33) Gab: 11
34) Gab: E 35) Gab: 29 36) Gab: B
37) Gab: A 38) Gab: 14 39) Gab: 20
40) Gab: 27
12
LISTA 3 - LANÇAMENTO OBLÍQUO
RESUMO TEÓRICO
Quando uma bola é chutada em uma partida de futebol,
podemos observar que ela realiza um movimento parabólico.
Esse movimento é chamado de lançamento oblíquo.
Considere um corpo sendo lançado a partir do solo, formando
um ângulo α com a horizontal, com velocidade inicial v0.
Desprezando as forças dissipativas, o corpo fica sujeito
apenas à ação da gravidade, descrevendo uma trajetória
parabólica.
Movimento Horizontal
Assim como no Lançamento Horizontal, o movimento na
direção do eixo x, no lançamento oblíquo, é uniforme, pois a
velocidade é constante. Portanto, a função horária do
movimento horizontal é:
x = vx.t
A distância horizontal percorrida pelo corpo desde o
lançamento é chamada alcance máximo. Podemos
determinar o alcance máximo pela equação:
Para determinar a posição do móvel em relação à horizontal
temos que determinar a componente da velocidade inicial v0
na direção do eixo x. O módulo da velocidade na direção do
eixo x é:
vx = v0 . cosα
Movimento Vertical
O movimento vertical está sob a ação da gravidade, isso
implica que o movimento é uniformemente variado e a
velocidade vy diminui à medida que a altura em relação ao
solo aumenta.
O componente da velocidade inicial na direção do eixo y é:
v0y = v0 . senα
As funções horárias do movimento vertical são:
Função horária do espaço
y = v0yt + gt2
2
Função horária da velocidade
vy = v0y + gt
Equação de Torricelli
vy
2
= v0y
2
+ 2gy
A altura máxima pode ser calculada usando a equação:
Após atingir a altura máxima, o corpo move-se em queda
livre sob ação da gravidade, e sua velocidade aumenta com
o tempo.
EXERCÍCIOS
TEXTO: 1 - Comum à questão: 1
Três bolas – X, Y e Z – são lançadas da borda de uma
mesa, com velocidades iniciais paralelas ao solo e
mesma direção e sentido.
A tabela abaixo mostra as magnitudes das massas e das
velocidades iniciais das bolas.
810Z
105Y
205X
(m/s)
INICIALVELOCIDADE
)g(
MASSA
BOLAS
Questão 01 - (UERJ/2012)
As relações entre os respectivos alcances horizontais Ax,
Ay e Az das bolas X, Y e Z, com relação à borda da mesa,
estão apresentadas em:
a) Ax < Ay < Az
b) Ay = Ax = Az
c) Az < Ay < Ax
d) Ay < Az < Ax
Questão 02 - (UEM PR)
Dois corpos idênticos A e B são lançados,
simultaneamente, da mesma posição, com a mesma
velocidade inicial, formando o mesmo ângulo a com a
horizontal. Sobre o corpo A, atua apenas a força peso.
Sobre o corpo B, além do próprio peso, atua,
favoravelmente ao movimento, uma força horizontal
constante. Pode-se afirmar que os corpos
01. chegam ao solo simultaneamente.
02. têm o mesmo alcance horizontal.
04. atingem a mesma altura máxima.
08. têm a mesma velocidade quando atingem o solo.
16. têm a mesma aceleração.
Questão 03 - (PUC RJ)
Três bolinhas são largadas da mesma altura h como
mostra a figura. As bolinhas A e B parte do repouso e a
bolinha C tem velocidade inicial vo horizontal.
A B C
S
h
Vo
Qual é a afirmativa correta? despreze a resistência do ar
e o atrito da bolinha A com o solo.
a) As três bolinhas chegam ao mesmo tempo na região
horizontal S do solo.
b) A bolinha A chega primeiro em S e as bolinhas B e
C chegam juntas depois.
c) bolinha B chega primeiro em S, em seguida a
bolinha A e por último a bolinha C.
13
d) As bolinhas A e B chegam juntas em S e depois a
bolinha C.
e) As boinhas B e C chegam juntas em S e depois
chega a bolinha A.
Questão 04 - (UNESP/2007)
Em uma partida de futebol, a bola é chutada a partir do
solo descrevendo uma trajetória parabólica cuja altura
máxima e o alcance atingido são, respectivamente, h e
s. Desprezando o efeito do atrito do ar, a rotação da bola
e sabendo que o ângulo de lançamento foi de 45º em
relação ao solo horizontal, calcule a razão s/h.
Dado: 2/245cos45sen 00

Questão 05 - (UFU MG)
Da superfície de uma mesa são lançadas
horizontalmente duas pequenas esferas A e B, com
velocidades iniciais v0 e 2v0, respectivamente. Se a
resistência do ar for considerada irrelevante, como se
relacionam os tempos de queda das esferas?
a) tA = tB
b) tA = tB/4
c) tA = tB/2
d) tA = 4tB
e) tA = 2tB
Questão 06 - (FFFCMPA RS/2007)
Uma pedra pe arremessada horizontalmente, com uma
velocidade de 20m/s, de uma ponte que está a 16m
acima da superfície da água. Qual a velocidade da
pedra, após atingir a água?
(Considere g = 9,8m/s2
)
a) 26,7m/s.
b) 13,5m/s.
c) 713,6m/s.
d) 42,5m/s.
e) 246,4m/s.
Questão 07 - (UFF RJ/2011)
Após um ataque frustrado do time adversário, o goleiro
se prepara para lançar a bola e armar um contraataque.
Para dificultar a recuperação da defesa adversária, a
bola deve chegar aos pés de um atacante no menor
tempo possível. O goleiro vai chutar a bola, imprimindo
sempre a mesma velocidade, e deve controlar apenas o
ângulo de lançamento. A figura mostra as duas
trajetórias possíveis da bola num certo momento da
partida.
Assinale a alternativa que expressa se é possível ou não
determinar qual destes dois jogadores receberia a bola
no menor tempo. Despreze o efeito da resistência do
ar.
a) Sim, é possível, e o jogador mais próximo
receberia a bola no menor tempo.
b) Sim, é possível, e o jogador mais distante
receberia a bola no menor tempo.
c) Os dois jogadores receberiam a bola em tempos
iguais.
d) Não, pois é necessário conhecer os valores da
velocidade inicial e dos ângulos de lançamento.
e) Não, pois é necessário conhecer o valor da
velocidade inicial.
Questão 08 - (UFC CE/2007)
Considere o lançamento de dois projéteis com o mesmo
ângulo inicial. O projétil 1 tem a metade da massa do
outro (denotado por 2). Qual a relação entre as
velocidades iniciais dos projéteis para que tenham o
mesmo alcance?
a) 21 v
2
1
v 
b) v1 = 2v2
c) v1 = v2
d) 21 v
3
1
v 
e) v1 = 3v2
TEXTO: 2 - Comuns às questões: 9, 10
Considere o enunciado abaixo.
Na figura que segue, estão apresentadas as trajetórias
de dois projéteis, A e B, no campo gravitacional
terrestre. O projétil A é solto da borda de uma mesa
horizontal de altura H e cai verticalmente; o projétil B é
lançado da borda dessa mesa com velocidade horizontal
de 1,5 m/s.
(O efeito do ar é desprezível no movimento desses
projéteis.)
Questão 09 - (UFRGS/2007)
Se o projétil A leva 0,4 s para atingir o solo, quanto
tempo levará o projétil B?
a) 0,2 s
b) 0,4 s
c) 0,6 s
d) 0,8 s
e) 1,0 s
Questão 10 - (UFRGS/2007)
Qual será o valor do alcance horizontal X do projétil B?
a) 0,2 m
b) 0,4 m
c) 0,6 m
d) 0,8 m
e) 1,0 m
TEXTO: 3 - Comum à questão: 11
Dados que podem ser necessários:
valor da aceleração da gravidade: g = 10,0 m/s2
;
velocidade da luz no vácuo: c = 3,0 x 108
m/s;
equivalente mecânico da caloria: 1 cal = 4,186 joules;
duração do intervalo de tempo de um ano na Terra: 3,0
x 107
s;
Questão 11 - (UNIOESTE PR/2008)
Considere a seguinte situação: uma bola de futebol foi
chutada por um jogador com um ângulo de 30 graus
acima da horizontal com uma velocidade inicial de
módulo 20,0 m/s, passando a executar um movimento
em duas dimensões, em um plano vertical. Suponha que
14
a resistência do ar seja desprezível e que a aceleração
da gravidade não varie, tendo sempre o valor de 10,0
m/s2
.
Com base no enunciado acima, considere as afirmativas
a seguir:
I. A força que acelera a bola de futebol tem um valor
que depende da altura na qual a bola estiver situada.
II. O movimento da bola de futebol pode ser
decomposto nas direções horizontal e vertical e os
movimentos nas duas direções podem ser
considerados independentes entre si.
III. A quantidade de movimento da bola é uma grandeza
escalar que se conserva durante o movimento da
bola.
IV. A energia mecânica da bola é uma grandeza escalar
que pode ser expressa em kW·h e que se conserva
durante o movimento da bola.
Estão corretas apenas as afirmativas:
a) I e II.
b) II e IV.
c) III e IV.
d) I, II e III.
e) I, III e IV.
Questão 12 - (UEL PR/2010)
Um sistema mecânico que consiste de um pequeno tubo
com uma mola consegue imprimir a uma esfera de
massa m uma velocidade fixa v0. Tal sistema é posto
para funcionar impulsionando a massa na direção
vertical, a massa atingindo a altura máxima h e voltando
a cair. Em seguida o procedimento é efetuado com o eixo
do tubo formando um determinado ângulo com a direção
horizontal de modo que o alcance R nesta direção seja
maximizado. Tais situações estão representadas na
figura a seguir.
Os experimentos ocorrem em um local onde a
aceleração da gravidade g′ é um pouco menor que seu
valor na superfície terrestre g = 9,8 m/s2
.
Baseado nesses dados e concordando com expressões
cinemáticas para os movimentos de queda livre e
lançamento oblíquo, é correto afirmar:
a) A razão
R
h
obedecerá a relação
'g2
g
R
h

b) A razão
R
h
obedecerá a relação
'g
g2
R
h

c) A razão
R
h
obedecerá a relação
g2
'g
R
h

d) A distância R a ser alcançada pela massa será a
mesma que se obteria em um experimento na
superfície terrestre porque tal quantidade só
depende do valor da componente horizontal da
velocidade v0 cos(θ).
e) R e h serão diferentes de seus valores obtidos em
experimentos realizados na superfície mas a relação
2
1
R
h
 se manterá porque esta independe do valor
local da aceleração da gravidade.
Questão 13 -
Um jogador de golf desfere uma tacada, imprimindo à
bola uma velocidade inicial com módulo v0 = 20 m/s e
ângulo  = 45º em relação ao eixo-x horizontal, de
acordo com a figura abaixo. Desprezando a resistência
aerodinâmica do ar e considerando que o módulo da
aceleração da gravidade vale g = 10 m/s2
, determine o
alcance máximo A da bola de golf.
a) 4 metros
b) 200 metros
c) 100 metros
d) 40 metros
e) 2 metros
Questão 14 - (UEM PR/2013)
Uma pequena esfera é lançada do solo com velocidade
inicial de módulo vo, em uma direção que forma um
ângulo  com a horizontal. Desprezando o atrito com o
ar, assinale o que for correto.
01. A função horária da posição da esfera no eixo
horizontal é uma função de primeiro grau.
02. Tanto a função horária da posição da esfera no
eixo vertical, quanto a função que descreve a
trajetória da esfera (função da posição da esfera
no eixo vertical em relação ao eixo horizontal) são
funções de segundo grau.
04. A componente horizontal da velocidade inicial da
esfera é dada por vo cos.
08. O intervalo de tempo que a esfera leva até atingir
a altura máxima é igual ao que ela levaria se fosse
lançada verticalmente para cima com a mesma
velocidade inicial vo.
16. O módulo da velocidade com que a esfera atinge
o solo é vo.
Questão 15 - (UEM PR/2006)
Em uma cena de filme, um policial em perseguição a um
bandido salta com uma moto do topo de um prédio a
outro. Considere que ambos os prédios têm o topo
quadrado com uma área de 900 m2
e que o policial
motorizado se lança horizontalmente com uma
velocidade de 72 km/h. Considere ainda que a distância
15
entre os prédios é de 20 m e que o topo do segundo
prédio está 10 m abaixo do topo do primeiro. Nessas
condições, pode-se afirmar que essa cena poderia ser
real? (Considere a aceleração gravitacional igual a 10
m/s2
. Despreze a resistência do ar.)
a) Sim, pois o policial alcançaria o topo do segundo
prédio aproximadamente 8 m após a primeira borda
do prédio.
b) Não, pois com essa velocidade inicial, o policial
ultrapassaria o topo do segundo prédio.
c) Não, pois o policial cairia entre os prédios em queda
livre.
d) Não, pois o policial atingiria a parede lateral do
prédio em alguma altura do edifício.
e) Não, pois o policial alcançaria o topo do segundo
prédio a aproximadamente 0,5 m da segunda borda
do prédio, sem espaço suficiente para parar a moto.
Questão 16 - (UEM PR)
Uma pedra é lançada com um ângulo de 45º em relação
ao eixo horizontal x e na direção positiva de x.
Desprezando-se a resistência do ar, quais dos gráficos
melhor representam a componente horizontal da
velocidade (vx) versus tempo (t) e a componente vertical
da velocidade (vy) versus tempo (t), respectivamente?
I. II.
III. IV.
V.
vx versus t vy versus t
a) I e IV
b) II e I
c) II e III
d) II e V
e) IV e V
Questão 17 - (UEM PR/2008)
Um objeto ao nível do mar é lançado obliquamente com
velocidade inicial de 100,0 m/s, com um ângulo de
lançamento  tal que o 6,0)cos(  (obs.: despreze a
resistência do ar). Considere g = 10,0 m/s2
. Assinale o
que for correto.
01. As componentes horizontal e vertical da velocidade
no instante de lançamento são vx = 60,0 m/s e vy =
80,0 m/s.
02. Desprezando a resistência do ar, o objeto não
retorna ao nível de lançamento.
04. O alcance máximo do objeto é superior a 500 m.
08. O tempo necessário para o objeto atingir o alcance
máximo é 16,0 s.
16. O módulo da componente da velocidade no eixo
paralelo ao solo se mantém constante durante o
percurso.
Questão 18 - (UFG GO)
50 ANOS DE BOMBA ATÔMICA
As explosões das bombas atômicas, em agosto de 1945,
sobre as cidades de Hiroshima e Nagasaki, fizeram, em
1995, cinqüenta anos. O fim da guerra e o início da era
atômica se anunciavam e o clarão escurecia os olhares
do mundo que oscilavam entre o pavor e o medo. O
desconhecido explodia em solo japonês. A bomba
detonava os horrores da guerra, levantava a poeira
atômica das nações rivais e emitia sinais de que os
tempos seriam outros para as nações. Por certo, nunca
mais seriam os mesmos para os “filhos do clarão.”
Fontes: Superinteressante, ano 09,nº07, 1995; Veja,
02/08/95.
Considerando que, nas proposições a seguir, as
informações técnicas (nomes, distâncias, massas, etc.),
a respeito dos lançamentos e efeitos das bombas
atômicas sobre Hiroshima e Nagasaki, são verdadeiras,
analise as proposições, sob o aspecto da Física, e
identifiq44ue as correta.
01. o avião B-29, chamado “Enola Gay”, soltou a bomba
apelidada de “Little Boy” sobre Hiroshima. A bomba
foi solta a 9600m de altura e explodiu a
aproximadamente 580m do solo. Se a resistência do
ar fosse desprezada, a bomba levaria
aproximadamente 32 segundos para explodir
(adotar g = 10m/s2
);
02. o avião B-29, chamado “Bock’s Car”, soltou a
bomba apelidada de “Fat Man” acima de Nagasaki.
A intenção era atingir o centro da cidade, mas por
problemas de visibilidade atingiu a periferia. Ao
soltar a bomba o avião estava justamente acima do
ponto atingido;
04. pessoas que estavam distantes do ponto da
explosão, escutaram primeiramente o som e depois
viram o clarão da explosão;
08. “Little Boy”, de 4,0 toneladas, constituída de urânio,
possuía poder de destruição equivalente a 15.000
toneladas de TNT e “Fat Man”, de 4,5 toneladas,
constituída de plutônio, a 20.000 toneladas de TNT.
Se a resistência do ar fosse desprezada e as duas
bombas fossem soltas da mesma altura e no mesmo
instante, “Fat Man” atingiria o solo antes de “Little
Boy ”.
16. Segundo o físico Naomi Shohno, o deslocamento de
ar provocado pelas bombas percorreu 740m no
segundo posterior à explosão, 4 km em 10
segundos e 11 km em 30 segundos. Portanto, a
velocidade média de deslocamento do ar no
16
primeiro segundo foi de 2664 km/h e em 30
segundos foi de aproximadamente 1320km/h.
Questão 19 - (UFOP MG)
Uma partícula desloca-se em movimento retilíneo
uniforme sobre uma plataforma horizontal lisa do ponto
A ao ponto B, com velocidade v0 = 10m/s.
A partir do ponto B, a partícula se movimenta sob a ação
de seu peso até atingir o ponto D localizado em outra
plataforma horizontal, como mostra a figura abaixo.
a) Calcule a distância do ponto C ao ponto D.
b) Calcule o tempo que a partícula gasta para se
deslocar do ponto A ao ponto D.
c) Determine a velocidade da partícula imediatamente
antes de atingir o ponto D.
Dados: AB = 10m; BC = 20m; g = 10m/s2
.
Questão 20 - (ITA SP)
Uma bola é lançada horizontalmente do alto de um
edifício, tocando o solo decorridos aproximadamente 2s.
Sendo de 2,5m a altura de cada andar, o número de
andares do edifício é
a) 5
b) 6
c) 8
d) 9
e) indeterminado pois a velocidade horizontal de
arremesso da bola não foi fornecida.
GABARITO:
1) Gab: C
2) Gab: 01-04
3) Gab: E
4) Gab:
4
h
s

5) Gab: A
6) Gab: A
7) Gab: B
8) Gab: C
9) Gab: B
10) Gab: C
11) Gab: B
12) Gab: E
13) Gab: D
14) Gab: 23
15) Gab: A
16) Gab: C
17) Gab: 29
18) Gab: 01-F; 02-F; 04-F; 08-F; 16-V.
19) Gab:
a) 20m;
b) 3s;
c) vR = 22,4m/s
20) Gab: C
21) Gab: D
22) Gab: D
23) Gab: D
24) Gab: A
25) Gab: 17
26) Gab: C
27) Gab: B
28) Gab: D
29) Gab: 04-08-16
30) Gab: B
31) Gab: D
32) Gab: A
33) Gab: E
34) Gab: B
35) Gab: 05
36) Gab: D
37) Gab: 19
38) Gab: 31
39) Gab: B
40) Gab: 21
17
LISTA 4 - MOVIMENTO CIRCULAR
RESUMO TEÓRICO
O movimento circular uniforme (MCU) é o movimento no
qual o corpo descreve trajetória circular, podendo ser uma
circunferência ou um arco de circunferência. A velocidade
escalar permanece constante durante todo o trajeto e a
velocidade vetorial apresenta módulo constante, no entanto
sua direção é variável. A aceleração tangencial é nula (at =
0), porém, com a aceleração centrípeta não ocorre o
mesmo, ou seja, a aceleração não é nula (ac ≠ 0). A direção
da aceleração centrípeta, em cada ponto da trajetória, é
perpendicular à velocidade vetorial e aponta para o centro
da trajetória. O módulo da aceleração centrípeta é escrito
da seguinte forma: ac = v2
/r, onde r é o raio da
circunferência descrita pelo móvel.
Um corpo que descreve um movimento circular uniforme
passa de tempo em tempo no mesmo ponto da trajetória,
sempre com a mesma velocidade. Assim, podemos dizer
que esse movimento é repetitivo, e pode ser chamado de
movimento periódico. Nos movimentos
periódicos existem dois conceitos muito importantes que
são: frequência e período.
Frequência: é o número de voltas que o corpo efetua em
um determinado tempo (f = 1/ T).
Período: é o tempo gasto para se completar um ciclo (T =
1/ f).
Ao observar a definição de período e de frequência
podemos dizer que o período é o inverso da frequência.
Equações do Movimento Circular
As equações que determinam o movimento circular são as
seguintes:
Posição angular: S = φ .R, onde R é o raio da
circunferência.
Velocidade angular média: ωm = Δφ/Δt
Aceleração centrípeta: ac = v2
/R, onde R é o raio da
circunferência.
Força Centrípeta
Para que um móvel possa descrever o movimento circular
uniforme é necessário que esteja atuando uma força sobre
ele, de modo que faça com que ele mude de posição, pois
se tal fato não ocorrer o móvel passaria a descrever um
movimento retilíneo uniforme. Essa força tem o nome
de força centrípeta, e matematicamente é descrita da
seguinte forma:
Fc = m. ac
Onde ac é a aceleração centrípeta, ac = v2
/R. Substituindo
na equação acima temos:
Fc = m. v2
/R
A força centrípeta é sempre direcionada para o centro da
circunferência. No cotidiano existem alguns exemplos de
força centrípeta como a secadora de roupas e os satélites
que ficam em órbita circular em torno do centro da Terra.
Fonte:
http://www.brasilescola.com/fisica/movimento-
circular.htm
EXERCÍCIOS
Questão 01 - (UNIMAR SP)
Um ciclista descreve um movimento circular uniforme no
sentido anti-horário, conforme a trajetória abaixo. No
ponto X, o vetor aceleração é melhor ilustrado por:
.X
a) 
b) 
c) 
d) 
e) 
Questão 02 - (PUC RS)
A velocidade angular do movimento de rotação da Terra
é, aproximadamente,
a) (/12) rad/h
b) (/6) rad/h
c) (/4) rad/h
d)  rad/h
e) 2 rad/h
Questão 03 - (UECE)
Em um relógio, o período de rotação do ponteiro dos
segundos, o dos minutos e o das horas são,
respectivamente:
a) um segundo, um minuto e uma hora
b) um minuto, uma hora e um dia
c) um minuto, meia hora e um dia
d) um minuto, uma hora e meio dia
Questão 04 - (UFSC/2008)
Um carro com velocidade de módulo constante de 20
m/s percorre a trajetória descrita na figura, sendo que
de A a C a trajetória é retilínea e de D a F é circular, no
sentido indicado.
Assinale a(s) proposição(ões) CORRETA(S).
01. O carro tem movimento uniforme de A até C.
02. O carro tem movimento uniforme de A até F.
04. O carro tem aceleração de A até C.
08. O carro tem aceleração de D até F.
16. O carro tem movimento retilíneo uniformemente
variado de D até F.
Questão 05 - (UFU MG/2007)
Três rodas de raios Ra, Rb e Rc possuem velocidades
angulares wa, wb e wc, respectivamente, e estão ligadas
entre si por meio de uma correia, como ilustra figura
abaixo.
18
Ao mesmo tempo que a roda de raio Rb realiza duas
voltas, a roda de raio Rc realiza uma volta. Não há
deslizamento entre as rodas e a correia. Sendo ac R3R 
, é correto afirmar que:
a) caab w
3
4
weR
3
4
R  .
b) caab w3weR
3
4
R  .
c) caab w
3
4
weR
2
3
R  .
d) caab w3weR
2
3
R  .
Questão 06 - (UNINOVE SP/2009)
As rodas de um automóvel têm diâmetro de 60cm.
Quando o veículo transita a 36 km/h e suas rodas não
derrapam sobre o piso, a frequência com que elas giram
é, em Hz, de, aproximadamente,
a) 16,7.
b) 10,6.
c) 5,3.
d) 2,7.
e) 1,4.
Questão 07 - (UFT TO/2008)
Em uma aula de física, os alunos observam um objeto
descrevendo um movimento circular uniforme. Seja v

a
velocidade e v

a aceleração do objeto. Após observarem
o fenômeno, fazem os seguintes comentários:
I. No movimento circular uniforme a soma das forças
que agem no objeto não é nula, portanto existe
aceleração não nula.
II. No movimento circular uniforme v

muda
constantemente, enquanto a velocidade angular é
constante.
a) Os comentários I e II estão corretos.
b) Os comentários I e II estão errados.
c) Apenas o comentário I está correto.
d) Apenas o comentário II está correto.
Questão 08 - (ESPCEX/2009)
Uma máquina industrial é movida por um motor elétrico
que utiliza um conjunto de duas polias, acopladas por
uma correia, conforme figura abaixo. A polia de raio R1
= 15 cm está acoplada ao eixo do motor e executa 3000
rotações por minuto. Não ocorre escorregamento no
contato da correia com as polias. O número de rotações
por minuto, que a polia de raio R2 = 60 cm executa, é
de
Desenho Ilustrativo
a) 250
b) 500
c) 750
d) 1000
e) 1200
Questão 09 - (FAMECA SP/2010)
A relação entre as velocidades angulares de duas
pessoas paradas, em relação à Terra, uma sobre o
equador terrestre e outra, no polo norte, é
a) zero.
b) 1/24.
c) 1/12.
d) 1.
e) 6.
Questão 10 - (UNIMONTES MG/2010)
Na figura, estão representadas duas polias, A e B, com
raios RA < RB, acopladas por um eixo.
É CORRETO afirmar:
a) As velocidades angulares dos pontos periféricos da
polia A são iguais às dos pontos periféricos da polia
B.
b) As velocidades angulares dos pontos periféricos da
polia A são maiores do que as dos pontos
periféricos da polia B.
c) As velocidades lineares dos pontos periféricos da
polia A são iguais às dos pontos periféricos da polia
B.
d) As velocidades lineares dos pontos periféricos da
polia A são maiores do que as dos pontos
periféricos da polia B.
Questão 11 - (UEL)
Considere as seguintes afirmativas:
I. No movimento circular uniforme, os vetores
velocidade e aceleração são perpendiculares entre
si.
II. Objetos de mesma forma e dimensões, mas com
massas diferentes, quando soltos de uma mesma
altura, por estarem sob a influência da mesma
aceleração gravitacional, chegam ao solo no mesmo
instante.
III. Do ponto de vista microscópico, as forças
responsáveis pelo atrito entre duas superfícies são
as forças gravitacionais que atuam nas regiões em
que as duas superfícies estão em contato.
Assinale a alternativa correta.
19
a) Apenas as afirmativas I e III são verdadeiras.
b) Apenas as afirmativas II e III são verdadeiras.
c) Apenas as afirmativas I e II são verdadeiras.
d) Apenas a afirmativa III é verdadeira.
e) Todas as afirmativas são verdadeiras.
Questão 12 - (UEL PR)
A bicicleta tem o pedal preso a um disco denominado
“coroa”. A corrente liga a coroa à catraca, que é o disco
preso à roda traseira. A cada pedalada, a catraca gira
várias vezes, pois seu diâmetro é menor que o diâmetro
da coroa. Qual é a distância percorrida por uma bicicleta
de aro 33 (raio da roda igual a 33cm), cuja coroa tem
raio três vezes maior que o raio da catraca, no período
igual a uma pedalada?
a) 5,3 m
b) 5,7 m
c) 6,2 m
d) 6,8 m
e) 7,1 m
Questão 13 - (UEL PR/2006)
Os primeiros relógios baseavam-se no aparente
movimento do Sol na abóboda celeste e no
deslocamento da sombra projetada sobre a superfície de
um corpo iluminado pelo astro. Considere que: a Terra
é esférica e seu período de rotação é de 24 horas no
sentido oeste-leste; o tempo gasto a cada 15º de
rotação é de 1 hora; o triângulo Brasília/Centro da
Terra/Luzaka (Zâmbia) forma, em seu vértice central,
um ângulo de 75°.
A hora marcada em Luzaka, num relógio solar, quando
o sol está a pino em Brasília é:
a) 5 horas.
b) 9 horas.
c) 12 horas.
d) 17 horas.
e) 21 horas.
Questão 14 - (UEL PR/2010)
Considere uma esfera sólida de raio r e momento de
inércia inicial 2
ii mr
5
2
I  que gira com período T ao redor
de um eixo vertical que passa por seu centro. Essa
esfera possui matéria uniformemente distribuída através
de seu volume. Devido a um desequilíbrio de forças,
essa matéria rearranja-se em uma nova configuração de
equilíbrio cuja geometria é a de uma casca com formato
esférico e momento de inércia final 2
ff mr
3
2
I  .
Sob que condições o período de rotação da esfera
permanecerá inalterado?
a) Aumento na velocidade de rotação .
b) Esta condição será satisfeita se os raios iniciais e
finais forem iguais: ri = rf já que neste caso o
momento angular será conservado.
c) A conservação do momento angular implica em uma
diminuição da velocidade angular e ao mesmo
tempo um aumento no raio da esfera de forma que
if r
3
5
r  .
d) A conservação do momento angular implica em uma
diminuição do raio da esfera de forma que if r
3
2
r 
.
e) A conservação do momento angular implica em uma
diminuição do raio da esfera de forma que if r
5
3
r 
.
Questão 15 - (UEL PR/2010)
Um ciclista descreve uma volta completa em uma pista
que se compõe de duas retas de comprimento L e duas
semicircunferências de raio R conforme representado na
figura a seguir.
A volta dá-se de forma que a velocidade escalar média
nos trechos retos é v e nos trechos curvos é v
3
2 . O
ciclista completa a volta com uma velocidade escalar
média em todo o percurso igual a v
5
4 .
A partir dessas informações, é correto afirmar que o raio
dos semicírculos é dado pela expressão:
a) L = πR
b)
2
R
L


c)
3
R
L


d)
4
R
L


e)
2
R3
L


Questão 16 - (UEM PR)
Um carro se move com velocidade constante em uma
estrada curva num plano horizontal. Desprezando-se a
resistência do ar, pode-se afirmar corretamente que
sobre o carro atua;
01. uma força na mesma direção e em sentido contrário
ao centro da curva.
02. uma força de atrito na mesma direção e no mesmo
sentido do centro da curva.
04. uma força perpendicular à trajetória e dirigida para
cima.
20
08. uma força perpendicular à trajetória e dirigida para
baixo.
16. uma força na mesma direção e no mesmo sentido
do movimento do carro.
Questão 17 - (UEM PR)
Uma barra constituída de material isolante tem, em cada
extremidade, uma carga de 1 C. Se a barra girar em
torno de seu ponto médio com velocidade angular w = 8
 rad/s, podemos afirmar que
01. a barra completará quinze voltas em cada
segundo.
02. a corrente elétrica proporcionada pelo
movimento da barra será de 8 A.
04. o período das oscilações da barra será de
0,13 segundos.
08. a barra não executará um M.H.S..
16. a corrente elétrica terá dimensão de
segundo por Coulomb.
Questão 18 - (UEM PR)
Das afirmativas a seguir, assinale o que for correto.
01. Quando um móvel executa um movimento circular
uniforme, sua aceleração é nula.
02. No movimento circular uniforme, a frequência é
constante.
04. No movimento circular uniforme, o vetor velocidade
tangencial é variável.
08. A forma angular da equação horária do movimento
circular uniforme é t0  , onde  é a posição
angular do móvel no instante t, 0 é a posição
angular do móvel no instante 0t0  e  é a
velocidade angular do móvel.
16. A frequência é inversamente proporcional ao
quadrado do período.
32. Quando um ponto material percorre uma
circunferência em movimento circular uniforme, a
projeção do ponto material sobre um diâmetro da
circunferência realiza um movimento harmônico
simples.
Questão 19 - (UEM PR)
Imagine que você esteja em um carrossel de parque de
diversões que gira em um movimento circular uniforme.
A figura abaixo representa o carrossel visto de cima. O
brinquedo gira sempre paralelo ao chão, sem
movimentos verticais. Imagine agora que você lança, do
ponto P, uma chave para um amigo parado a uma certa
distância do brinquedo. Em que posição deveria estar
esse amigo para apanhar a chave? (Despreze a
resistência do ar.)
a) I
b) II
c) III
d) IV
e) V
Questão 20 - (UEM PR/2009)
Duas polias, A e B, de raios R1 = 10cm e R2 = 20cm,
giram acopladas por uma correia de massa desprezível
que não desliza, e a polia A gira com uma frequência de
rotação de 20 rpm. Assinale a(s) alternativa(s)
correta(s).
01. A velocidade de qualquer ponto P da correia é
aproximadamente 0,21 m/s.
02. A frequência angular de rotação da polia B é 2,0
rad/s.
04. A razão entre as frequências de rotação das polias
A e B é 2.
08. O período de rotação da polia A é 3,0 s.
16. A aceleração centrípeta experimentada por uma
partícula de massa m, colocada na extremidade da
polia A (borda mais externa), é maior do que se a
mesma partícula fosse colocada na extremidade da
polia B.
GABARITO:
1) Gab: D 2) Gab: A 3) Gab: D
4) Gab: 11 5) Gab: D 6) Gab: C
7) Gab: A 8) Gab: C 9) Gab: D
10) Gab: A 11) Gab: C 12) Gab: C
13) Gab: D 14) Gab: E 15) Gab: A
16) Gab: 02-04-08 17) Gab: 10
18) Gab: 46 19) Gab: C 20) Gab: 29
21) Gab: E 22) Gab: B 23) Gab: C
24) Gab: C 25) Gab: A 26) Gab: E
27) Gab: 02 28) Gab: 51 29) Gab: A
30) Gab: D 31)
3
1

B
A
I
I 32) Gab: E
33) Gab: B 34) Gab: B 35) Gab: 29
36) Gab: E
37) Gab:
a) f = 3,3 rad/s
b) tf = 3s
c) c = 3 rad/s
d)  = 4,1 rad
38) Gab: 14 39) Gab: C 40) Gab: FFVF
41) Gab: A
21
LISTA 5 - VETORES E CINEMÁTICA VETORIAL
RESUMO TEÓRICO
Vetores
Determinado por um segmento orientado AB, é o conjunto
de todos os segmentos orientados equipolentes a AB.
Se indicarmos com este conjunto, simbolicamente
poderemos escrever:
onde XY é um segmento qualquer do conjunto.
O vetor determinado por AB é indicado por ou B - A
ou .
Um mesmo vetor é determinado por uma infinidade
de segmentos orientados, chamados representantes desse
vetor, os quais são todos equipolentes entre si. Assim, um
segmento determina um conjunto que é o vetor, e qualquer
um destes representantes determina o mesmo vetor.
Usando um pouco mais nossa capacidade de abstração, se
considerarmos todos os infinitos segmentos orientados de
origem comum, estaremos caracterizando, através de
representantes, a totalidade dos vetores do espaço. Ora,
cada um destes segmentos é um representante de um só
vetor. Consequentemente, todos os vetores se acham
representados naquele conjunto que imaginamos.
As características de um vetor são as mesmas de
qualquer um de seus representantes, isto é: o módulo, a
direção e o sentido do vetor são o módulo, a direção e o
sentido de qualquer um de seus representantes.
O módulo de se indica por | | .
Soma de vetores
Se v=(a,b) e w=(c,d), definimos a soma de v e w, por:
v + w = (a+c,b+d)
Propriedades da Soma de vetores
Diferença de vetores
Se v=(a,b) e w=(c,d), definimos a diferença entre v e w,
por:
v - w = (a-c,b-d)
Produto de um número escalar por um vetor
Se v=(a,b) é um vetor e c é um número real, definimos a
multiplicação de c por v como:
c.v = (ca,cb)
Propriedades do produto de escalar por vetor
Quaisquer que sejam k e c escalares, v e w vetores:
Módulo de um vetor
O módulo ou comprimento do vetor v=(a,b) é um número
real não negativo, definido por:
Vetor unitário
Vetor unitário é o que tem o módulo igual a 1.
Existem dois vetores unitários que formam a base
canônica para o espaço R², que são dados por:
i = (1,0) j = (0,1)
Para construir um vetor unitário u que tenha a mesma
direção e sentido que um outro vetor v, basta dividir o
vetor v pelo seu módulo, isto é:
Observação:
Para construir um vetor u paralelo a um vetor v, basta
tomar u=cv, onde c é um escalar não nulo. Nesse
caso, u e v serão paralelos:
Se c = 0, então u será o vetor nulo.
Se 0 < c < 1, então u terá comprimento menor do que v.
Se c > 1, então u terá comprimento maior do que v.
Se c < 0, então u terá sentido oposto ao de v.
Decomposição de vetores em Vetores Unitários
Para fazer cálculos de vetores em apenas um dos planos
em que ele se apresenta, pode-se decompor este vetor em
vetores unitários em cada um dos planos apresentados.
Sendo simbolizados, por convenção, î como vetor unitário
do plano x e como vetor unitário do plano y. Caso o
problema a ser resolvido seja dado em três dimensões, o
vetor utilizado para o plano z é o vetor unitário .
22
Então, a projeção do vetor no eixo x do plano cartesiano
será dado por , e sua projeção no eixo ydo plano
será: . Este vetor pode ser escrito como:
=( , ), respeitando que sempre o primeiro
componente entre parênteses é a projeção em x e o
segundo é a projeção no eixo y. Caso apareça um terceiro
componente, será o componente do eixo z.
No caso onde o vetor não se encontra na origem, é possível
redesenhá-lo, para que esteja na origem, ou então
descontar a parte do plano onde o vetor não é projetado.
Fonte:
http://www.sofisica.com.br/conteudos/Mecanica/Cinematica
/Vetores.php
EXERCÍCOS
Questão 01 - (UDESC/2012)
Observando o movimento de um carrossel no parque de
diversões, conclui-se que seu movimento é do tipo
circular uniforme.
Assinale a alternativa correta em relação ao
movimento.
a) Não é acelerado porque o módulo da velocidade
permanece constante.
b) É acelerado porque o vetor velocidade muda de
direção, embora mantenha o mesmo módulo.
c) É acelerado porque o módulo da velocidade varia.
d) Não é acelerado porque a trajetória não é retilínea.
e) Não é acelerado porque a direção da velocidade
não varia.
Questão 02 - (MACK SP/2012)
Um avião, após deslocar-se 120 km para nordeste (NE),
desloca-se 160 km para sudeste (SE). Sendo um quarto
de hora, o tempo total dessa viagem, o módulo da
velocidade vetorial média do avião, nesse tempo, foi de
a) 320 km/h
b) 480 km/h
c) 540 km/h
d) 640 km/h
e) 800 km/h
Questão 03 - (FMTM MG/2006)
A figura apresenta uma “árvore vetorial” cuja resultante
da soma de todos os vetores representados tem módulo,
em cm, igual a
a) 8.
b) 26.
c) 34.
d) 40.
e) 52.
Questão 04 - (UNIFICADO RJ)
Considere um helicóptero movimentando-se no ar em
três situações diferentes:
I. Subindo verticalmente com velocidade constante;
II. Descendo verticalmente com velocidade constante;
III. Deslocando-se horizontalmente para a direita, com
velocidade constante.
A resultante das forças exercidas pelo ar sobre o
helicóptero, em cada uma dessas situações, é
corretamente representada por:
a.
b.
c.
d.
e.
 

 


I II III
Questão 05 - (MACK SP/2007)
O movimento de uma partícula é caracterizado por ter
vetor velocidade e vetor aceleração não nulo de mesma
direção. Nessas condições, podemos afirmar que esse
movimento é
a) uniforme.
b) uniformemente variado.
c) harmônico simples.
d) circular uniforme.
e) retilíneo.
Questão 06 - (PUC MG)
Marque a alternativa INCORRETA.
a) Quando o módulo da velocidade é constante, a
aceleração pode não ser nula.
b) Quando a aceleração é nula, o módulo da velocidade
é constante.
c) A componente de um vetor é um vetor.
d) Quando um vetor é nulo, ele pode ter componentes
não nulas.
Questão 07 - (UNIUBE MG)
Em uma pista de Fórmula Indy, um piloto faz o trajeto
CD e FA com movimentos retardado, os trajetos AB e
DE com movimento acelerado e os trechos BC e EF com
movimento uniforme. A alternativa que mostra o vetor
aceleração resultante corretamente, supondo que o
movimento se realize na ordem alfabética, é:
23
F E
B C
A D
a.
F E
B C
A D
b.
F E
B C
A D
c.
F E
B C
A D
d.
A
F E
B C
D
e.
Questão 08 - (PUC MG)
Você e um amigo resolvem ir ao último andar de um
edifício. Vocês partem juntos do primeiro andar.
Entretanto, você vai pelas escadas e seu amigo, pelo
elevador. Depois de se encontrarem na porta do
elevador, descem juntos pelo elevador até o primeiro
andar. É CORRETO afirmar que:
a) o seu deslocamento foi maior que o de seu amigo.
b) o deslocamento foi igual para você e seu amigo.
c) o deslocamento de seu amigo foi maior que o seu.
d) a distância que seu amigo percorreu foi maior que
a sua.
Questão 09 - (UFSC)
Assinale a(s) afirmativa correta, some os valores
respectivos e marque o resultado no cartão-resposta.
A representação correta da velocidade tangencial “ v

”,
da quantidade de movimento “ q

”, da aceleração
centrípeta “ a

” e da força normal “ F

”, atuantes sobre
um ponto material de massa m que descreve um
movimento circular uniforme (MCU), está(ão)
corretamente representada(s) no(s) esquema(s)
01.
F
a
0
q
v
02.
F
a
0
q
v
04.
F
a
0
q v
08.
Fa
0
q
v
16.
F
a
0
q
v 32.
F
a
0
q
v
64. Fa
0
q
v
Questão 10 - (CESGRANRIO RJ)
A figura a seguir mostra a fotografia estroboscópica do
movimento de uma partícula.
V
P
IV III
II
I
A aceleração da partícula, no ponto P da trajetória, é
melhor representada pelo vetor:
a) I;
b) II;
c) III;
d) IV;
e) V.
Questão 11 - (UEL PR/2008)
Um ciclista percorre as rotas 1 e 2 para se deslocar do
ponto A ao ponto B, como mostrado no mapa a seguir,
e registra em cada uma a distância percorrida.
Assinale a alternativa que apresenta os valores
aproximados da distância percorrida na rota 1 e na rota
2.
Considere como aproximação todos os quarteirões
quadrados com 100 m de lado. As rotas 1 e 2
encontram-se pontilhadas.
a) rota 1  800 m;
rota 2  800 m.
b) rota 1  700 m;
rota 2  700 m.
c) rota 1  800 m;
rota 2  900 m.
d) rota 1  900 m;
rota 2  700 m.
e) rota 1  900 m;
24
rota 2  600 m
Questão 12 - (UEL PR/2013)
Uma família viaja para Belém (PA) em seu automóvel.
Em um dado instante, o GPS do veículo indica que ele
se localiza nas seguintes coordenadas: latitude 21°20’
Sul e longitude 48°30’ Oeste. O motorista solicita a um
dos passageiros que acesse a Internet em seu celular e
obtenha o raio médio da Terra, que é de 6730 km, e as
coordenadas geográficas de Belém, que são latitude
1°20’ Sul e longitude 48°30’ Oeste. A partir desses
dados, supondo que a superfície da Terra é esférica, o
motorista calcula a distância D, do veículo a Belém,
sobre o meridiano 48°30’Oeste.
Assinale a alternativa que apresenta, corretamente, o
valor da distância D, emkm.
a) 6730
9
D


b) 2
)6730(
18
D


c) 6730
9
D


d) 6730
36
D


e) 6730
3
D
2





 

Questão 13 - (PUCCAMP SP/2011)
Analise o esquema abaixo.
O vetor resultante ou soma vetorial das três medidas
acima representadas tem módulo
a) 11
b) 13
c) 15
d) 17
e) 19
Questão 14 - (UNESP/2012)
No dia 11 de março de 2011, o Japão foi sacudido por
terremoto com intensidade de 8,9 na Escala Richter,
com o epicentro no Oceano Pacífico, a 360 km de
Tóquio, seguido de tsunami. A cidade de Sendai, a 320
km a nordeste de Tóquio, foi atingida pela primeira
onda do tsunami após 13 minutos.
(O Estado de S.Paulo, 13.03.2011. Adaptado.)
Baseando-se nos dados fornecidos e sabendo que cos 
 0,934, onde  é o ângulo Epicentro-Tóquio-Sendai, e
que 28
 32
 93,4  215 100, a velocidade média, em
km/h, com que a 1.ª onda do tsunami atingiu até a
cidade de Sendai foi de:
a) 10.
b) 50.
c) 100.
d) 250.
e) 600.
Questão 15 - (UFSCar SP)
Nos esquemas estão representadas a velocidade V e a
aceleração a do ponto material P. Assinale a
alternativa em que o módulo da velocidade desse ponto
material permanece constante.
a.
P v
a


b.
P
a
v

c.
P
a
v

.
d.
P
a v
. e.
P v
a


Questão 16 - (UNIFESP SP/2002)
Na figura, são dados os vetores a , b e c
Sendo u a unidade de medida do módulo desses vetores,
pode-se afirmar que o vetor cbad  :tem módulo
a) 2u, e sua orientação é vertical, para cima.
b) 2u, e sua orientação é vertical, para baixo.
c) 4u, e sua orientação é horizontal, para a direita.
d) 2 u, e sua orientação forma 45º com a horizontal,
no sentido horário.
e) 2 u, e sua orientação forma 45º com a horizontal,
no sentido anti-horário.
Questão 17 - (UNIFOR CE)
Numa sala cúbica, de aresta a, uma mosca voa numa
diagonal (segmento que une dois vértices, passando
25
pelo centro a sala). O deslocamento da mosca tem
módulo
a) a
b) 2a
c) 3a
d) a
4
9
e) 3a
Questão 18 - (FUVEST SP/2002)
Em uma estrada, dois carros, A e B, entram
simultaneamente em curvas paralelas, com raios RA e
RB. Os velocímetros de ambos os carros indicam, ao
longo de todo o trecho curvo, valores constantes VA e
VB. Se os carros saem das curvas ao mesmo tempo, a
relação entreVA e VB é:
a) VA = VB
b) VA/VB = RA/ RB
c) VA/VB = (RA/ RB )2
d) VA/VB = RB/ RA
e) VA/VB = (RB/ RA)2
Questão 19 - (UFOP MG)
A velocidade do centro de massa de um disco que rola
sem deslizar, em um plano horizontal é v. As
velocidades dos pontos A e B, mostrados na figura são,
respectivamente.
O
V
A
B
X
Y 
a) – v e v
b) 0 e v
c) 0 e 2v
d) v e v
e) v e 2v
Questão 20 - (MACK SP)
Um corpo é atirado verticalmente para cima a partir do
solo com velocidade inicial de módulo 50 m/s. O
módulo de sua velocidade vetorial média entre o
instante de lançamento e o instante em que retorna ao
solo é:
a) 50 m/s;
b) 25 m/s;
c) 5,0 m/s;
d) 2,5 m/s;
e) zero.
GABARITO:
1) Gab: B 2) Gab: E 3) Gab: C 4) Gab: A
5) Gab: E 6) Gab: D 7) Gab: B 8) Gab: B
9) Gab: 04 10) Gab: B 11) Gab: D 12) Gab: A
13) Gab: B 14) Gab: E 15) Gab: C 16) Gab: B
17) Gab: C 18) Gab: B 19) Gab: C 20) Gab: E
21) Gab: C 22) Gab: D 23) Gab: B 24) Gab: B
25) Gab: E 26) Gab: D 27) Gab: E 28) Gab: E
29) Gab: E 30) Gab: A 31) Gab: C 32) Gab: D
33) Gab: E 34) Gab: D 35) Gab: E 36) Gab: 25
37) Gab: A 38) Gab: 07 39) Gab: 03
40) Gab: 25
26
LISTA 6 - LEIS DE NEWTON
RESUMO TEÓRICO
DINÂMICA
Quando se fala em dinâmica de corpos, a imagem que vem
à cabeça é a clássica e mitológica de Isaac Newton, lendo
seu livro sob uma macieira. Repentinamente, uma maçã cai
sobre a sua cabeça. Segundo consta, este foi o primeiro
passo para o entendimento da gravidade, que atraia a
maçã.
Com o entendimento da gravidade, vieram o entendimento
de Força, e as três Leis de Newton.
Na cinemática, estuda-se o movimento sem compreender
sua causa. Na dinâmica, estudamos a relação entre a força
e movimento.
Força: É uma interação entre dois corpos.
O conceito de força é algo intuitivo, mas para compreendê-
lo, pode-se basear em efeitos causados por ela, como:
Aceleração: faz com que o corpo altere a sua velocidade,
quando uma força é aplicada.
Deformação: faz com que o corpo mude seu formato,
quando sofre a ação de uma força.
Força Resultante: É a força que produz o mesmo efeito que
todas as outras aplicadas a um corpo.
Dadas várias forças aplicadas a um corpo qualquer:
A força resultante será igual a soma vetorial de todas as
forças aplicadas:
Leis de Newton
As leis de Newton constituem os três pilares fundamentais
do que chamamos Mecânica Clássica, que justamente por
isso também é conhecida por Mecânica Newtoniana.
1ª Lei de Newton - Princípio da Inércia
 Quando estamos dentro de um carro, e este
contorna uma curva, nosso corpo tende a
permanecer com a mesma velocidade vetorial a
que estava submetido antes da curva, isto dá a
impressão que se está sendo "jogado" para o lado
contrário à curva. Isso porque a velocidade vetorial
é tangente a trajetória.
 Quando estamos em um carro em movimento e
este freia repentinamente, nos sentimos como se
fôssemos atirados para frente, pois nosso corpo
tende a continuar em movimento.
estes e vários outros efeitos semelhantes são explicados
pelo princípio da inércia, cujo enunciado é:
"Um corpo em repouso tende a permanecer em
repouso, e um corpo em movimento tende a
permanecer em movimento."
Então, conclui-se que um corpo só altera seu estado de
inércia, se alguém, ou alguma coisa aplicar nele uma força
resultante diferente se zero.
2ª Lei de Newton - Princípio Fundamental da
Dinâmica
Quando aplicamos uma mesma força em dois corpos de
massas diferentes observamos que elas não produzem
aceleração igual.
A 2ª lei de Newton diz que a Força é sempre
diretamente proporcional ao produto da aceleração
de um corpo pela sua massa, ou seja:
ou em módulo: F=ma
Onde:
F é a resultante de todas as forças que agem sobre o corpo
(em N);
m é a massa do corpo a qual as forças atuam (em kg);
a é a aceleração adquirida (em m/s²).
A unidade de força, no sistema internacional, é o N
(Newton), que equivale a kg m/s² (quilograma metro por
segundo ao quadrado).
3ª Lei de Newton - Princípio da Ação e Reação
Quando uma pessoa empurra um caixa com um força F,
podemos dizer que esta é uma força de ação. mas
conforme a 3ª lei de Newton, sempre que isso ocorre, há
uma outra força com módulo e direção iguais, e sentido
oposto a força de ação, esta é chamada força de reação.
Esta é o princípio da ação e reação, cujo enunciado é:
"As forças atuam sempre em pares, para toda força
de ação, existe uma força de reação."
EXERCÍCIOS
TEXTO: 1 - Comum à questão: 1
Considere as Leis de Newton e as informações a seguir.
Uma pessoa empurra uma caixa sobre o piso de uma
sala. As forças aplicadas sobre a caixa na direção do
movimento são:
- Fp: força paralela ao solo exercida pela pessoa;
- Fa: força de atrito exercida pelo piso.
A caixa se desloca na mesma direção e sentido de Fp .
A força que a caixa exerce sobre a pessoa é Fc .
Questão 01 - (UERJ/2012)
Se o deslocamento da caixa ocorre com velocidade
constante, as magnitudes das forças citadas
apresentam a seguinte relação:
a) Fp = Fc = Fa
b) Fp > Fc = Fa
c) Fp = Fc > Fa
d) Fp = Fc < Fa
Questão 02 - (UPE/2012)
Um corpo de massa m está suspenso por duas molas
ideais, paralelas, com constantes elásticas k e
27
deformadas de d. Sabendo que o sistema se encontra
em equilíbrio, assinale a alternativa que expressa k.
Dado: Considere a aceleração da gravidade g.
a)
d
mg2
b)
d
mg
c)
d
mg
2
d)
mg
d2
e)
mg
d
Questão 03 - (UFJF MG)
Colocando-se sobre as bordas de uma mesa dois lápis
L1 e L2 de modo que uma parte deles sobressaia da
mesa, tal que possam sustentar um palito de churrasco
c1 como mostra a figura, golpeando-se rapidamente o
palito de churrasco, ele se quebra e os lápis não. Este
fenômeno está relacionado com:
L1
L2
C1
a) O princípio da inércia.
b) A conservação do momento.
c) A conservação da energia.
d) A conservação do momento e energia.
e) A lei de conservação dos choques.
Questão 04 - (UFOP MG)
Quando um ônibus é freado bruscamente, todos os
passageiros são lançados” para a frente. Explique
fisicamente este fenômeno.
Questão 05 - (UFSC)
Uma mola comprimida no interior de um tubo cilíndrico
impulsiona uma bola, projetando–a horizontalmente
para fora do tubo. Desprezando–se a resistência do ar,
o esquema que representa corretamente a(s) força(s)
atuante(s) sobre a bola fora do tubo é:
01.
02.
04.
08.
16.
Questão 06 - (UFU MG/2011)
Um objeto é lançado verticalmente na atmosfera
terrestre. A velocidade do objeto, a aceleração
gravitacional e a resistência do ar estão representadas
pelos vetores v

, g

e atritoF

, respectivamente.
Considerando apenas estas três grandezas físicas no
movimento vertical do objeto, assinale a alternativa
correta.
a)
b)
c)
d)
Questão 07 - (UFTM/2007)
As confusões entre os conceitos de peso e massa são
muitas, mas sabemos que em um corpo qualquer, essas
grandezas são distintas. Analise as afirmações:
I. A massa de um corpo é uma medida da inércia
desse corpo.
II. Peso está relacionado à força com que a Terra atrai
um corpo.
III. O peso é uma grandeza vetorial. Já a massa é uma
grandeza escalar.
É correto o contido em
a) I, apenas.
b) I e II, apenas.
c) I e III, apenas.
d) II e III, apenas.
e) I, II e III.
Questão 08 - (MACK SP/2008)
No sistema a seguir, o fio e a polia são considerados
ideais e o atrito entre as superfícies em contato é
desprezível. Abandonando-se o corpo B a partir do
repouso, no ponto M, verifica-se que, após 2 s, ele passa
pelo ponto N com velocidade de 8 m/s. Sabendo-se que
a massa do corpo A é de 5 kg, a massa do corpo B é
28
a) 1 kg
b) 2 kg
c) 3 kg
d) 4 kg
e) 5 kg
Questão 09 - (UFLA MG)
Dois corpos têm massas m1 e m2. Aplicando-se as forças
F1

e F2

em m1 e m2, estas passam a ter acelerações
a1

e a2

, respectivamente. Como resultados possíveis
para as acelerações, teremos, EXCETO:
a) Se
a2a1, entãom2m1eF2F1


b) Se
aa, entãommeFF 212121


c) Se
a2a1, entãom2m1eF2F1


d) Se
aa, entãommeFF 212121


e) Se aa, entãommeFF 212121


Questão 10 - (UNIFOR)
Num anel atuam simultaneamente três forças
coplanares, 1F , 2F e 3F , representadas abaixo.
A resultante 321 FFFR  tem módulo, em newtons,
a) 11
b) 9,0
c) 8,0
d) 7,0
e) 5,0
Questão 11 - (UNIOESTE PR)
O equilíbrio é uma situação física comum no nosso
cotidiano. Os engenheiros, por exemplo, ao elaborarem
muitos de seus projetos, estão constantemente atentos
para atender adequadamente às condições necessárias
e suficientes para que o equilíbrio ocorra. Assinale, entre
as alternativas a seguir, aquela que apresenta um corpo
em equilíbrio.
a) Um brinquedo em movimento circular uniforme,
preso a uma corda.
b) Um satélite em órbita em torno da Terra.
c) Um livro no ponto mais alto da trajetória, quando
lançado verticalmente para cima por um aluno.
d) Uma bola que se movimenta em uma trajetória
parabólica, após ter sido chutada pelo goleiro em
um jogo de futebol.
e) Um elevador em movimento vertical com velocidade
constante.
Questão 12 - (UNIOESTE PR)
São fornecidas abaixo diversas afirmativas, envolvendo
conceitos de mecânica. Assinale a(s) alternativa(s)
correta(s).
01. Um objeto, inicialmente em repouso, recebe um
impulso e adquire velocidade. Após um curto intervalo
de tempo, o impulso deixa de atuar. Pode-se afirmar
que, a partir deste momento, como consequência do
impulso, o objeto passa a ter uma força, a qual, aos
poucos, vai cessando, até que o objeto volte ao
repouso. Considere que o objeto não sofre nenhum
outro tipo de interação.
02. Um objeto não pode realizar uma trajetória curva
com velocidade escalar constante, caso a soma
vetorial de todas as forças que sobre ele atuam seja
nula.
04. A velocidade de um projétil, sujeito apenas ao
campo gravitacional da Terra, lançado
obliquamente, aumenta até atingir um valor
máximo, após o que o projétil se desloca
verticalmente, de volta para o chão.
08. É possível exercer uma força sobre um objeto sem
que haja realização de trabalho.
16. Um objeto que está em repouso não pode estar
sendo submetido à ação da gravidade.
32. A atração gravitacional entre dois objetos leva um
certo tempo para ocorrer, não sendo instantânea.
64. Para que um objeto esteja em movimento, não é
necessário haver uma força aplicada sobre ele.
Questão 13 - (UNIOESTE PR)
No teto de um elevador, está pendurado um
dinamômetro que tem, na sua outra extremidade, um
pequeno corpo de peso 1,6 N. O dinamômetro, no
entanto, acusa 2,0 N. O elevador está
a) subindo com velocidade constante.
b) em repouso.
c) descendo com velocidade constante.
d) subindo com velocidade crescente.
e) descendo com velocidade crescente.
Questão 14 - (UNIOESTE PR)
O bloco A sobre uma mesa tem massa 15,0kg, e o bloco
B suspenso tem massa 5,0kg. O coeficiente de atrito
entre o bloco A e a mesa vale 0,20. Considere que o fio
utilizado para unir os blocos é ideal (sem peso e
inextensível) e que a aceleração gravitacional vale
10,0m/s2
.
Analise as afirmações e assinale-as devidamente.
29
00. A aceleração adquirida pelos corpos, durante seus
movimentos, é de 1,0 m/s2
.
01. No fio ideal, a tração é de 45,0N.
02. A velocidade do bloco A, após 3,0s de iniciado o
movimento, é de 30,0 m/s.
03. A distância percorrida pelo bloco A, após 3,0 s de
iniciado o movimento, é de 4,5 m.
04. Os blocos A e B têm acelerações diferentes.
Questão 15 - (UEL PR)
“Em média, as grandes usinas geradoras brasileiras
precisam de 2180 litros de água para gerar 1MW. A
usina hidrelétrica de Três Marias tem 396 MW de
potência instalada. Para gerar 1MW ela precisa que 2325
litros passem por suas turbinas. Quando 920 mil litros
passam pelas suas turbinas, ela gera 396 MW, que é sua
potência instalada. Qualquer quantidade superior de
água não gerará mais energia, pois o máximo que a
hidrelétrica pode produzir é 396MW.”
Matéria publicada na Folha de São Paulo do dia 22 de abril
de 2001, página B4.
A água que desce da represa para a usina é usada com
o objetivo de acionar as turbinas dos geradores de
eletricidade. Na usina de Três Marias, quando 920 mil
litros de água chegam na entrada das turbinas, é correto
afirmar:
a) A velocidade será muito maior que a velocidade de
2325 litros de água.
b) A velocidade será igual à velocidade de 2325 litros
de água.
c) A velocidade será menor que a velocidade de 2325
litros de água.
d) Não se pode comparar 920 mil litros de água com
2325 litros de água, porque as suas acelerações
serão diferentes.
e) É necessário calcular a aceleração de cada uma das
quantidades de água envolvidas, para se proceder
à comparação.
Questão 16 - (UEL PR)
Uma pessoa apoia-se em um bastão sobre uma balança,
conforme a figura abaixo. A balança assinala 70kg. Se a
pessoa pressiona a bengala, progressivamente, contra a
balança, a nova leitura:
a) Indicará um valor maior que 70kg.
b) Indicará um valor menor que 70kg.
c) Indicará os mesmos 70kg.
d) Dependerá da força exercida sobre o bastão.
e) Dependerá do ponto em que o bastão é apoiado na
balança.
Questão 17 - (UEL PR)
O cabo de um reboque arrebenta se nele for aplicada
uma força que exceda 1800N. Suponha que o cabo seja
usado para rebocar um carro de 900kg ao longo de uma
rua plana e retilínea. Nesse caso, que aceleração
máxima o cabo suporta?
a) 0,5m/s2
.
b) 1,0m/s2
.
c) 2,0m/s2
.
d) 4,0m/s2
.
e) 9,0m/s2
.
Questão 18 - (UEL PR)
Um jogador de tênis, ao acertar a bola com a raquete,
devolve-a para o campo do adversário. Sobre isso, é
correto afirmar:
a) De acordo com a Segunda Lei de Newton, a força
que a bola exerce sobre a raquete é igual, em
módulo, à força que a raquete exerce sobre a bola.
b) De acordo com a Primeira Lei de Newton, após o
impacto com a raquete, a aceleração da bola é
grande porque a sua massa é pequena.
c) A força que a raquete exerce sobre a bola é maior
que a força que a bola exerce sobre a raquete,
porque a massa da bola é menor que a massa da
raquete.
d) A bola teve o seu movimento alterado pela raquete.
A Primeira Lei de Newton explica esse
comportamento.
e) Conforme a Segunda Lei de Newton, a raquete
adquire, em módulo, a mesma aceleração que a
bola.
Questão 19 - (UEM PR)
Uma massa m é presa a uma associação de duas molas
ideais 1 e 2, de constantes elásticas, respectivamente,
k1 e k2, como ilustradas na figura abaixo.
Considerando-se x1 e x2, respectivamente, os
deslocamentos das posições de equilíbrio das molas 1 e
2, e kE a constante elástica resultante da associação das
molas, podemos afirmar que:
01. k2 x2 = k1 x1.
02. k2 x2 = mg.
04. kE = k1 + k2.
08. kE x = mg, sendo x = x1 + x2.
16. – kE x = resultante das forças que atua na massa
m, sendo x = x1 + x2.
Questão 20 - (UEM PR)
Através de uma polia ideal, passa uma corda C1, que
sustenta duas massas, m1 e m2. Outra corda, C2, presa
a m2, sustenta uma massa m3, conforme a figura.
Considere as cordas idênticas e ideais. Considere
também que a tração em C1 é T1, e em C2 é T2. Se m1
m2  m3, pode-se afirmar corretamente que:
30
01. quando m3 + m2 = m1, a aceleração do sistema é
nula.
02. quando m3 + m2 = m1, T1 = T2.
04. quando m3 + m2 = 2m1, a aceleração do sistema é
g/2.
08. invertendo–se a posição das massas m2 e m3, a
aceleração do sistema não se altera.
16. quando m3 > m2, T2 > T1.
GABARITO:
1) Gab: A 2) Gab: C 3) Gab: A
4) Gab: inércia 5) Gab: 08 6) Gab: A
7) Gab: E 8) Gab: C 9) Gab: D
10) Gab: E 11) Gab: E 12) Gab: 02+08+64
13) Gab: D 14) Gab: VVFVF 15) Gab: B
16) Gab: C 17) Gab: C 18) Gab: D
19) Gab: 01-02-08 20) Gab: 01-08
21) Gab: E 22) Gab: 17 23) Gab: A
24) Gab: 02+04+08+16 25) Gab: 01-02-04-08
26) Gab: 4,5 27) Gab: C 28) Gab: A
29) Gab: C 30) Gab: B 31) Gab: B
32) Gab: 17 33) Gab: A 34) Gab: B
35) Gab: A 36) Gab: C 37) Gab: A
38) Gab: D 39) Gab: C 40) Gab: B
31
LISTA 7 - APLICAÇÕES DAS LEIS DE NEWTON
RESUMO TEÓRICO
A maior dificuldade de estudantes de física do ensino médio
é com relação à solução de problemas, nas mais diversas
frentes da disciplina.
Apresentarei aqui a solução de um exercício sobre interação
entre blocos, relacionado a aplicação das Leis de Newton.
Exemplo 1) (UF-PE) A figura abaixo mostra três blocos de
massas mA = 1,0 kg, mB = 2,0 kg e mc = 3,0 kg. Os
blocos se movem em conjunto, sob a ação de uma força F
constante e horizontal, de módulo 4,2 N.
Desprezando o atrito, qual o módulo da força resultante
sobre o bloco B?
a) 1,0 N
b) 1,4 N
c) 1,8 N
d) 2,2 N
e) 2,6 N
Solução.
Primeiramente devemos saber que F = m.a (Segunda lei de
Newton). Devemos saber também, que os três corpos se
movem com a mesma aceleração, e que essa aceleração
tem a mesma direção e sentido da força F.
Assim podemos calcular a aceleração dos blocos pelo
Sistema de Corpos Isolados (SCI)
Na figura abaixo representamos as forças que agem em A,
B e C
Em que F é a força aplicada.
Fa no primeiro bloco é a reação de b em a devido a F.
Fa no segundo bloco é a ação de a em b devido a F
Fc no segundo bloco é a reação de c em b devido a F
Fc no terceiro bloco é a ação de de b em c devido a F
n é a força normal e p é a força peso nos três casos
Simplificando-se os pesos com as forças de reação normal
em cada caso temos que:
Como F = 4,2 N, temos:
4,2 = (1,0 + 2,0 + 3,0).a
a = 4,2/6
a = 0,7 m/s2
Encontrada a aceleração devemos encontrar o valor da
resultante em B.
A resultante em B é: FB = FA – FC
Subtraído as equações B e C temos:
FA = (mB + mC) . a
FA = (2,0 + 3,0). 0,7
FA = 5,0 . 0,7
FA = 3,5 N
Encontrando o valor de FC
FC = mC . a
FC = 3,0 . 0,7
FC = 2,1 N
Assim:
FB = 3,5 – 2,1
FB = 1,4 N
Concluímos então que a força resultante em B é igual a 1,4
N.
EXERCÍCIOS
Questão 01 - (UFT TO/2011)
Um piloto de teste pisa no acelerador de uma Ferrari,
para aumentar sua velocidade escalar, em uma pista
plana horizontal. Considere que o coeficiente de atrito
estático entre os pneus da Ferrari e a pista vale 0,5.
Nesta situação, qual valor melhor representa o módulo
da aceleração máxima que esta Ferrari pode atingir
nesta pista?
Desconsidere o atrito com o ar, e considere o módulo
da aceleração da gravidade igual a 9,8 m/s2
.
a) 3,8 m/s2
b) 4,9 m/s2
c) 16,2 m/s2
d) 11,1 m/s2
e) 9,8 m/s2
Questão 02 - (ACAFE SC/2011)
Um garoto obeso tem um peso de módulo 1200 N
(120kgf) e tenta mover uma caixa pesada conforme a
figura. O coeficiente de atrito estático entre os sapatos
do garoto e o piso é 0,5.
Assinale a alternativa correta que apresenta o módulo
da máxima força horizontal, em newtons, que o garoto
pode aplicar na caixa.
a) 1200
b) 600
c) 1800
d) 300
Questão 03 - (FEI SP)
Um caminhão, partindo do repouso, carrega uma bobina
de aço de massa m sobre a sua carroceria sem que haja
escorregamento. Quanto à força de atrito de
32
escorregamento na bobina, quando o caminhão estiver
acelerando podemos afirmar que:
a) não há força de atrito entre a bobina e a carroceria
b) tem direção normal à carroceria e sentido igual ao
de deslocamento do caminhão
c) tem direção paralela à carroceria e sentido igual ao
de deslocamento do caminhão
d) tem direção paralela à carroceria e sentido contrário
ao de deslocamento do caminhão
e) tem direção normal à carroceria e sentido contrário
ao de deslocamento do caminhão
Questão 04 - (UDESC/2011)
A figura abaixo mostra uma caixa de madeira que
desliza para baixo com velocidade constante sobre o
plano inclinado, sob a ação das seguintes forças: peso,
normal e de atrito.
Assinale a alternativa que representa corretamente o
esquema das forças exercidas sobre a caixa de madeira.
a)
b)
c)
d)
e)
Questão 05 - (UFOP MG)
Em um bloco em movimento sobre uma superfície
horizontal com velocidade constante atuam as quatro
forças mostradas na figura abaixo. Das forças indicadas,
assinar quais representam um par ação-reação.
Fa FV
N
P
a) (N, P)
b) (F, Fa)
c) (N, Fa)
d) (N, P) e (F, Fa)
e) nenhum par construído com as forças da figura
constitui um par ação-reação.
Questão 06 - (Unifacs BA/2011)
Considerando-se um pequeno bloco, preso na
extremidade de um fio ideal, que descreve um
movimento circular uniforme sobre uma superfície
horizontal, sem atrito, pode-se afirmar:
01. A força resultante que atua sobre o bloco é zero.
02. O período do movimento independe do
comprimento do fio.
03. A tração no fio produz variação na direção da
velocidade do bloco.
04. O bloco afastará radialmente, sob a ação exclusiva
da força centrífuga, se o fio sofrer rompimento.
05. O módulo da aceleração centrípeta do bloco é
diretamente proporcional à sua velocidade
angular.
Questão 07 - (UERJ)
Um balão, de peso igual a 0,1 N, está preso a um fio.
Além da força de empuxo E, o ar exerce uma forço
horizontal F que empurro e inclina o fio em relação à
vertical, conforme mostra afigura.
A tração no fio tem módulo igual o 0,2 N.
Calcule, em newtons, os módulos de:
a) E;
b) F.
Questão 08 - (FATEC SP)
Um corpo em movimento, num plano horizontal,
descreve uma trajetória curva. É correto afirmar que:
a) o movimento é necessariamente circular uniforme.
b) a força resultante é necessariamente centrípeta.
c) a trajetória é necessariamente parabólica.
d) a força centrípeta existe apenas quando a trajetória
é circular.
Questão 09 - (UFF RJ)
A figura mostra, em vista lateral, o exato instante em
que uma pipa paira no ar, em equilíbrio, sob a ação do
vento que sopra com uma velocidade horizontal
constante.
33
A força que o vento faz sobre a pipa nesta situação está
mais bem representada, na figura, pelo vetor:
1Fa.
2Fb.
3Fc.
4Fd.
5Fe.
Questão 10 - (FURG RS)
Um carro faz uma curva de 80 m de raio, com velocidade
de módulo constante igual a 72 km/h. Podemos afirmar
que sua aceleração é:
a) Zero m/s2
b) 0,5 m/s2
c) 0,9 m/s2
d) 4 m/s2
e) 5 m/s2
Questão 11 - (UEL PR/2008)
Com relação a um corpo em movimento circular
uniforme e sem atrito, considere as afirmativas
seguintes:
I. O vetor velocidade linear é constante.
II. A aceleração centrípeta é nula.
III. O módulo do vetor velocidade é constante.
IV. A força atua sempre perpendicularmente ao
deslocamento.
Assinale a alternativa que contém todas as afirmativas
corretas.
a) I e IV.
b) II e III.
c) III e IV.
d) I, II e III.
e) I, II e IV.
Questão 12 - (UEM PR)
O goleiro de um time de futebol bate um tiro de meta e
a bola percorre a trajetória esquematizada abaixo.
Despreze a resistência do ar e assinale o que for correto
(o ponto B corresponde ao instante em que a bola atinge
o solo).
01. No ponto A, a resultante das forças que atua sobre
a bola é para a direita e para cima.
02. No ponto B, a resultante das forças que atua sobre
a bola é nula.
04. No ponto A, a velocidade resultante da bola é para
a direita e para cima.
08. No ponto B, a velocidade resultante da bola é nula.
16. No ponto A, a energia total da bola é maior que no
ponto B.
Questão 13 - (UEM PR)
Um motociclista descreve uma circunferência num “globo
da morte" de raio 4 m, em movimento circular uniforme,
no sentido indicado pela seta curva, na figura abaixo.
A massa total (motorista + moto) é de 150 kg.
Considere g = 10 m/s e assinale o que for correto.
01.A velocidade do motociclista em B é tangente à
circunferência e dirigida para baixo ().
02.A aceleração do motociclista no ponto C é dirigida para
o centro da circunferência.
04.A força resultante sobre o motociclista no ponto A é
dirigida para fora da circunferência e perpendicular à
mesma ().
08.Se a velocidade do motociclista no ponto mais alto da
circunferência for 12 m/s, a força exercida sobre o
globo nesse ponto será 3900 N.
16.No ponto mais baixo da circunferência, a força
exercida sobre o globo é a mesma que a da parte mais
alta.
32.A velocidade mínima que o motociclista deve ter no
ponto mais alto da circunferência para que ele consiga
fazer a volta completa sem cair é 6,3 m/s.
Questão 14 - (UEM PR/2008)
Sobre aviões de acrobacia que executam loopings
(movimento circular cuja trajetória ocorre em um plano
vertical em relação ao horizonte) no céu, é correto
afirmar que, durante a execução do looping,
a) a aceleração centrífuga é duas vezes maior que a
centrípeta.
b) a energia potencial gravitacional se anula durante o
looping.
c) existe somente energia rotacional na execução da
manobra.
d) a aceleração centrípeta torna-se mais intensa que a
aceleração da gravidade.
e) a aceleração centrífuga reduz-se a um quarto da
centrípeta.
Questão 15 - (UNIOESTE PR)
Um carro de massa 1800 kg está em movimento circular
sobre uma rodovia circular plana de raio igual a 200 m.
O módulo de sua velocidade é constante. O coeficiente
de atrito estático entre os pneus do carro e a rodovia é
de 20,0e  . Considere a aceleração da gravidade g =
10 m/s2
e desconsidere o atrito com o ar. Para as
condições de movimento descritas, assinale a alternativa
correta.
a) O módulo da força de atrito que atua sobre o carro,
resultante da interação dos pneus do carro e a
rodovia, tem valor constante de 3600 N,
34
independentemente do módulo da velocidade de
movimento do carro.
b) A força de atrito sobre o carro, resultante da
interação entre os pneus do carro e a rodovia, é a
força centrípeta que permite o movimento do carro
em círculo.
c) Os vetores força peso e força centrípeta sobre o
carro possuem o mesmo sentido.
d) A força de atrito sobre o carro, resultante da
interação entre os pneus do carro e a rodovia, atua
em sentido contrário à força centrípeta sobre o
carro, que o mantém em movimento sobre a
rodovia.
e) A força resultante sobre o carro é nula.
Questão 16 - (UNIOESTE PR/2008)
Um carrinho de brinquedo é solto a partir do repouso
para percorrer uma pista sinuosa como mostra a figura
abaixo. Depois de descer a rampa de altura h, o carrinho
encontra uma lombada, cuja elevação acompanha a
forma de um semicírculo de raio r. Supondo que não
exista nenhum atrito agindo no brinquedo, qual o valor
máximo da razão h/r, para que o carrinho permaneça
em contato com a pista na parte superior da lombada?
a) 1/2.
b) 3/2.
c) 4/3.
d) 5/3.
e) 5/2.
Questão 17 - (UNIOESTE PR/2010)
Uma criança pendura-se na extremidade livre de uma
corda que tem a outra extremidade presa ao teto de uma
sala de ginástica. Ela, então, impulsiona-se e faz uma
trajetória circular cujo diâmetro é 0,8 m. Se a velocidade
tangencial da criança, cuja massa é 40 kg, for 2,0 m/s,
qual será o ângulo que a corda faz com uma linha
vertical perpendicular ao solo? Considere a criança como
uma partícula, a massa da corda desprezível e a
aceleração gravitacional igual 10 m/s2
.
a) 30º.
b) 35º.
c) 45º.
d) 20º.
e) 60º.
Questão 18 - (UEL PR)
A força de atrito é sempre dissipativa e resiste ao
movimento, mas há situações em que, embora esta
força seja resistente ao movimento, ela possibilita que o
movimento seja favorecido. Assinale a alternativa que
apresenta a situação física em que a força de atrito com
a superfície, ou de resistência de um fluido, favorece o
movimento.
a) Força de resistência do ar que atua em um
automóvel em movimento.
b) Força de resistência do ar que atua em um
paraquedas.
c) Força de atrito entre duas placas de vidro bem
polido molhadas.
d) Força de atrito que o chão aplica nos pneus de um
carro em movimento.
e) Força de atrito entre o pistão e o cilindro no motor
do automóvel.
Questão 19 - (UEL PR/2012)
Uma pessoa, de massa 80,0 kg, consegue aplicar uma
força de tração máxima de 800,0 N. Um corpo de massa
M necessita ser levantado como indicado na figura a
seguir. O coeficiente de atrito estático entre a sola do
sapato da pessoa e o chão de concreto é e = 1,0.
Faça um esboço de todas as forças que atuam em todo
o sistema e determine qual a maior massa M que pode
ser levantada pela pessoa sem que esta deslize, para
um ângulo  = 45º.
Questão 20 - (UEM PR/2006)
Uma caixa contendo ferramentas está em repouso sobre
uma superfície horizontal áspera. Uma pessoa está
tentando colocá-la em movimento, empurrandoa com
uma força paralela à superfície, mas não está
conseguindo. Qual a razão para isso?
a) A força que a mão da pessoa faz sobre a caixa é a
mesma que a caixa faz sobre a mão.
b) A força que o solo faz sobre a caixa devido ao atrito
cinético é muito maior do que a força que a pessoa
faz sobre a caixa.
c) A força de atrito estático que o solo exerce sobre a
caixa é de mesma intensidade da força que a mão
faz sobre a caixa.
d) A quantidade de momento que a caixa possui.
e) O fato de o torque que a mão imprime à caixa ser
menor do que a força de atrito estático que o solo
exerce sobre a caixa.
GABARITO:
1) Gab: B 2) Gab: B 3) Gab: C
4) Gab: E 5) Gab: E 6) Gab: 03
7) Gab: a) 0,27N b) 0,1N
8) Gab: C 9) Gab: B 10) Gab: E
11) Gab: C 12) Gab: 04 13) Gab: 43
14) Gab: D 15) Gab: B 16) Gab: B
17) Gab: C 18) Gab: D 19) Gab:
35
M = 40 2 Kg. 20) Gab: C
21) Gab: 11 22) Gab: 03 23) Gab: C
24) Gab: B 25) Gab: C 26) Gab: D
27) Gab: a)v = 4m/s b) aR = 0,8m/s2
c) NN = 552N e NJ = 756N
28) Gab: 04 29) Gab: E 30) Gab: 42
31) Gab: C 32) Gab: D 33) Gab: 29
34) Gab: 23 35) Gab: C 36) Gab: 09
37) Gab: C 38) Gab: C 39) Gab: D
40) Gab: 50
Curso de Física com Listas de Exercícios
Curso de Física com Listas de Exercícios
Curso de Física com Listas de Exercícios
Curso de Física com Listas de Exercícios
Curso de Física com Listas de Exercícios
Curso de Física com Listas de Exercícios
Curso de Física com Listas de Exercícios
Curso de Física com Listas de Exercícios
Curso de Física com Listas de Exercícios
Curso de Física com Listas de Exercícios
Curso de Física com Listas de Exercícios
Curso de Física com Listas de Exercícios
Curso de Física com Listas de Exercícios
Curso de Física com Listas de Exercícios
Curso de Física com Listas de Exercícios
Curso de Física com Listas de Exercícios
Curso de Física com Listas de Exercícios
Curso de Física com Listas de Exercícios
Curso de Física com Listas de Exercícios
Curso de Física com Listas de Exercícios
Curso de Física com Listas de Exercícios
Curso de Física com Listas de Exercícios
Curso de Física com Listas de Exercícios
Curso de Física com Listas de Exercícios
Curso de Física com Listas de Exercícios
Curso de Física com Listas de Exercícios
Curso de Física com Listas de Exercícios
Curso de Física com Listas de Exercícios
Curso de Física com Listas de Exercícios
Curso de Física com Listas de Exercícios
Curso de Física com Listas de Exercícios
Curso de Física com Listas de Exercícios
Curso de Física com Listas de Exercícios
Curso de Física com Listas de Exercícios
Curso de Física com Listas de Exercícios
Curso de Física com Listas de Exercícios
Curso de Física com Listas de Exercícios
Curso de Física com Listas de Exercícios
Curso de Física com Listas de Exercícios
Curso de Física com Listas de Exercícios
Curso de Física com Listas de Exercícios
Curso de Física com Listas de Exercícios
Curso de Física com Listas de Exercícios
Curso de Física com Listas de Exercícios
Curso de Física com Listas de Exercícios
Curso de Física com Listas de Exercícios
Curso de Física com Listas de Exercícios
Curso de Física com Listas de Exercícios
Curso de Física com Listas de Exercícios
Curso de Física com Listas de Exercícios
Curso de Física com Listas de Exercícios
Curso de Física com Listas de Exercícios
Curso de Física com Listas de Exercícios
Curso de Física com Listas de Exercícios
Curso de Física com Listas de Exercícios
Curso de Física com Listas de Exercícios
Curso de Física com Listas de Exercícios
Curso de Física com Listas de Exercícios
Curso de Física com Listas de Exercícios
Curso de Física com Listas de Exercícios
Curso de Física com Listas de Exercícios
Curso de Física com Listas de Exercícios
Curso de Física com Listas de Exercícios
Curso de Física com Listas de Exercícios
Curso de Física com Listas de Exercícios
Curso de Física com Listas de Exercícios
Curso de Física com Listas de Exercícios
Curso de Física com Listas de Exercícios
Curso de Física com Listas de Exercícios
Curso de Física com Listas de Exercícios
Curso de Física com Listas de Exercícios
Curso de Física com Listas de Exercícios
Curso de Física com Listas de Exercícios
Curso de Física com Listas de Exercícios
Curso de Física com Listas de Exercícios
Curso de Física com Listas de Exercícios
Curso de Física com Listas de Exercícios
Curso de Física com Listas de Exercícios
Curso de Física com Listas de Exercícios
Curso de Física com Listas de Exercícios
Curso de Física com Listas de Exercícios
Curso de Física com Listas de Exercícios
Curso de Física com Listas de Exercícios
Curso de Física com Listas de Exercícios
Curso de Física com Listas de Exercícios
Curso de Física com Listas de Exercícios
Curso de Física com Listas de Exercícios
Curso de Física com Listas de Exercícios
Curso de Física com Listas de Exercícios
Curso de Física com Listas de Exercícios
Curso de Física com Listas de Exercícios
Curso de Física com Listas de Exercícios
Curso de Física com Listas de Exercícios
Curso de Física com Listas de Exercícios
Curso de Física com Listas de Exercícios
Curso de Física com Listas de Exercícios
Curso de Física com Listas de Exercícios
Curso de Física com Listas de Exercícios
Curso de Física com Listas de Exercícios
Curso de Física com Listas de Exercícios
Curso de Física com Listas de Exercícios
Curso de Física com Listas de Exercícios
Curso de Física com Listas de Exercícios
Curso de Física com Listas de Exercícios
Curso de Física com Listas de Exercícios
Curso de Física com Listas de Exercícios
Curso de Física com Listas de Exercícios
Curso de Física com Listas de Exercícios
Curso de Física com Listas de Exercícios
Curso de Física com Listas de Exercícios
Curso de Física com Listas de Exercícios
Curso de Física com Listas de Exercícios
Curso de Física com Listas de Exercícios
Curso de Física com Listas de Exercícios
Curso de Física com Listas de Exercícios
Curso de Física com Listas de Exercícios
Curso de Física com Listas de Exercícios
Curso de Física com Listas de Exercícios
Curso de Física com Listas de Exercícios
Curso de Física com Listas de Exercícios
Curso de Física com Listas de Exercícios
Curso de Física com Listas de Exercícios
Curso de Física com Listas de Exercícios
Curso de Física com Listas de Exercícios
Curso de Física com Listas de Exercícios
Curso de Física com Listas de Exercícios
Curso de Física com Listas de Exercícios
Curso de Física com Listas de Exercícios
Curso de Física com Listas de Exercícios
Curso de Física com Listas de Exercícios
Curso de Física com Listas de Exercícios
Curso de Física com Listas de Exercícios
Curso de Física com Listas de Exercícios
Curso de Física com Listas de Exercícios
Curso de Física com Listas de Exercícios
Curso de Física com Listas de Exercícios
Curso de Física com Listas de Exercícios
Curso de Física com Listas de Exercícios
Curso de Física com Listas de Exercícios
Curso de Física com Listas de Exercícios
Curso de Física com Listas de Exercícios
Curso de Física com Listas de Exercícios
Curso de Física com Listas de Exercícios
Curso de Física com Listas de Exercícios
Curso de Física com Listas de Exercícios

Más contenido relacionado

Destacado

Lista 18 eletrodinamica 1
Lista 18 eletrodinamica 1Lista 18 eletrodinamica 1
Lista 18 eletrodinamica 1rodrigoateneu
 
Basica vetores e força
Basica vetores e forçaBasica vetores e força
Basica vetores e forçarodrigoateneu
 
Lista 25 mhs e ondas
Lista 25 mhs e ondasLista 25 mhs e ondas
Lista 25 mhs e ondasrodrigoateneu
 
Lista 13 calorimetria
Lista 13 calorimetriaLista 13 calorimetria
Lista 13 calorimetriarodrigoateneu
 
Lista 27 magnetismo e fontes de campo
Lista 27 magnetismo e fontes de campoLista 27 magnetismo e fontes de campo
Lista 27 magnetismo e fontes de camporodrigoateneu
 
Lista 28 força magnética e induçao
Lista 28 força magnética e induçaoLista 28 força magnética e induçao
Lista 28 força magnética e induçaorodrigoateneu
 
Lista 26 interferencia e acustica
Lista 26 interferencia e acusticaLista 26 interferencia e acustica
Lista 26 interferencia e acusticarodrigoateneu
 
005 dinamica leis_de_newton_exercicios
005 dinamica leis_de_newton_exercicios005 dinamica leis_de_newton_exercicios
005 dinamica leis_de_newton_exerciciosandryellebatista
 
Eletrostatica e lei de coulomb
Eletrostatica e lei de coulombEletrostatica e lei de coulomb
Eletrostatica e lei de coulombrodrigoateneu
 
Lista 22 espelho esférico
Lista 22 espelho esféricoLista 22 espelho esférico
Lista 22 espelho esféricorodrigoateneu
 
Lista 8 trabalho e energia
Lista 8 trabalho e energiaLista 8 trabalho e energia
Lista 8 trabalho e energiarodrigoateneu
 

Destacado (20)

Lista 4 mcu
Lista 4 mcuLista 4 mcu
Lista 4 mcu
 
Basica mu
Basica muBasica mu
Basica mu
 
Basica muv
Basica muvBasica muv
Basica muv
 
Acelerado
AceleradoAcelerado
Acelerado
 
Lista 18 eletrodinamica 1
Lista 18 eletrodinamica 1Lista 18 eletrodinamica 1
Lista 18 eletrodinamica 1
 
Basica vetores e força
Basica vetores e forçaBasica vetores e força
Basica vetores e força
 
Lista 25 mhs e ondas
Lista 25 mhs e ondasLista 25 mhs e ondas
Lista 25 mhs e ondas
 
Lista 13 calorimetria
Lista 13 calorimetriaLista 13 calorimetria
Lista 13 calorimetria
 
Lista 27 magnetismo e fontes de campo
Lista 27 magnetismo e fontes de campoLista 27 magnetismo e fontes de campo
Lista 27 magnetismo e fontes de campo
 
Lista 28 força magnética e induçao
Lista 28 força magnética e induçaoLista 28 força magnética e induçao
Lista 28 força magnética e induçao
 
Lista 26 interferencia e acustica
Lista 26 interferencia e acusticaLista 26 interferencia e acustica
Lista 26 interferencia e acustica
 
005 dinamica leis_de_newton_exercicios
005 dinamica leis_de_newton_exercicios005 dinamica leis_de_newton_exercicios
005 dinamica leis_de_newton_exercicios
 
Eletrostatica e lei de coulomb
Eletrostatica e lei de coulombEletrostatica e lei de coulomb
Eletrostatica e lei de coulomb
 
Lista 24 lentes
Lista 24 lentesLista 24 lentes
Lista 24 lentes
 
Lista 21 optica 1
Lista 21 optica 1Lista 21 optica 1
Lista 21 optica 1
 
Lista 23 refração
Lista 23 refraçãoLista 23 refração
Lista 23 refração
 
Lista 22 espelho esférico
Lista 22 espelho esféricoLista 22 espelho esférico
Lista 22 espelho esférico
 
Lista 2 muv
Lista 2 muvLista 2 muv
Lista 2 muv
 
Lista 8 trabalho e energia
Lista 8 trabalho e energiaLista 8 trabalho e energia
Lista 8 trabalho e energia
 
Lista 5 vetores
Lista 5 vetoresLista 5 vetores
Lista 5 vetores
 

Similar a Curso de Física com Listas de Exercícios

Resumo de MCU (Movimento Circular e Uniforme)
Resumo de MCU (Movimento Circular e Uniforme)Resumo de MCU (Movimento Circular e Uniforme)
Resumo de MCU (Movimento Circular e Uniforme)tioivys
 
227516957 cad-c3-curso-a-prof-exercicios-fisica
227516957 cad-c3-curso-a-prof-exercicios-fisica227516957 cad-c3-curso-a-prof-exercicios-fisica
227516957 cad-c3-curso-a-prof-exercicios-fisicaheltonjos
 
Problemas de resistencia_dos_materiais_s usp
Problemas de resistencia_dos_materiais_s uspProblemas de resistencia_dos_materiais_s usp
Problemas de resistencia_dos_materiais_s uspadalberto miran
 
Problemas de resistencia dos materiais
Problemas de resistencia dos materiaisProblemas de resistencia dos materiais
Problemas de resistencia dos materiaisfercrotti
 
Problemas de resistencia_dos_materiais_s
Problemas de resistencia_dos_materiais_sProblemas de resistencia_dos_materiais_s
Problemas de resistencia_dos_materiais_sVictor Pelanz
 
Banco de questões fisica bonjorno ftd
Banco de questões fisica   bonjorno ftdBanco de questões fisica   bonjorno ftd
Banco de questões fisica bonjorno ftdNHOPTU
 
Bonjorno questc3b5es
Bonjorno questc3b5esBonjorno questc3b5es
Bonjorno questc3b5esPaulo Souto
 
Fisica bonjorno
Fisica   bonjornoFisica   bonjorno
Fisica bonjornoJose Costa
 
Simulação numérica da injeção
Simulação numérica da injeçãoSimulação numérica da injeção
Simulação numérica da injeçãopaulo2915
 
Atividade de movimento circular
Atividade de movimento circularAtividade de movimento circular
Atividade de movimento circulartiowans
 
Projeto de um Veio de Transmissão de “Caixa de Velocidades de Automóvel”
Projeto de um Veio de Transmissão de “Caixa de Velocidades de Automóvel”Projeto de um Veio de Transmissão de “Caixa de Velocidades de Automóvel”
Projeto de um Veio de Transmissão de “Caixa de Velocidades de Automóvel”Ivan Soares
 
Evaluation of the direction of arrival estimation methods for the wireless co...
Evaluation of the direction of arrival estimation methods for the wireless co...Evaluation of the direction of arrival estimation methods for the wireless co...
Evaluation of the direction of arrival estimation methods for the wireless co...Alberto Boaventura
 
Simulado 2 _abril1398617914_exercicio
Simulado 2 _abril1398617914_exercicioSimulado 2 _abril1398617914_exercicio
Simulado 2 _abril1398617914_exercicioROBSONVINAS
 
28358 lista de exercícios de mru e mruv
28358 lista de exercícios de mru e mruv28358 lista de exercícios de mru e mruv
28358 lista de exercícios de mru e mruvafpinto
 
Lista Cinemática (C5) MCU
Lista Cinemática (C5) MCULista Cinemática (C5) MCU
Lista Cinemática (C5) MCUGilberto Rocha
 

Similar a Curso de Física com Listas de Exercícios (20)

Resumo de MCU (Movimento Circular e Uniforme)
Resumo de MCU (Movimento Circular e Uniforme)Resumo de MCU (Movimento Circular e Uniforme)
Resumo de MCU (Movimento Circular e Uniforme)
 
7forças
7forças7forças
7forças
 
227516957 cad-c3-curso-a-prof-exercicios-fisica
227516957 cad-c3-curso-a-prof-exercicios-fisica227516957 cad-c3-curso-a-prof-exercicios-fisica
227516957 cad-c3-curso-a-prof-exercicios-fisica
 
Desenho tecnico unimar
Desenho tecnico unimarDesenho tecnico unimar
Desenho tecnico unimar
 
Parafuso tirefond.pdf
Parafuso tirefond.pdfParafuso tirefond.pdf
Parafuso tirefond.pdf
 
Problemas de resistencia_dos_materiais_s usp
Problemas de resistencia_dos_materiais_s uspProblemas de resistencia_dos_materiais_s usp
Problemas de resistencia_dos_materiais_s usp
 
Problemas de resistencia dos materiais
Problemas de resistencia dos materiaisProblemas de resistencia dos materiais
Problemas de resistencia dos materiais
 
Problemas de resistencia_dos_materiais_s
Problemas de resistencia_dos_materiais_sProblemas de resistencia_dos_materiais_s
Problemas de resistencia_dos_materiais_s
 
Banco de questões fisica bonjorno ftd
Banco de questões fisica   bonjorno ftdBanco de questões fisica   bonjorno ftd
Banco de questões fisica bonjorno ftd
 
Bonjorno questc3b5es
Bonjorno questc3b5esBonjorno questc3b5es
Bonjorno questc3b5es
 
Fisica bonjorno
Fisica   bonjornoFisica   bonjorno
Fisica bonjorno
 
Fisica bonjorno
Fisica   bonjornoFisica   bonjorno
Fisica bonjorno
 
Fisica bonjorno
Fisica   bonjornoFisica   bonjorno
Fisica bonjorno
 
Simulação numérica da injeção
Simulação numérica da injeçãoSimulação numérica da injeção
Simulação numérica da injeção
 
Atividade de movimento circular
Atividade de movimento circularAtividade de movimento circular
Atividade de movimento circular
 
Projeto de um Veio de Transmissão de “Caixa de Velocidades de Automóvel”
Projeto de um Veio de Transmissão de “Caixa de Velocidades de Automóvel”Projeto de um Veio de Transmissão de “Caixa de Velocidades de Automóvel”
Projeto de um Veio de Transmissão de “Caixa de Velocidades de Automóvel”
 
Evaluation of the direction of arrival estimation methods for the wireless co...
Evaluation of the direction of arrival estimation methods for the wireless co...Evaluation of the direction of arrival estimation methods for the wireless co...
Evaluation of the direction of arrival estimation methods for the wireless co...
 
Simulado 2 _abril1398617914_exercicio
Simulado 2 _abril1398617914_exercicioSimulado 2 _abril1398617914_exercicio
Simulado 2 _abril1398617914_exercicio
 
28358 lista de exercícios de mru e mruv
28358 lista de exercícios de mru e mruv28358 lista de exercícios de mru e mruv
28358 lista de exercícios de mru e mruv
 
Lista Cinemática (C5) MCU
Lista Cinemática (C5) MCULista Cinemática (C5) MCU
Lista Cinemática (C5) MCU
 

Más de rodrigoateneu

Lista 20 eletrodinamica 3
Lista 20 eletrodinamica 3Lista 20 eletrodinamica 3
Lista 20 eletrodinamica 3rodrigoateneu
 
Lista 19 eletrodinamica 2
Lista 19 eletrodinamica 2Lista 19 eletrodinamica 2
Lista 19 eletrodinamica 2rodrigoateneu
 
Basica campo e potencial
Basica campo e potencialBasica campo e potencial
Basica campo e potencialrodrigoateneu
 
Lista 17 eletrostatica 2
Lista 17 eletrostatica 2Lista 17 eletrostatica 2
Lista 17 eletrostatica 2rodrigoateneu
 
Lista 16 eletrostatica1
Lista 16 eletrostatica1Lista 16 eletrostatica1
Lista 16 eletrostatica1rodrigoateneu
 
Lista 15 termodin+ómica
Lista 15 termodin+ómicaLista 15 termodin+ómica
Lista 15 termodin+ómicarodrigoateneu
 
Lista 12 termometria e dilata+º+úo
Lista 12 termometria e dilata+º+úoLista 12 termometria e dilata+º+úo
Lista 12 termometria e dilata+º+úorodrigoateneu
 
Lista 11 gravita+º+úo
Lista 11 gravita+º+úoLista 11 gravita+º+úo
Lista 11 gravita+º+úorodrigoateneu
 
Lista 10 est+ítica hidrost+ítica
Lista 10 est+ítica hidrost+íticaLista 10 est+ítica hidrost+ítica
Lista 10 est+ítica hidrost+íticarodrigoateneu
 
Lista 9 dinamica impulsiva
Lista 9 dinamica impulsivaLista 9 dinamica impulsiva
Lista 9 dinamica impulsivarodrigoateneu
 
Lista 6 leis de newton
Lista 6 leis de newtonLista 6 leis de newton
Lista 6 leis de newtonrodrigoateneu
 

Más de rodrigoateneu (15)

Lista 20 eletrodinamica 3
Lista 20 eletrodinamica 3Lista 20 eletrodinamica 3
Lista 20 eletrodinamica 3
 
Lista 19 eletrodinamica 2
Lista 19 eletrodinamica 2Lista 19 eletrodinamica 2
Lista 19 eletrodinamica 2
 
Basica refração
Basica refraçãoBasica refração
Basica refração
 
Básica circuitos
Básica circuitosBásica circuitos
Básica circuitos
 
Basica resistores
Basica resistoresBasica resistores
Basica resistores
 
Basica campo e potencial
Basica campo e potencialBasica campo e potencial
Basica campo e potencial
 
Lista 17 eletrostatica 2
Lista 17 eletrostatica 2Lista 17 eletrostatica 2
Lista 17 eletrostatica 2
 
Lista 16 eletrostatica1
Lista 16 eletrostatica1Lista 16 eletrostatica1
Lista 16 eletrostatica1
 
Lista 15 termodin+ómica
Lista 15 termodin+ómicaLista 15 termodin+ómica
Lista 15 termodin+ómica
 
Lista 14 gases
Lista 14 gasesLista 14 gases
Lista 14 gases
 
Lista 12 termometria e dilata+º+úo
Lista 12 termometria e dilata+º+úoLista 12 termometria e dilata+º+úo
Lista 12 termometria e dilata+º+úo
 
Lista 11 gravita+º+úo
Lista 11 gravita+º+úoLista 11 gravita+º+úo
Lista 11 gravita+º+úo
 
Lista 10 est+ítica hidrost+ítica
Lista 10 est+ítica hidrost+íticaLista 10 est+ítica hidrost+ítica
Lista 10 est+ítica hidrost+ítica
 
Lista 9 dinamica impulsiva
Lista 9 dinamica impulsivaLista 9 dinamica impulsiva
Lista 9 dinamica impulsiva
 
Lista 6 leis de newton
Lista 6 leis de newtonLista 6 leis de newton
Lista 6 leis de newton
 

Último

Apresentação ISBET Jovem Aprendiz e Estágio 2023.pdf
Apresentação ISBET Jovem Aprendiz e Estágio 2023.pdfApresentação ISBET Jovem Aprendiz e Estágio 2023.pdf
Apresentação ISBET Jovem Aprendiz e Estágio 2023.pdfcomercial400681
 
Construção (C)erta - Nós Propomos! Sertã
Construção (C)erta - Nós Propomos! SertãConstrução (C)erta - Nós Propomos! Sertã
Construção (C)erta - Nós Propomos! SertãIlda Bicacro
 
Historia da Arte europeia e não só. .pdf
Historia da Arte europeia e não só. .pdfHistoria da Arte europeia e não só. .pdf
Historia da Arte europeia e não só. .pdfEmanuel Pio
 
Projeto_de_Extensão_Agronomia_adquira_ja_(91)_98764-0830.pdf
Projeto_de_Extensão_Agronomia_adquira_ja_(91)_98764-0830.pdfProjeto_de_Extensão_Agronomia_adquira_ja_(91)_98764-0830.pdf
Projeto_de_Extensão_Agronomia_adquira_ja_(91)_98764-0830.pdfHELENO FAVACHO
 
PROJETO DE EXTENSÃO I - Radiologia Tecnologia
PROJETO DE EXTENSÃO I - Radiologia TecnologiaPROJETO DE EXTENSÃO I - Radiologia Tecnologia
PROJETO DE EXTENSÃO I - Radiologia TecnologiaHELENO FAVACHO
 
Teoria heterotrófica e autotrófica dos primeiros seres vivos..pptx
Teoria heterotrófica e autotrófica dos primeiros seres vivos..pptxTeoria heterotrófica e autotrófica dos primeiros seres vivos..pptx
Teoria heterotrófica e autotrófica dos primeiros seres vivos..pptxTailsonSantos1
 
PROJETO DE EXTENSÃO I - TERAPIAS INTEGRATIVAS E COMPLEMENTARES.pdf
PROJETO DE EXTENSÃO I - TERAPIAS INTEGRATIVAS E COMPLEMENTARES.pdfPROJETO DE EXTENSÃO I - TERAPIAS INTEGRATIVAS E COMPLEMENTARES.pdf
PROJETO DE EXTENSÃO I - TERAPIAS INTEGRATIVAS E COMPLEMENTARES.pdfHELENO FAVACHO
 
A QUATRO MÃOS - MARILDA CASTANHA . pdf
A QUATRO MÃOS  -  MARILDA CASTANHA . pdfA QUATRO MÃOS  -  MARILDA CASTANHA . pdf
A QUATRO MÃOS - MARILDA CASTANHA . pdfAna Lemos
 
PROJETO DE EXTENSÃO I - TECNOLOGIA DA INFORMAÇÃO Relatório Final de Atividade...
PROJETO DE EXTENSÃO I - TECNOLOGIA DA INFORMAÇÃO Relatório Final de Atividade...PROJETO DE EXTENSÃO I - TECNOLOGIA DA INFORMAÇÃO Relatório Final de Atividade...
PROJETO DE EXTENSÃO I - TECNOLOGIA DA INFORMAÇÃO Relatório Final de Atividade...HELENO FAVACHO
 
PROVA - ESTUDO CONTEMPORÂNEO E TRANSVERSAL: LEITURA DE IMAGENS, GRÁFICOS E MA...
PROVA - ESTUDO CONTEMPORÂNEO E TRANSVERSAL: LEITURA DE IMAGENS, GRÁFICOS E MA...PROVA - ESTUDO CONTEMPORÂNEO E TRANSVERSAL: LEITURA DE IMAGENS, GRÁFICOS E MA...
PROVA - ESTUDO CONTEMPORÂNEO E TRANSVERSAL: LEITURA DE IMAGENS, GRÁFICOS E MA...azulassessoria9
 
PRÉDIOS HISTÓRICOS DE ASSARÉ Prof. Francisco Leite.pdf
PRÉDIOS HISTÓRICOS DE ASSARÉ Prof. Francisco Leite.pdfPRÉDIOS HISTÓRICOS DE ASSARÉ Prof. Francisco Leite.pdf
PRÉDIOS HISTÓRICOS DE ASSARÉ Prof. Francisco Leite.pdfprofesfrancleite
 
2° ANO - ENSINO FUNDAMENTAL ENSINO RELIGIOSO
2° ANO - ENSINO FUNDAMENTAL ENSINO RELIGIOSO2° ANO - ENSINO FUNDAMENTAL ENSINO RELIGIOSO
2° ANO - ENSINO FUNDAMENTAL ENSINO RELIGIOSOLeloIurk1
 
Considere a seguinte situação fictícia: Durante uma reunião de equipe em uma...
Considere a seguinte situação fictícia:  Durante uma reunião de equipe em uma...Considere a seguinte situação fictícia:  Durante uma reunião de equipe em uma...
Considere a seguinte situação fictícia: Durante uma reunião de equipe em uma...azulassessoria9
 
PROVA - ESTUDO CONTEMPORÂNEO E TRANSVERSAL: COMUNICAÇÃO ASSERTIVA E INTERPESS...
PROVA - ESTUDO CONTEMPORÂNEO E TRANSVERSAL: COMUNICAÇÃO ASSERTIVA E INTERPESS...PROVA - ESTUDO CONTEMPORÂNEO E TRANSVERSAL: COMUNICAÇÃO ASSERTIVA E INTERPESS...
PROVA - ESTUDO CONTEMPORÂNEO E TRANSVERSAL: COMUNICAÇÃO ASSERTIVA E INTERPESS...azulassessoria9
 
Currículo - Ícaro Kleisson - Tutor acadêmico.pdf
Currículo - Ícaro Kleisson - Tutor acadêmico.pdfCurrículo - Ícaro Kleisson - Tutor acadêmico.pdf
Currículo - Ícaro Kleisson - Tutor acadêmico.pdfTutor de matemática Ícaro
 
Rota das Ribeiras Camp, Projeto Nós Propomos!
Rota das Ribeiras Camp, Projeto Nós Propomos!Rota das Ribeiras Camp, Projeto Nós Propomos!
Rota das Ribeiras Camp, Projeto Nós Propomos!Ilda Bicacro
 
DeClara n.º 75 Abril 2024 - O Jornal digital do Agrupamento de Escolas Clara ...
DeClara n.º 75 Abril 2024 - O Jornal digital do Agrupamento de Escolas Clara ...DeClara n.º 75 Abril 2024 - O Jornal digital do Agrupamento de Escolas Clara ...
DeClara n.º 75 Abril 2024 - O Jornal digital do Agrupamento de Escolas Clara ...IsabelPereira2010
 
About Vila Galé- Cadeia Empresarial de Hotéis
About Vila Galé- Cadeia Empresarial de HotéisAbout Vila Galé- Cadeia Empresarial de Hotéis
About Vila Galé- Cadeia Empresarial de Hotéisines09cachapa
 
PROJETO DE EXTENSÃO I - AGRONOMIA.pdf AGRONOMIAAGRONOMIA
PROJETO DE EXTENSÃO I - AGRONOMIA.pdf AGRONOMIAAGRONOMIAPROJETO DE EXTENSÃO I - AGRONOMIA.pdf AGRONOMIAAGRONOMIA
PROJETO DE EXTENSÃO I - AGRONOMIA.pdf AGRONOMIAAGRONOMIAHELENO FAVACHO
 

Último (20)

Apresentação ISBET Jovem Aprendiz e Estágio 2023.pdf
Apresentação ISBET Jovem Aprendiz e Estágio 2023.pdfApresentação ISBET Jovem Aprendiz e Estágio 2023.pdf
Apresentação ISBET Jovem Aprendiz e Estágio 2023.pdf
 
Construção (C)erta - Nós Propomos! Sertã
Construção (C)erta - Nós Propomos! SertãConstrução (C)erta - Nós Propomos! Sertã
Construção (C)erta - Nós Propomos! Sertã
 
Historia da Arte europeia e não só. .pdf
Historia da Arte europeia e não só. .pdfHistoria da Arte europeia e não só. .pdf
Historia da Arte europeia e não só. .pdf
 
Projeto_de_Extensão_Agronomia_adquira_ja_(91)_98764-0830.pdf
Projeto_de_Extensão_Agronomia_adquira_ja_(91)_98764-0830.pdfProjeto_de_Extensão_Agronomia_adquira_ja_(91)_98764-0830.pdf
Projeto_de_Extensão_Agronomia_adquira_ja_(91)_98764-0830.pdf
 
PROJETO DE EXTENSÃO I - Radiologia Tecnologia
PROJETO DE EXTENSÃO I - Radiologia TecnologiaPROJETO DE EXTENSÃO I - Radiologia Tecnologia
PROJETO DE EXTENSÃO I - Radiologia Tecnologia
 
Teoria heterotrófica e autotrófica dos primeiros seres vivos..pptx
Teoria heterotrófica e autotrófica dos primeiros seres vivos..pptxTeoria heterotrófica e autotrófica dos primeiros seres vivos..pptx
Teoria heterotrófica e autotrófica dos primeiros seres vivos..pptx
 
PROJETO DE EXTENSÃO I - TERAPIAS INTEGRATIVAS E COMPLEMENTARES.pdf
PROJETO DE EXTENSÃO I - TERAPIAS INTEGRATIVAS E COMPLEMENTARES.pdfPROJETO DE EXTENSÃO I - TERAPIAS INTEGRATIVAS E COMPLEMENTARES.pdf
PROJETO DE EXTENSÃO I - TERAPIAS INTEGRATIVAS E COMPLEMENTARES.pdf
 
A QUATRO MÃOS - MARILDA CASTANHA . pdf
A QUATRO MÃOS  -  MARILDA CASTANHA . pdfA QUATRO MÃOS  -  MARILDA CASTANHA . pdf
A QUATRO MÃOS - MARILDA CASTANHA . pdf
 
PROJETO DE EXTENSÃO I - TECNOLOGIA DA INFORMAÇÃO Relatório Final de Atividade...
PROJETO DE EXTENSÃO I - TECNOLOGIA DA INFORMAÇÃO Relatório Final de Atividade...PROJETO DE EXTENSÃO I - TECNOLOGIA DA INFORMAÇÃO Relatório Final de Atividade...
PROJETO DE EXTENSÃO I - TECNOLOGIA DA INFORMAÇÃO Relatório Final de Atividade...
 
PROVA - ESTUDO CONTEMPORÂNEO E TRANSVERSAL: LEITURA DE IMAGENS, GRÁFICOS E MA...
PROVA - ESTUDO CONTEMPORÂNEO E TRANSVERSAL: LEITURA DE IMAGENS, GRÁFICOS E MA...PROVA - ESTUDO CONTEMPORÂNEO E TRANSVERSAL: LEITURA DE IMAGENS, GRÁFICOS E MA...
PROVA - ESTUDO CONTEMPORÂNEO E TRANSVERSAL: LEITURA DE IMAGENS, GRÁFICOS E MA...
 
PRÉDIOS HISTÓRICOS DE ASSARÉ Prof. Francisco Leite.pdf
PRÉDIOS HISTÓRICOS DE ASSARÉ Prof. Francisco Leite.pdfPRÉDIOS HISTÓRICOS DE ASSARÉ Prof. Francisco Leite.pdf
PRÉDIOS HISTÓRICOS DE ASSARÉ Prof. Francisco Leite.pdf
 
2° ANO - ENSINO FUNDAMENTAL ENSINO RELIGIOSO
2° ANO - ENSINO FUNDAMENTAL ENSINO RELIGIOSO2° ANO - ENSINO FUNDAMENTAL ENSINO RELIGIOSO
2° ANO - ENSINO FUNDAMENTAL ENSINO RELIGIOSO
 
Aula sobre o Imperialismo Europeu no século XIX
Aula sobre o Imperialismo Europeu no século XIXAula sobre o Imperialismo Europeu no século XIX
Aula sobre o Imperialismo Europeu no século XIX
 
Considere a seguinte situação fictícia: Durante uma reunião de equipe em uma...
Considere a seguinte situação fictícia:  Durante uma reunião de equipe em uma...Considere a seguinte situação fictícia:  Durante uma reunião de equipe em uma...
Considere a seguinte situação fictícia: Durante uma reunião de equipe em uma...
 
PROVA - ESTUDO CONTEMPORÂNEO E TRANSVERSAL: COMUNICAÇÃO ASSERTIVA E INTERPESS...
PROVA - ESTUDO CONTEMPORÂNEO E TRANSVERSAL: COMUNICAÇÃO ASSERTIVA E INTERPESS...PROVA - ESTUDO CONTEMPORÂNEO E TRANSVERSAL: COMUNICAÇÃO ASSERTIVA E INTERPESS...
PROVA - ESTUDO CONTEMPORÂNEO E TRANSVERSAL: COMUNICAÇÃO ASSERTIVA E INTERPESS...
 
Currículo - Ícaro Kleisson - Tutor acadêmico.pdf
Currículo - Ícaro Kleisson - Tutor acadêmico.pdfCurrículo - Ícaro Kleisson - Tutor acadêmico.pdf
Currículo - Ícaro Kleisson - Tutor acadêmico.pdf
 
Rota das Ribeiras Camp, Projeto Nós Propomos!
Rota das Ribeiras Camp, Projeto Nós Propomos!Rota das Ribeiras Camp, Projeto Nós Propomos!
Rota das Ribeiras Camp, Projeto Nós Propomos!
 
DeClara n.º 75 Abril 2024 - O Jornal digital do Agrupamento de Escolas Clara ...
DeClara n.º 75 Abril 2024 - O Jornal digital do Agrupamento de Escolas Clara ...DeClara n.º 75 Abril 2024 - O Jornal digital do Agrupamento de Escolas Clara ...
DeClara n.º 75 Abril 2024 - O Jornal digital do Agrupamento de Escolas Clara ...
 
About Vila Galé- Cadeia Empresarial de Hotéis
About Vila Galé- Cadeia Empresarial de HotéisAbout Vila Galé- Cadeia Empresarial de Hotéis
About Vila Galé- Cadeia Empresarial de Hotéis
 
PROJETO DE EXTENSÃO I - AGRONOMIA.pdf AGRONOMIAAGRONOMIA
PROJETO DE EXTENSÃO I - AGRONOMIA.pdf AGRONOMIAAGRONOMIAPROJETO DE EXTENSÃO I - AGRONOMIA.pdf AGRONOMIAAGRONOMIA
PROJETO DE EXTENSÃO I - AGRONOMIA.pdf AGRONOMIAAGRONOMIA
 

Curso de Física com Listas de Exercícios

  • 1. 1 PROF.MENDONÇA PARA APOSTILA COMPLETA VENHA FAZER UM DE MEUS CURSOS: Segunda feira 18:30 até 21:45 Curso e colégio integral (Específica semestral) Fone: (44) 3028-5280 Maringá Terça feira 17:30 até 19:30 Curso Prime (Física básica semestral) Fone: (43) 3025-1020 Londrina Quarta feira 14.00 até 17:00 Curso Sigma ( Específica modular) Fone: (43) 3321-2652 Londrina Quinta feira 19:15 até 22:30 Curso Saber (Específica aprofundada) Fone: (43) 3326-1212 Londrina Sexta feira 16:00 até 18:00 Curso e colégio integral (Física básica semestral) Fone: (44) 3028-5280 Maringá 2017
  • 2. 2 Sumário LISTA 1 - MOVIMENTO UNIFORME...................................................................................................................................3 LISTA 2 - MOVIMENTO UNIFORMEMENTE VARIADO.......................................................................................................7 LISTA 3 - LANÇAMENTO OBLÍQUO..................................................................................................................................12 LISTA 4 - MOVIMENTO CIRCULAR...................................................................................................................................17 LISTA 5 - VETORES E CINEMÁTICA VETORIAL..................................................................................................................21 LISTA 6 - LEIS DE NEWTON..............................................................................................................................................26 LISTA 7 - APLICAÇÕES DAS LEIS DE NEWTON .................................................................................................................31 LISTA 8 - TRABALHO E ENERGIA......................................................................................................................................36 LISTA 9 - DINÂMICA IMPULSIVA E COLISÕES ..................................................................................................................41 LISTA 10 - ESTÁTICA E HIDROSTÁTICA ............................................................................................................................45 LISTA 11 – LEIS DE KEPLER E GRAVITAÇÃO.....................................................................................................................51 LISTA 12 - TERMOMETRIA E DILATAÇÃO ........................................................................................................................57 LISTA 13 - CALORIMETRIA...............................................................................................................................................61 LISTA 14 - GASES E TERMODINÂMICA ............................................................................................................................66 LISTA 15 - TERMODINÂMICA ..........................................................................................................................................72 LISTA 16 - ELETROSTÁTICA 1 – LEI DE COULOMB E CAMPO ELÉTRICO...........................................................................77 LISTA 17 – ELETROSTÁTICA 2 – POTENCIAL, ENERGIA E TRABALHO ..............................................................................85 LISTA 18 – ELETRODINÂMICA 1 – LEIS DE ÔHM..............................................................................................................90 LISTA 19 - ELETRODINÂMICA 2 - GERADORES E RECEPTORES........................................................................................97 LISTA 20 – ELETRODINÂMICA 3 – LEIS DE KIRCHHOFF .................................................................................................105 LISTA 21 – ÓPTICA 1 – FUNDAMENTOS E REFLEXÃO DA LUZ .......................................................................................111 LISTA 22 – REFLEXÃO DA LUZ ESPELHOS ESFÉRICOS ....................................................................................................118 LISTA 23 – REFRAÇÃO DA LUZ LEI DE SNELL-DESCARTES..............................................................................................124 LISTA 24 – ESTUDO DAS LENTES ESFÉRICAS .................................................................................................................130 LISTA 25 – MOVIMENTO HARMÔNICO SIMPLES E ONDULATÓRIA ..............................................................................136 LISTA 26 – INTERFERÊNCIA E ACÚSTICA........................................................................................................................145 LISTA 27 – MAGNETISMO E FONTES DE CAMPO MAGNÉTICO.....................................................................................150 LISTA 28 – FORÇA MAGNÉTICA E INDUÇÃO MAGNÉTICA ............................................................................................157 LISTA 29 – FÍSICA MODERNA ........................................................................................................................................164 PROVAS DE VESTIBULARES .........................................................................................................................176 UEL 1ª FASE 2017..............................................................................................................................................176 UEL 2ª FASE 2017..............................................................................................................................................177 UEM 1º DIA 2016 ...............................................................................................................................................178
  • 3. 3 LISTA 1 - MOVIMENTO UNIFORME RESUMO DE CONTEÚDO Características do Movimento Uniforme O movimento uniforme é o movimento que possui módulo da velocidade constante, ou seja, ela não varia com o passar do tempo. Entretanto, essa velocidade, apesar de ser constante, é diferente de zero, ou seja, ela pode assumir qualquer outro valor que não seja o zero. Sendo a aceleração definida da seguinte forma: E sabendo que no movimento uniforme a variação do módulo da velocidade é igual a zero, pois a velocidade final é igual à velocidade inicial, concluímos que a aceleração escalar é constante e igual a zero. A função Horária do Movimento Uniforme No movimento uniforme temos que a velocidade escalar é constante e coincide com a velocidade escalar média em qualquer instante ou intervalo de tempo. Matematicamente, a velocidade escalar média pode ser expressa da seguinte forma: Onde: • ΔS é a variação de posição do móvel, ΔS = S – So; • Δt é a variação do tempo, Δt = t – to. Substituído ΔS e Δt na equação da velocidade descrita acima, temos: Fazendo tempo inicial igual a zero, to= 0, temos a função horária do movimento uniforme. S = So + Vt EXERCÍCIOS Questão 01 - (UEG GO/2012) A órbita do planeta Terra, em torno do Sol, possui uma distância aproximada de 930 milhões de quilômetros. Sabendo-se que o ano possui 365 dias e 5 horas, a velocidade média exercida pela Terra para executar essa órbita é, aproximadamente, de a) 106.103 km/h b) 1.061 km/h c) 106 km/h d) 10,6 km/h Questão 02 - (FMABC/2012) Duas esferas de dimensões desprezíveis dirigem-se uma ao encontro da outra, executando movimentos retilíneos e uniformes (veja a figura). As esferas possuem velocidades cujos módulos valem 4m/s e 7m/s. A distância entre elas nos 4 segundos que antecedem a colisão é de a) 50 b) 44 c) 28 d) 16 e) 12 Questão 03 - (UEL PR) Sabe-se que o cabelo de uma pessoa cresce em média 3cm a cada dois meses. Supondo que o cabelo não seja cortado e nem caia, o comprimento total, após terem se passado 10 anos será: a) 800mm b) 1200mm c) 1000mm d) 1800mm e) 150mm Questão 04 - (UFLA MG) Considerando uma partícula em movimento retilíneo com velocidade constante, as seguintes afirmações são CORRETAS, exceto: a) O momento linear da partícula mantém-se constante durante o movimento. b) A força resultante não-nula que atua na partícula é constante em módulo, direção e sentido. c) A posição da partícula dependerá linearmente do tempo. d) A energia cinética da partícula será conservada. e) A aceleração do movimento da partícula será nula. Questão 05 - (FURG RS) O gráfico representa o módulo das velocidades de dois automóveis como função do tempo. Com relação à área hachurada, podemos dizer que ela representa: a) a diferença entre as acelerações dos dois automóveis. b) a diferença entre as distâncias percorridas pelos dois automóveis. c) a aceleração do automóvel A em relação ao automóvel B. d) a diferença entre as velocidades dos dois automóveis. e) uma grandeza sem qualquer significado físico. Questão 06 - (FUVEST SP/2006) Um automóvel e um ônibus trafegam em uma estrada plana, mantendo velocidades constantes em torno de 100 km/h e 75 km/h, respectivamente. Os dois veículos passam lado a lado em um posto de pedágio. Quarenta minutos (2/3 de hora) depois, nessa mesma estrada, o motorista do ônibus vê o automóvel ultrapassá-lo. Ele
  • 4. 4 supõe, então, que o automóvel deve ter realizado, nesse período, uma parada com duração aproximada de a) 4 minutos b) 7 minutos c) 10 minutos d) 15 minutos e) 25 minutos Questão 07 - (UFLA MG) O gráfico abaixo representa a variação das posições de um móvel em função do tempo (S = f(t)). 10 0 -10 1 2 3 4 5 6 7 86 S(m) t(s) O gráfico de v x t que melhor representa o movimento acima é: 5 -5 10 0 V(m/s) 2 4 6 8 | | | | t(s) a. 5 -5 10 0 V(m/s) 2 4 6 8 | | | | t(s) b. 5 -5 10 0 V(m/s) 2 4 6 8 | | | | t(s) c. 5 -5 10 0 V(m/s) 2 4 6 8 | | | | t(s) d. Questão 08 - (UFPE) A equação horária para o movimento de uma partícula é x(t) = 15 – 2 t, onde x é dado em metros e t em segundos. Calcule o tempo, em s, para que a partícula percorra uma distância que é o dobro da distância da partícula à origem no instante t = 0 s. Questão 09 - (UERJ/2011) Uma partícula se afasta de um ponto de referência O, a partir de uma posição inicial A, no instante t = 0 s, deslocando-se em movimento retilíneo e uniforme, sempre no mesmo sentido. A distância da partícula em relação ao ponto O, no instante t = 3,0 s, é igual a 28,0 m e, no instante t = 8,0 s, é igual a 58,0 m. Determine a distância, em metros, da posição inicial A em relação ao ponto de referência O. Questão 10 - (PUC MG/2006) O gráfico mostra a velocidade (v) de um objeto em movimento retilíneo, em função do tempo t. Sobre o movimento do objeto, é CORRETO afirmar: a) Analisando-se o gráfico como um todo, pode-se afirmar que o objeto tende a parar. b) Entre os instantes 0 e 0,8 s , o objeto está em movimento retilíneo uniformemente variado. c) Até 1,2 s , a distância percorrida pelo móvel foi de 2,4 m. d) A partir do instante 1,2 s , o objeto passa a se se mover em movimento retilíneo uniforme. Questão 11 - (UEL PR) Em grandes cidades, a rota das ambulâncias leva em consideração fatores como proximidade do local do chamado e rapidez no deslocamento. Considere um chamado proveniente da região central de uma cidade, às 19h, conforme ilustra a figura, e que para atendê-lo, estão disponíveis quatro bases de ambulâncias, X, Y, W e Z. Para se definir a melhor rota, foram consideradas as velocidades médias desenvolvidas pelas ambulâncias em alguns intervalos de horários:
  • 5. 5 Assim, o chamado comunicado às 19h será atendido mais rapidamente pela ambulância da base a) X, seguindo pela rota 1. b) Z, seguindo pela rota 2. c) W, seguindo pela rota 3. d) Y, seguindo pela rota 4. e) Z, seguindo pela rota 5. Questão 12 - (UEL PR) Um pequeno animal desloca-se com velocidade média igual a 0,5 m/s. A velocidade desse animal em km/dia é: a) 13,8 b) 48,3 c) 43,2 d) 1,80 e) 4,30 Questão 13 - (UEL PR) Nos edifícios, os números que identificam os apartamentos representam vetorialmente suas posições, isto é, esses números são compostos por dígitos que representam o andar (posição na vertical) e a localização do apartamento no andar (posição na horizontal). Em um edifício de 10 andares, que tem um apartamento por andar, cada apartamento é identificado por um número que varia de 1 a 10. Se nesse edifício, cada andar tem altura de 5 metros, qual é a distância percorrida na direção vertical por alguém que sai do apartamento 3 e vai para o apartamento 9? a) 6 m b) 15 m c) 30 m d) 45 m e) 60 m Questão 14 - (UEL PR) O raio médio da órbita elíptica da Terra em torno do Sol é considerado para definir 1 Unidade Astronômica (U.A.): 1 U.A.  1,49108 km = 1,491011 m. A U.A. é utilizada para medir os raios das órbitas dos planetas do sistema solar, entretanto é uma unidade muito pequena para ser utilizada como parâmetro de medida para as distâncias das estrelas. Para essas distâncias, é utilizado o Ano-Luz (A. L.) que é a distância percorrida pela luz em um ano. Por exemplo, a estrela -Centauri está a 4,3 A. L. de distância da Terra. Se a velocidade de propagação da luz é igual a c = 3  108 m/s, é correto afirmar que a distância média entre o Sol e a Terra é de: a) 150 Segundos-Luz. b) 300 Segundos-Luz. c) 430 Segundos-Luz. d) 500 Segundos-Luz. e) 600 Segundos-Luz. Questão 15 - (UEM PR) Um trem se move com velocidade constante. Dentro dele estão o observador A e um garoto. Na estação, parado sobre a plataforma, está o observador B. Quando o trem passa pela plataforma, o garoto joga uma bola verticalmente para cima. Desprezando-se a resistência do ar, podemos afirmar que: 01. o observador A vê a bola se mover verticalmente para cima e cair nas mãos do garoto. 02. o observador B vê a bola descrever uma parábola e cair nas mãos do garoto. 04. os dois observadores vêem a bola se mover com a mesma aceleração. 08. o observador B vê a bola se mover verticalmente para cima e cair atrás do garoto. 16. o observador A vê a bola descrever uma parábola e cair atrás do garoto. Questão 16 - (UEM PR) Com velocidade constante, um caminhão se move num trecho retilíneo horizontal, sem atrito. Ele transporta, sobre a carroceria, pedras e um garoto. Se o garoto começa a arremessar pedras, pode–se concluir que a velocidade do caminhão, na direção inicial do movimento, 01. aumenta, se as pedras forem arremessadas para trás. 02. diminui, se as pedras forem arremessadas para frente. 04. diminui, se as pedras forem arremessadas verticalmente para cima. 08. aumenta, se as pedras forem arremessadas lateralmente, perpendicularmente à direção do movimento do caminhão. 16. permanece constante, qualquer que seja a direção em que o garoto arremessar as pedras. Questão 17 - (UEM PR) Os diagramas abaixo (a, b, c, d) mostram uma esfera movendo-se em quatro situações diferentes. Considerando que, em todas as situações, não existem forças dissipativas atuando, que, em a e em b, é dado um pequeno empurrão na esfera para que ela comece a se mover e que, em d, a colisão entre a esfera e a superfície é perfeitamente elástica, podese afirmar que, 01. em a, a esfera está em movimento retilíneo uniforme (MRU). 02. em b, a esfera está em MRU. 04. em c, a esfera está em movimento harmônico simples (MHS). 08. em d, a esfera está em MHS. 16. tanto em c como em d, decorrido algum tempo, a esfera pára. 32. tanto em a como em b, a força que a superfície faz sobre a esfera é constante. Questão 18 - (PUC MG)
  • 6. 6 A tabela abaixo contém as velocidades, consideradas constantes, em metros por segundo, que quatro nadadoras apresentaram na ida e na volta nadando estilo livre em uma piscina de 50 metros de comprimento. nadadora A B C D E ida 1,00 1,25 0,50 0,60 0,80 volta 1,00 0,80 1,60 0,90 0,70 Qual delas fez a virada em primeiro lugar? a) A b) B c) C d) D e) E Questão 19 Um candidato sai de sua residência para prestar vestibular pretendendo percorrer a distância total até o local da prova em uma hora, conduzindo seu automóvel com velocidade média de 60 km/h. Após percorrer os primeiros 10 km do percurso em 10 minutos, percebe que esqueceu o documento de identificação e retorna para apanhá-lo. Sua mãe o espera no portão com o documento. Desprezando-se o tempo para receber o documento e manobrar o carro, para que esse candidato consiga chegar ao local da prova no horário previsto anteriormente, ele deverá desenvolver no percurso de retorno à sua casa e ida até o local da prova uma velocidade média, em km/h, igual a a) 78. b) 84. c) 90. d) 98. e) 72. Questão 20 - (UnB DF) Qual é o tempo gasto para que um metrô de 200m a uma velocidade de 180km/h atravesse um túnel de 150m? Dê sua resposta em segundos. GABARITO: 1) Gab: A 2) Gab: B 3) Gab: D 4) Gab: B 5) Gab: B 6) Gab: C 7) Gab: B 8) Gab: 15s 9) Gab: 10,0m 10) Gab: D 11) Gab: E 12) Gab: C 13) Gab: C 14) Gab: D 15) Gab: 01-02-04 16) Gab: 01-02 17) Gab: 02-04-08 18) Gab: B 19) Gab: B 20) Gab: 07 21) Gab: 04 litros 22) Gab: B 23) Gab: A 24) Gab: C 25) Gab: C 26) Gab: A 27) Gab: B 28) Gab: E 29) Gab: D 30) Gab: D 31) Gab: D 32) Gab: B 33) Gab: a) vm  17,5 km/s b) 't = 16667 s  4,63 h c) 2181. 34) Gab: C 35) Gab: D 36) Gab: D 37) Gab: C 38) Gab: B 39) Gab: C 40) Gab: A
  • 7. 7 LISTA 2 - MOVIMENTO UNIFORMEMENTE VARIADO RESUMO DE CONTEÚDO O movimento de uma partícula só é considerado uniformemente variado quando o módulo de sua velocidade varia, aumentando ou diminuindo ao longo do tempo. Quando observamos que a velocidade aumenta ou diminui de maneira uniforme, independentemente da trajetória descrita por essa partícula, quer dizer que a partícula possui aceleração constante. Exemplo: Se a aceleração é 10m/s2 , a velocidade varia 10m/s a cada segundo, ou seja, significa que o valor da velocidade da partícula aumenta numa taxa constante. Classificação do Movimento uniformemente variado (MUV) Para descrevermos o movimento de uma partícula devemos adotar um referencial. Podemos classificar esse movimento como acelerado ou retardado: Se a variação do módulo da velocidade escalar é positivo, ou seja, o valor da velocidade escalar em módulo aumenta classificamos o movimento como acelerado. ( Movimento acelerado progressivo ou movimento acelerado regressivo). Se a variação do módulo da velocidade escalar é negativo, ou seja, o valor da velocidade escalar em módulo diminui, classificamos o movimento como retardado. ( Movimento retardado progressivo ou movimento retardado regressivo). Resumindo: Movimento acelerado - |v|=> aumenta. - a e v possuem o mesmo sinal. Movimento retardado - |v|=> diminui. - a e v possuem sinais contrários. Fonte: http://www.infoescola.com/fisica/movimento- uniformemente-variado-muv/ EXERCÍCIOS TEXTO: 1 - Leia o texto e analise o gráfico. Um objeto que não pode ser considerado uma partícula é solto de uma dada altura sobre um lago. O gráfico abaixo apresenta a velocidade desse objeto em função do tempo. No tempo t = 1, 0s, o objeto toca a superfície da água. Despreze somente a resistência no ar. Questão 01 - (UEL PR/2011) Qual a profundidade do lago? a) 1 m b) 5 m c) 7 m d) 100 m e) 1000 m Questão 02 - (ESCS DF/2009) A velocidade de um corpo em função do tempo é dada pelo gráfico: O espaço percorrido pelo corpo entre 0 e 4s é: a) 30m; b) 35m; c) 40m; d) 45m; e) 50m. Questão 03 - (UFF RJ) No gráfico, v2 indica o quadrado da velocidade e x, a posição. O movimento foi realizado em trajetória retilínea, partindo de posição inicial nula. V x0 2 O gráfico corresponde a um movimento: a) uniforme b) uniformemente acelerado com velocidade inicial não nula c) uniformemente retardado com velocidade inicial nula. d) uniformemente acelerado com velocidade inicial nula. e) uniformemente retardado com velocidade inicial não nula. Questão 04 - (FEEVALE RS) O gráfico abaixo representa a velocidade de um automóvel que se movimenta em uma avenida retilínea, partindo de um semáforo que abriu, até parar em um outro semáforo fechado.
  • 8. 8 t(s)604020 20 v(m/s) A distância entre os dois semáforos vale a) 200 m b) 400 m c) 600 m d) 800 m e) 1000 m Questão 05 - (CESJF MG) O gráfico abaixo representa a variação da velocidade de dois carros , A e B , em função do tempo . Os carros partem de uma mesma posição no mesmo instante , seguindo uma mesma trajetória retilínea e rumo a uma mesma cidade. 0 0,1 0,2 0,3 0,4 t(h) V(km/h) 60 80 B A Após 6,0 minutos de movimento a distância entre os carros é de : a) 1,0 km b) 3,0 km c) 6,0 km d) 5,0 km e) N. R. A . Questão 06 - (UNESP) Um atleta de corridas de curto alcance, partindo do repouso, consegue imprimir a si próprio uma aceleração constante de 5,0m/s2 durante 2,0s e, depois, percorre o resto do percurso com a mesma velocidade adquirida no final do período de aceleração. a) Esboce o gráfico da velocidade do atleta em função do tempo, numa corrida de 5s. b) Qual é a distância total que ele percorre nessa corrida de 5s? Questão 07 - (UNIRIO RJ) Analisando o gráfico abaixo, que relaciona a posição dos móveis A e B com o tempo t, assinale a opção correta. s A B t c a) VA = VB b) VA > VB c) VA < VB d) aA > aB e) aA < aB Questão 08 - (UNIRIO RJ) A velocidade de uma partícula varia com o passar do tempo conforme o gráfico abaixo. v(m/s) t(s)0 1 2 3 4 O seu deslocamento do instante 0 s até o instante 1 s foi de 1,5 m. Através da observação do gráfico podemos concluir que seu deslocamento entre os instantes 2 s e 3 s, em m, foi de: a) 2,0 b) 2,05 c) 3,0 d) 3,5 e) 4,0 Questão 09 - (UNIFOR CE) O gráfico fornece a velocidade de um corpo, que se move em linha reta, em função do tempo. Sabe-se que, no instante t  0, o corpo se encontra na posição 20 m. No instante t  8,0 s, o corpo estará na posição a) 40 m b) 20 m c) 20 m d) 40 m e) 60 m Questão 10 - (UNIFOR CE) O gráfico abaixo representa, em função do tempo, a velocidade escalar de uma partícula que está em movimento retilíneo.
  • 9. 9 O movimento é acelerado SOMENTE no trecho a) I b) II c) III d) IV e) V Questão 11 - (UNIOESTE PR/2007) Em uma competição esportiva entre escolas, dois alunos, João e Pedro, disputam quem vai chegar primeiro à posição C. João está inicialmente em repouso na posição A, conforme a figura abaixo, e precisa se deslocar 144 m para alcançar o ponto C, seguindo a trajetória retilínea AC. No mesmo instante em que João inicia seu movimento com uma aceleração uniforme de 0,5 m/s2 mantida em todo o trajeto, Pedro está passando pela posição B com uma velocidade de 2 m/s. Pedro segue a trajetória retilínea BC de comprimento igual a 145 m e alcança o ponto C no mesmo instante que João. Assinale a alternativa que fornece o valor da aceleração de Pedro, em metros por segundo ao quadrado (com arredondamento na segunda casa decimal), no trecho BC. a) 0,50 b) 0,01 c) 0,11 d) 0,17 e) 0,34. Questão 12 - (UNIOESTE PR) Um feixe de íons negativos é acelerado no interior de um tubo de raios catódicos. Cada íon é acelerado desde o repouso até a velocidade final de 2,0107 m/s em uma distância de 10 mm. Determine, em fN (femto newtons), o módulo da força resultante que atua sobre cada íon nesse tubo de raios catódicos. Use, para a massa do íon, o valor de 10,01031 kg e lembre-se de que o multiplicador f (femto) vale 1015 . Questão 13 - (UNIOESTE PR) Uma pedra é atirada verticalmente para cima, a partir de uma janela de um edifício, situada a 5 m de altura em relação ao solo, com uma velocidade inicial igual a 20 m/s. No mesmo instante, de uma janela situada a 25 m de altura em relação ao solo, exatamente acima da primeira janela, é atirada verticalmente para cima uma outra pedra com uma velocidade inicial de 10 m/s. Desprezando a resistência do ar, determine após quantos segundos, a partir do instante do arremesso, as duas pedras estarão à mesma altura. Questão 14 - (UNIOESTE PR/2006) Considere a queda livre de um corpo a partir do repouso. Tome g = 10,0 m/s2 . Com relação a esse movimento, pode-se afirmar que 00. a aceleração do corpo é constante e igual a 10,0m/s2 , independente de considerar o atrito com o ar. 01. a distância que o corpo cai após 3,0s é de 50,0m, quando não há atrito. 02. se não há atrito com o ar, a sua velocidade, após ter caído 20,0m, é de 20,0m/s. 03. caso não haja atrito com o ar, o tempo necessário para o corpo atingir uma velocidade de 40,0m/s é de 6,0 s. 04. não havendo atrito com o ar, o tempo gasto para o corpo cair 500,0m é de 10,0s. Questão 15 - (UNIOESTE PR/2009) Em uma pista de testes um automóvel, partindo do repouso e com aceleração constante de 3 m/s2 , percorre certa distância em 20 s. Para fazer o mesmo trajeto no mesmo intervalo de tempo, porém com aceleração nula, um segundo automóvel deve desenvolver velocidade de a) 20 m/s b) 25 m/s c) 80 km/h d) 100 km/h e) 108 km/h Questão 16 - (UEL PR/2011) No circuito automobilístico de Spa Francorchamps, na Bélgica, um carro de Fórmula 1 sai da curva Raidillion e, depois de uma longa reta, chega à curva Les Combes. Figura: Circuito automobilístico de Spa Francorchamps A telemetria da velocidade versus tempo do carro foi registrada e é apresentada no gráfico a seguir. Qual das alternativas a seguir contém o gráfico que melhor representa a aceleração do carro de F-1 em função deste mesmo intervalo de tempo?
  • 10. 10 a) b) c) d) e) Questão 17 Analise as alternativas abaixo e assinale o que for correto. 01. O gráfico da velocidade em função do tempo, para um móvel descrevendo um Movimento Retilíneo e Uniforme, é uma reta paralela ao eixo dos tempos. 02. O gráfico da posição em função do tempo, para um móvel descrevendo um movimento Retilíneo e Uniforme, é uma reta, e o coeficiente angular dessa reta fornece a velocidade do móvel. 04. O gráfico do espaço percorrido em função do tempo é uma reta para um móvel que realiza um Movimento Uniforme qualquer. 08. O espaço percorrido por um móvel, em um dado intervalo de tempo, pode ser obtido calculando-se a “área sob a curva” do gráfico da velocidade em função do tempo, para aquele dado intervalo de tempo. 16. O gráfico da velocidade em função do tempo, para um móvel descrevendo um Movimento Retilíneo Uniformemente Variado, é uma parábola. Questão 18 - (UEL PR) O que acontece com o movimento de dois corpos, de massas diferentes, ao serem lançados horizontalmente com a mesma velocidade, de uma mesma altura e ao mesmo tempo, quando a resistência do ar é desprezada? a) O objeto de maior massa atingirá o solo primeiro. b) O objeto de menor massa atingirá o solo primeiro. c) Os dois atingirão o solo simultaneamente. d) O objeto mais leve percorrerá distância maior. e) As acelerações de cada objeto serão diferentes. Questão 19 - (UEL PR) Um motorista dirige um automóvel a 72 km/h quando percebe que o semáforo a sua frente está fechado. Ele pisa, então, no pedal do freio e a velocidade do automóvel diminui como mostra o gráfico abaixo. 0 5,0 t(s) 72 v(km/h) A menor distância que o automóvel deve estar do semáforo, no instante em que o motorista pisa no pedal do freio, para que não avance o semáforo é, em metros, a) 144 b) 72 c) 50 d) 30 e) 18 Questão 20 - (UEL PR/2008) No departamento de Física da UEL, foi realizado um experimento de queda livre cuja equação de movimento foi obtida com auxílio de um computador. O experimento consistiu na aquisição de um sinal elétrico cada vez que um objeto, em queda, interrompia um feixe de luz laser que era direcionado por espelhos (separados por 2 cm) até ser coletado numa fotocélula (ver figura). O objeto de estudo foi uma pena, para qual a resistência do ar não pode ser desprezada. A fotocélula, quando recebia luz, produzia uma tensão elétrica, e o tempo entre as interrupções de luz eram registradas pelo computador. Ao final da queda, obteve- se um gráfico de espaço percorrido versus tempo (S x t) cujos dados são mostrados no gráfico abaixo. Pelo arranjo experimental, conseguiu-se simplificar a equação que descreve o movimento, uma vez que o espaço inicial, bem como a velocidade inicial, puderam ser considerados zero.
  • 11. 11 Sabendo que a função polinomial que descreve o movimento é do tipo 2 CxBxAxF )( , e que cm0SA  e cm/s 0vB , qual o valor aproximado da aceleração “a” da pena em cm/s2 ? a) 9,8 cm/s2 . b) 12,2 cm/s2 . c) 5,3 cm/s2 . d) 3,6 cm/s2 . e) 7,1 cm/s2 . GABARITO: 1) Gab: C 2) Gab: C 3) Gab: B 4) Gab: D 5) Gab: A 6) Gab: a) V(m/s) t(s) 10 5 0 1 2 3 4 5 ; b) d = 40m 7) Gab: A 8) Gab: D 9) Gab: E 10) Gab: D 11) Gab: E 12) Gab: 20 13) Gab: 14) Gab: FFVFV 15) Gab: E 16) Gab: D 17) Gab: 15 18) Gab: C 19) Gab: C 20) Gab: E 21) Gab: 03 22) Gab: 07 23) Gab: 25 24) Gab: 30 25) Gab: 15 26) Gab: B 27) Gab: D 28) Gab: D 29) Gab: a) a = 6,0m/s2 ; b) v = 11m/s; c) vH = 6,2m/s 30) Gab: 01-F; 02-V; 04-F; 08-V; 16-F; 32-F. 31) Gab: 05 32) Gab: B 33) Gab: 11 34) Gab: E 35) Gab: 29 36) Gab: B 37) Gab: A 38) Gab: 14 39) Gab: 20 40) Gab: 27
  • 12. 12 LISTA 3 - LANÇAMENTO OBLÍQUO RESUMO TEÓRICO Quando uma bola é chutada em uma partida de futebol, podemos observar que ela realiza um movimento parabólico. Esse movimento é chamado de lançamento oblíquo. Considere um corpo sendo lançado a partir do solo, formando um ângulo α com a horizontal, com velocidade inicial v0. Desprezando as forças dissipativas, o corpo fica sujeito apenas à ação da gravidade, descrevendo uma trajetória parabólica. Movimento Horizontal Assim como no Lançamento Horizontal, o movimento na direção do eixo x, no lançamento oblíquo, é uniforme, pois a velocidade é constante. Portanto, a função horária do movimento horizontal é: x = vx.t A distância horizontal percorrida pelo corpo desde o lançamento é chamada alcance máximo. Podemos determinar o alcance máximo pela equação: Para determinar a posição do móvel em relação à horizontal temos que determinar a componente da velocidade inicial v0 na direção do eixo x. O módulo da velocidade na direção do eixo x é: vx = v0 . cosα Movimento Vertical O movimento vertical está sob a ação da gravidade, isso implica que o movimento é uniformemente variado e a velocidade vy diminui à medida que a altura em relação ao solo aumenta. O componente da velocidade inicial na direção do eixo y é: v0y = v0 . senα As funções horárias do movimento vertical são: Função horária do espaço y = v0yt + gt2 2 Função horária da velocidade vy = v0y + gt Equação de Torricelli vy 2 = v0y 2 + 2gy A altura máxima pode ser calculada usando a equação: Após atingir a altura máxima, o corpo move-se em queda livre sob ação da gravidade, e sua velocidade aumenta com o tempo. EXERCÍCIOS TEXTO: 1 - Comum à questão: 1 Três bolas – X, Y e Z – são lançadas da borda de uma mesa, com velocidades iniciais paralelas ao solo e mesma direção e sentido. A tabela abaixo mostra as magnitudes das massas e das velocidades iniciais das bolas. 810Z 105Y 205X (m/s) INICIALVELOCIDADE )g( MASSA BOLAS Questão 01 - (UERJ/2012) As relações entre os respectivos alcances horizontais Ax, Ay e Az das bolas X, Y e Z, com relação à borda da mesa, estão apresentadas em: a) Ax < Ay < Az b) Ay = Ax = Az c) Az < Ay < Ax d) Ay < Az < Ax Questão 02 - (UEM PR) Dois corpos idênticos A e B são lançados, simultaneamente, da mesma posição, com a mesma velocidade inicial, formando o mesmo ângulo a com a horizontal. Sobre o corpo A, atua apenas a força peso. Sobre o corpo B, além do próprio peso, atua, favoravelmente ao movimento, uma força horizontal constante. Pode-se afirmar que os corpos 01. chegam ao solo simultaneamente. 02. têm o mesmo alcance horizontal. 04. atingem a mesma altura máxima. 08. têm a mesma velocidade quando atingem o solo. 16. têm a mesma aceleração. Questão 03 - (PUC RJ) Três bolinhas são largadas da mesma altura h como mostra a figura. As bolinhas A e B parte do repouso e a bolinha C tem velocidade inicial vo horizontal. A B C S h Vo Qual é a afirmativa correta? despreze a resistência do ar e o atrito da bolinha A com o solo. a) As três bolinhas chegam ao mesmo tempo na região horizontal S do solo. b) A bolinha A chega primeiro em S e as bolinhas B e C chegam juntas depois. c) bolinha B chega primeiro em S, em seguida a bolinha A e por último a bolinha C.
  • 13. 13 d) As bolinhas A e B chegam juntas em S e depois a bolinha C. e) As boinhas B e C chegam juntas em S e depois chega a bolinha A. Questão 04 - (UNESP/2007) Em uma partida de futebol, a bola é chutada a partir do solo descrevendo uma trajetória parabólica cuja altura máxima e o alcance atingido são, respectivamente, h e s. Desprezando o efeito do atrito do ar, a rotação da bola e sabendo que o ângulo de lançamento foi de 45º em relação ao solo horizontal, calcule a razão s/h. Dado: 2/245cos45sen 00  Questão 05 - (UFU MG) Da superfície de uma mesa são lançadas horizontalmente duas pequenas esferas A e B, com velocidades iniciais v0 e 2v0, respectivamente. Se a resistência do ar for considerada irrelevante, como se relacionam os tempos de queda das esferas? a) tA = tB b) tA = tB/4 c) tA = tB/2 d) tA = 4tB e) tA = 2tB Questão 06 - (FFFCMPA RS/2007) Uma pedra pe arremessada horizontalmente, com uma velocidade de 20m/s, de uma ponte que está a 16m acima da superfície da água. Qual a velocidade da pedra, após atingir a água? (Considere g = 9,8m/s2 ) a) 26,7m/s. b) 13,5m/s. c) 713,6m/s. d) 42,5m/s. e) 246,4m/s. Questão 07 - (UFF RJ/2011) Após um ataque frustrado do time adversário, o goleiro se prepara para lançar a bola e armar um contraataque. Para dificultar a recuperação da defesa adversária, a bola deve chegar aos pés de um atacante no menor tempo possível. O goleiro vai chutar a bola, imprimindo sempre a mesma velocidade, e deve controlar apenas o ângulo de lançamento. A figura mostra as duas trajetórias possíveis da bola num certo momento da partida. Assinale a alternativa que expressa se é possível ou não determinar qual destes dois jogadores receberia a bola no menor tempo. Despreze o efeito da resistência do ar. a) Sim, é possível, e o jogador mais próximo receberia a bola no menor tempo. b) Sim, é possível, e o jogador mais distante receberia a bola no menor tempo. c) Os dois jogadores receberiam a bola em tempos iguais. d) Não, pois é necessário conhecer os valores da velocidade inicial e dos ângulos de lançamento. e) Não, pois é necessário conhecer o valor da velocidade inicial. Questão 08 - (UFC CE/2007) Considere o lançamento de dois projéteis com o mesmo ângulo inicial. O projétil 1 tem a metade da massa do outro (denotado por 2). Qual a relação entre as velocidades iniciais dos projéteis para que tenham o mesmo alcance? a) 21 v 2 1 v  b) v1 = 2v2 c) v1 = v2 d) 21 v 3 1 v  e) v1 = 3v2 TEXTO: 2 - Comuns às questões: 9, 10 Considere o enunciado abaixo. Na figura que segue, estão apresentadas as trajetórias de dois projéteis, A e B, no campo gravitacional terrestre. O projétil A é solto da borda de uma mesa horizontal de altura H e cai verticalmente; o projétil B é lançado da borda dessa mesa com velocidade horizontal de 1,5 m/s. (O efeito do ar é desprezível no movimento desses projéteis.) Questão 09 - (UFRGS/2007) Se o projétil A leva 0,4 s para atingir o solo, quanto tempo levará o projétil B? a) 0,2 s b) 0,4 s c) 0,6 s d) 0,8 s e) 1,0 s Questão 10 - (UFRGS/2007) Qual será o valor do alcance horizontal X do projétil B? a) 0,2 m b) 0,4 m c) 0,6 m d) 0,8 m e) 1,0 m TEXTO: 3 - Comum à questão: 11 Dados que podem ser necessários: valor da aceleração da gravidade: g = 10,0 m/s2 ; velocidade da luz no vácuo: c = 3,0 x 108 m/s; equivalente mecânico da caloria: 1 cal = 4,186 joules; duração do intervalo de tempo de um ano na Terra: 3,0 x 107 s; Questão 11 - (UNIOESTE PR/2008) Considere a seguinte situação: uma bola de futebol foi chutada por um jogador com um ângulo de 30 graus acima da horizontal com uma velocidade inicial de módulo 20,0 m/s, passando a executar um movimento em duas dimensões, em um plano vertical. Suponha que
  • 14. 14 a resistência do ar seja desprezível e que a aceleração da gravidade não varie, tendo sempre o valor de 10,0 m/s2 . Com base no enunciado acima, considere as afirmativas a seguir: I. A força que acelera a bola de futebol tem um valor que depende da altura na qual a bola estiver situada. II. O movimento da bola de futebol pode ser decomposto nas direções horizontal e vertical e os movimentos nas duas direções podem ser considerados independentes entre si. III. A quantidade de movimento da bola é uma grandeza escalar que se conserva durante o movimento da bola. IV. A energia mecânica da bola é uma grandeza escalar que pode ser expressa em kW·h e que se conserva durante o movimento da bola. Estão corretas apenas as afirmativas: a) I e II. b) II e IV. c) III e IV. d) I, II e III. e) I, III e IV. Questão 12 - (UEL PR/2010) Um sistema mecânico que consiste de um pequeno tubo com uma mola consegue imprimir a uma esfera de massa m uma velocidade fixa v0. Tal sistema é posto para funcionar impulsionando a massa na direção vertical, a massa atingindo a altura máxima h e voltando a cair. Em seguida o procedimento é efetuado com o eixo do tubo formando um determinado ângulo com a direção horizontal de modo que o alcance R nesta direção seja maximizado. Tais situações estão representadas na figura a seguir. Os experimentos ocorrem em um local onde a aceleração da gravidade g′ é um pouco menor que seu valor na superfície terrestre g = 9,8 m/s2 . Baseado nesses dados e concordando com expressões cinemáticas para os movimentos de queda livre e lançamento oblíquo, é correto afirmar: a) A razão R h obedecerá a relação 'g2 g R h  b) A razão R h obedecerá a relação 'g g2 R h  c) A razão R h obedecerá a relação g2 'g R h  d) A distância R a ser alcançada pela massa será a mesma que se obteria em um experimento na superfície terrestre porque tal quantidade só depende do valor da componente horizontal da velocidade v0 cos(θ). e) R e h serão diferentes de seus valores obtidos em experimentos realizados na superfície mas a relação 2 1 R h  se manterá porque esta independe do valor local da aceleração da gravidade. Questão 13 - Um jogador de golf desfere uma tacada, imprimindo à bola uma velocidade inicial com módulo v0 = 20 m/s e ângulo  = 45º em relação ao eixo-x horizontal, de acordo com a figura abaixo. Desprezando a resistência aerodinâmica do ar e considerando que o módulo da aceleração da gravidade vale g = 10 m/s2 , determine o alcance máximo A da bola de golf. a) 4 metros b) 200 metros c) 100 metros d) 40 metros e) 2 metros Questão 14 - (UEM PR/2013) Uma pequena esfera é lançada do solo com velocidade inicial de módulo vo, em uma direção que forma um ângulo  com a horizontal. Desprezando o atrito com o ar, assinale o que for correto. 01. A função horária da posição da esfera no eixo horizontal é uma função de primeiro grau. 02. Tanto a função horária da posição da esfera no eixo vertical, quanto a função que descreve a trajetória da esfera (função da posição da esfera no eixo vertical em relação ao eixo horizontal) são funções de segundo grau. 04. A componente horizontal da velocidade inicial da esfera é dada por vo cos. 08. O intervalo de tempo que a esfera leva até atingir a altura máxima é igual ao que ela levaria se fosse lançada verticalmente para cima com a mesma velocidade inicial vo. 16. O módulo da velocidade com que a esfera atinge o solo é vo. Questão 15 - (UEM PR/2006) Em uma cena de filme, um policial em perseguição a um bandido salta com uma moto do topo de um prédio a outro. Considere que ambos os prédios têm o topo quadrado com uma área de 900 m2 e que o policial motorizado se lança horizontalmente com uma velocidade de 72 km/h. Considere ainda que a distância
  • 15. 15 entre os prédios é de 20 m e que o topo do segundo prédio está 10 m abaixo do topo do primeiro. Nessas condições, pode-se afirmar que essa cena poderia ser real? (Considere a aceleração gravitacional igual a 10 m/s2 . Despreze a resistência do ar.) a) Sim, pois o policial alcançaria o topo do segundo prédio aproximadamente 8 m após a primeira borda do prédio. b) Não, pois com essa velocidade inicial, o policial ultrapassaria o topo do segundo prédio. c) Não, pois o policial cairia entre os prédios em queda livre. d) Não, pois o policial atingiria a parede lateral do prédio em alguma altura do edifício. e) Não, pois o policial alcançaria o topo do segundo prédio a aproximadamente 0,5 m da segunda borda do prédio, sem espaço suficiente para parar a moto. Questão 16 - (UEM PR) Uma pedra é lançada com um ângulo de 45º em relação ao eixo horizontal x e na direção positiva de x. Desprezando-se a resistência do ar, quais dos gráficos melhor representam a componente horizontal da velocidade (vx) versus tempo (t) e a componente vertical da velocidade (vy) versus tempo (t), respectivamente? I. II. III. IV. V. vx versus t vy versus t a) I e IV b) II e I c) II e III d) II e V e) IV e V Questão 17 - (UEM PR/2008) Um objeto ao nível do mar é lançado obliquamente com velocidade inicial de 100,0 m/s, com um ângulo de lançamento  tal que o 6,0)cos(  (obs.: despreze a resistência do ar). Considere g = 10,0 m/s2 . Assinale o que for correto. 01. As componentes horizontal e vertical da velocidade no instante de lançamento são vx = 60,0 m/s e vy = 80,0 m/s. 02. Desprezando a resistência do ar, o objeto não retorna ao nível de lançamento. 04. O alcance máximo do objeto é superior a 500 m. 08. O tempo necessário para o objeto atingir o alcance máximo é 16,0 s. 16. O módulo da componente da velocidade no eixo paralelo ao solo se mantém constante durante o percurso. Questão 18 - (UFG GO) 50 ANOS DE BOMBA ATÔMICA As explosões das bombas atômicas, em agosto de 1945, sobre as cidades de Hiroshima e Nagasaki, fizeram, em 1995, cinqüenta anos. O fim da guerra e o início da era atômica se anunciavam e o clarão escurecia os olhares do mundo que oscilavam entre o pavor e o medo. O desconhecido explodia em solo japonês. A bomba detonava os horrores da guerra, levantava a poeira atômica das nações rivais e emitia sinais de que os tempos seriam outros para as nações. Por certo, nunca mais seriam os mesmos para os “filhos do clarão.” Fontes: Superinteressante, ano 09,nº07, 1995; Veja, 02/08/95. Considerando que, nas proposições a seguir, as informações técnicas (nomes, distâncias, massas, etc.), a respeito dos lançamentos e efeitos das bombas atômicas sobre Hiroshima e Nagasaki, são verdadeiras, analise as proposições, sob o aspecto da Física, e identifiq44ue as correta. 01. o avião B-29, chamado “Enola Gay”, soltou a bomba apelidada de “Little Boy” sobre Hiroshima. A bomba foi solta a 9600m de altura e explodiu a aproximadamente 580m do solo. Se a resistência do ar fosse desprezada, a bomba levaria aproximadamente 32 segundos para explodir (adotar g = 10m/s2 ); 02. o avião B-29, chamado “Bock’s Car”, soltou a bomba apelidada de “Fat Man” acima de Nagasaki. A intenção era atingir o centro da cidade, mas por problemas de visibilidade atingiu a periferia. Ao soltar a bomba o avião estava justamente acima do ponto atingido; 04. pessoas que estavam distantes do ponto da explosão, escutaram primeiramente o som e depois viram o clarão da explosão; 08. “Little Boy”, de 4,0 toneladas, constituída de urânio, possuía poder de destruição equivalente a 15.000 toneladas de TNT e “Fat Man”, de 4,5 toneladas, constituída de plutônio, a 20.000 toneladas de TNT. Se a resistência do ar fosse desprezada e as duas bombas fossem soltas da mesma altura e no mesmo instante, “Fat Man” atingiria o solo antes de “Little Boy ”. 16. Segundo o físico Naomi Shohno, o deslocamento de ar provocado pelas bombas percorreu 740m no segundo posterior à explosão, 4 km em 10 segundos e 11 km em 30 segundos. Portanto, a velocidade média de deslocamento do ar no
  • 16. 16 primeiro segundo foi de 2664 km/h e em 30 segundos foi de aproximadamente 1320km/h. Questão 19 - (UFOP MG) Uma partícula desloca-se em movimento retilíneo uniforme sobre uma plataforma horizontal lisa do ponto A ao ponto B, com velocidade v0 = 10m/s. A partir do ponto B, a partícula se movimenta sob a ação de seu peso até atingir o ponto D localizado em outra plataforma horizontal, como mostra a figura abaixo. a) Calcule a distância do ponto C ao ponto D. b) Calcule o tempo que a partícula gasta para se deslocar do ponto A ao ponto D. c) Determine a velocidade da partícula imediatamente antes de atingir o ponto D. Dados: AB = 10m; BC = 20m; g = 10m/s2 . Questão 20 - (ITA SP) Uma bola é lançada horizontalmente do alto de um edifício, tocando o solo decorridos aproximadamente 2s. Sendo de 2,5m a altura de cada andar, o número de andares do edifício é a) 5 b) 6 c) 8 d) 9 e) indeterminado pois a velocidade horizontal de arremesso da bola não foi fornecida. GABARITO: 1) Gab: C 2) Gab: 01-04 3) Gab: E 4) Gab: 4 h s  5) Gab: A 6) Gab: A 7) Gab: B 8) Gab: C 9) Gab: B 10) Gab: C 11) Gab: B 12) Gab: E 13) Gab: D 14) Gab: 23 15) Gab: A 16) Gab: C 17) Gab: 29 18) Gab: 01-F; 02-F; 04-F; 08-F; 16-V. 19) Gab: a) 20m; b) 3s; c) vR = 22,4m/s 20) Gab: C 21) Gab: D 22) Gab: D 23) Gab: D 24) Gab: A 25) Gab: 17 26) Gab: C 27) Gab: B 28) Gab: D 29) Gab: 04-08-16 30) Gab: B 31) Gab: D 32) Gab: A 33) Gab: E 34) Gab: B 35) Gab: 05 36) Gab: D 37) Gab: 19 38) Gab: 31 39) Gab: B 40) Gab: 21
  • 17. 17 LISTA 4 - MOVIMENTO CIRCULAR RESUMO TEÓRICO O movimento circular uniforme (MCU) é o movimento no qual o corpo descreve trajetória circular, podendo ser uma circunferência ou um arco de circunferência. A velocidade escalar permanece constante durante todo o trajeto e a velocidade vetorial apresenta módulo constante, no entanto sua direção é variável. A aceleração tangencial é nula (at = 0), porém, com a aceleração centrípeta não ocorre o mesmo, ou seja, a aceleração não é nula (ac ≠ 0). A direção da aceleração centrípeta, em cada ponto da trajetória, é perpendicular à velocidade vetorial e aponta para o centro da trajetória. O módulo da aceleração centrípeta é escrito da seguinte forma: ac = v2 /r, onde r é o raio da circunferência descrita pelo móvel. Um corpo que descreve um movimento circular uniforme passa de tempo em tempo no mesmo ponto da trajetória, sempre com a mesma velocidade. Assim, podemos dizer que esse movimento é repetitivo, e pode ser chamado de movimento periódico. Nos movimentos periódicos existem dois conceitos muito importantes que são: frequência e período. Frequência: é o número de voltas que o corpo efetua em um determinado tempo (f = 1/ T). Período: é o tempo gasto para se completar um ciclo (T = 1/ f). Ao observar a definição de período e de frequência podemos dizer que o período é o inverso da frequência. Equações do Movimento Circular As equações que determinam o movimento circular são as seguintes: Posição angular: S = φ .R, onde R é o raio da circunferência. Velocidade angular média: ωm = Δφ/Δt Aceleração centrípeta: ac = v2 /R, onde R é o raio da circunferência. Força Centrípeta Para que um móvel possa descrever o movimento circular uniforme é necessário que esteja atuando uma força sobre ele, de modo que faça com que ele mude de posição, pois se tal fato não ocorrer o móvel passaria a descrever um movimento retilíneo uniforme. Essa força tem o nome de força centrípeta, e matematicamente é descrita da seguinte forma: Fc = m. ac Onde ac é a aceleração centrípeta, ac = v2 /R. Substituindo na equação acima temos: Fc = m. v2 /R A força centrípeta é sempre direcionada para o centro da circunferência. No cotidiano existem alguns exemplos de força centrípeta como a secadora de roupas e os satélites que ficam em órbita circular em torno do centro da Terra. Fonte: http://www.brasilescola.com/fisica/movimento- circular.htm EXERCÍCIOS Questão 01 - (UNIMAR SP) Um ciclista descreve um movimento circular uniforme no sentido anti-horário, conforme a trajetória abaixo. No ponto X, o vetor aceleração é melhor ilustrado por: .X a)  b)  c)  d)  e)  Questão 02 - (PUC RS) A velocidade angular do movimento de rotação da Terra é, aproximadamente, a) (/12) rad/h b) (/6) rad/h c) (/4) rad/h d)  rad/h e) 2 rad/h Questão 03 - (UECE) Em um relógio, o período de rotação do ponteiro dos segundos, o dos minutos e o das horas são, respectivamente: a) um segundo, um minuto e uma hora b) um minuto, uma hora e um dia c) um minuto, meia hora e um dia d) um minuto, uma hora e meio dia Questão 04 - (UFSC/2008) Um carro com velocidade de módulo constante de 20 m/s percorre a trajetória descrita na figura, sendo que de A a C a trajetória é retilínea e de D a F é circular, no sentido indicado. Assinale a(s) proposição(ões) CORRETA(S). 01. O carro tem movimento uniforme de A até C. 02. O carro tem movimento uniforme de A até F. 04. O carro tem aceleração de A até C. 08. O carro tem aceleração de D até F. 16. O carro tem movimento retilíneo uniformemente variado de D até F. Questão 05 - (UFU MG/2007) Três rodas de raios Ra, Rb e Rc possuem velocidades angulares wa, wb e wc, respectivamente, e estão ligadas entre si por meio de uma correia, como ilustra figura abaixo.
  • 18. 18 Ao mesmo tempo que a roda de raio Rb realiza duas voltas, a roda de raio Rc realiza uma volta. Não há deslizamento entre as rodas e a correia. Sendo ac R3R  , é correto afirmar que: a) caab w 3 4 weR 3 4 R  . b) caab w3weR 3 4 R  . c) caab w 3 4 weR 2 3 R  . d) caab w3weR 2 3 R  . Questão 06 - (UNINOVE SP/2009) As rodas de um automóvel têm diâmetro de 60cm. Quando o veículo transita a 36 km/h e suas rodas não derrapam sobre o piso, a frequência com que elas giram é, em Hz, de, aproximadamente, a) 16,7. b) 10,6. c) 5,3. d) 2,7. e) 1,4. Questão 07 - (UFT TO/2008) Em uma aula de física, os alunos observam um objeto descrevendo um movimento circular uniforme. Seja v  a velocidade e v  a aceleração do objeto. Após observarem o fenômeno, fazem os seguintes comentários: I. No movimento circular uniforme a soma das forças que agem no objeto não é nula, portanto existe aceleração não nula. II. No movimento circular uniforme v  muda constantemente, enquanto a velocidade angular é constante. a) Os comentários I e II estão corretos. b) Os comentários I e II estão errados. c) Apenas o comentário I está correto. d) Apenas o comentário II está correto. Questão 08 - (ESPCEX/2009) Uma máquina industrial é movida por um motor elétrico que utiliza um conjunto de duas polias, acopladas por uma correia, conforme figura abaixo. A polia de raio R1 = 15 cm está acoplada ao eixo do motor e executa 3000 rotações por minuto. Não ocorre escorregamento no contato da correia com as polias. O número de rotações por minuto, que a polia de raio R2 = 60 cm executa, é de Desenho Ilustrativo a) 250 b) 500 c) 750 d) 1000 e) 1200 Questão 09 - (FAMECA SP/2010) A relação entre as velocidades angulares de duas pessoas paradas, em relação à Terra, uma sobre o equador terrestre e outra, no polo norte, é a) zero. b) 1/24. c) 1/12. d) 1. e) 6. Questão 10 - (UNIMONTES MG/2010) Na figura, estão representadas duas polias, A e B, com raios RA < RB, acopladas por um eixo. É CORRETO afirmar: a) As velocidades angulares dos pontos periféricos da polia A são iguais às dos pontos periféricos da polia B. b) As velocidades angulares dos pontos periféricos da polia A são maiores do que as dos pontos periféricos da polia B. c) As velocidades lineares dos pontos periféricos da polia A são iguais às dos pontos periféricos da polia B. d) As velocidades lineares dos pontos periféricos da polia A são maiores do que as dos pontos periféricos da polia B. Questão 11 - (UEL) Considere as seguintes afirmativas: I. No movimento circular uniforme, os vetores velocidade e aceleração são perpendiculares entre si. II. Objetos de mesma forma e dimensões, mas com massas diferentes, quando soltos de uma mesma altura, por estarem sob a influência da mesma aceleração gravitacional, chegam ao solo no mesmo instante. III. Do ponto de vista microscópico, as forças responsáveis pelo atrito entre duas superfícies são as forças gravitacionais que atuam nas regiões em que as duas superfícies estão em contato. Assinale a alternativa correta.
  • 19. 19 a) Apenas as afirmativas I e III são verdadeiras. b) Apenas as afirmativas II e III são verdadeiras. c) Apenas as afirmativas I e II são verdadeiras. d) Apenas a afirmativa III é verdadeira. e) Todas as afirmativas são verdadeiras. Questão 12 - (UEL PR) A bicicleta tem o pedal preso a um disco denominado “coroa”. A corrente liga a coroa à catraca, que é o disco preso à roda traseira. A cada pedalada, a catraca gira várias vezes, pois seu diâmetro é menor que o diâmetro da coroa. Qual é a distância percorrida por uma bicicleta de aro 33 (raio da roda igual a 33cm), cuja coroa tem raio três vezes maior que o raio da catraca, no período igual a uma pedalada? a) 5,3 m b) 5,7 m c) 6,2 m d) 6,8 m e) 7,1 m Questão 13 - (UEL PR/2006) Os primeiros relógios baseavam-se no aparente movimento do Sol na abóboda celeste e no deslocamento da sombra projetada sobre a superfície de um corpo iluminado pelo astro. Considere que: a Terra é esférica e seu período de rotação é de 24 horas no sentido oeste-leste; o tempo gasto a cada 15º de rotação é de 1 hora; o triângulo Brasília/Centro da Terra/Luzaka (Zâmbia) forma, em seu vértice central, um ângulo de 75°. A hora marcada em Luzaka, num relógio solar, quando o sol está a pino em Brasília é: a) 5 horas. b) 9 horas. c) 12 horas. d) 17 horas. e) 21 horas. Questão 14 - (UEL PR/2010) Considere uma esfera sólida de raio r e momento de inércia inicial 2 ii mr 5 2 I  que gira com período T ao redor de um eixo vertical que passa por seu centro. Essa esfera possui matéria uniformemente distribuída através de seu volume. Devido a um desequilíbrio de forças, essa matéria rearranja-se em uma nova configuração de equilíbrio cuja geometria é a de uma casca com formato esférico e momento de inércia final 2 ff mr 3 2 I  . Sob que condições o período de rotação da esfera permanecerá inalterado? a) Aumento na velocidade de rotação . b) Esta condição será satisfeita se os raios iniciais e finais forem iguais: ri = rf já que neste caso o momento angular será conservado. c) A conservação do momento angular implica em uma diminuição da velocidade angular e ao mesmo tempo um aumento no raio da esfera de forma que if r 3 5 r  . d) A conservação do momento angular implica em uma diminuição do raio da esfera de forma que if r 3 2 r  . e) A conservação do momento angular implica em uma diminuição do raio da esfera de forma que if r 5 3 r  . Questão 15 - (UEL PR/2010) Um ciclista descreve uma volta completa em uma pista que se compõe de duas retas de comprimento L e duas semicircunferências de raio R conforme representado na figura a seguir. A volta dá-se de forma que a velocidade escalar média nos trechos retos é v e nos trechos curvos é v 3 2 . O ciclista completa a volta com uma velocidade escalar média em todo o percurso igual a v 5 4 . A partir dessas informações, é correto afirmar que o raio dos semicírculos é dado pela expressão: a) L = πR b) 2 R L   c) 3 R L   d) 4 R L   e) 2 R3 L   Questão 16 - (UEM PR) Um carro se move com velocidade constante em uma estrada curva num plano horizontal. Desprezando-se a resistência do ar, pode-se afirmar corretamente que sobre o carro atua; 01. uma força na mesma direção e em sentido contrário ao centro da curva. 02. uma força de atrito na mesma direção e no mesmo sentido do centro da curva. 04. uma força perpendicular à trajetória e dirigida para cima.
  • 20. 20 08. uma força perpendicular à trajetória e dirigida para baixo. 16. uma força na mesma direção e no mesmo sentido do movimento do carro. Questão 17 - (UEM PR) Uma barra constituída de material isolante tem, em cada extremidade, uma carga de 1 C. Se a barra girar em torno de seu ponto médio com velocidade angular w = 8  rad/s, podemos afirmar que 01. a barra completará quinze voltas em cada segundo. 02. a corrente elétrica proporcionada pelo movimento da barra será de 8 A. 04. o período das oscilações da barra será de 0,13 segundos. 08. a barra não executará um M.H.S.. 16. a corrente elétrica terá dimensão de segundo por Coulomb. Questão 18 - (UEM PR) Das afirmativas a seguir, assinale o que for correto. 01. Quando um móvel executa um movimento circular uniforme, sua aceleração é nula. 02. No movimento circular uniforme, a frequência é constante. 04. No movimento circular uniforme, o vetor velocidade tangencial é variável. 08. A forma angular da equação horária do movimento circular uniforme é t0  , onde  é a posição angular do móvel no instante t, 0 é a posição angular do móvel no instante 0t0  e  é a velocidade angular do móvel. 16. A frequência é inversamente proporcional ao quadrado do período. 32. Quando um ponto material percorre uma circunferência em movimento circular uniforme, a projeção do ponto material sobre um diâmetro da circunferência realiza um movimento harmônico simples. Questão 19 - (UEM PR) Imagine que você esteja em um carrossel de parque de diversões que gira em um movimento circular uniforme. A figura abaixo representa o carrossel visto de cima. O brinquedo gira sempre paralelo ao chão, sem movimentos verticais. Imagine agora que você lança, do ponto P, uma chave para um amigo parado a uma certa distância do brinquedo. Em que posição deveria estar esse amigo para apanhar a chave? (Despreze a resistência do ar.) a) I b) II c) III d) IV e) V Questão 20 - (UEM PR/2009) Duas polias, A e B, de raios R1 = 10cm e R2 = 20cm, giram acopladas por uma correia de massa desprezível que não desliza, e a polia A gira com uma frequência de rotação de 20 rpm. Assinale a(s) alternativa(s) correta(s). 01. A velocidade de qualquer ponto P da correia é aproximadamente 0,21 m/s. 02. A frequência angular de rotação da polia B é 2,0 rad/s. 04. A razão entre as frequências de rotação das polias A e B é 2. 08. O período de rotação da polia A é 3,0 s. 16. A aceleração centrípeta experimentada por uma partícula de massa m, colocada na extremidade da polia A (borda mais externa), é maior do que se a mesma partícula fosse colocada na extremidade da polia B. GABARITO: 1) Gab: D 2) Gab: A 3) Gab: D 4) Gab: 11 5) Gab: D 6) Gab: C 7) Gab: A 8) Gab: C 9) Gab: D 10) Gab: A 11) Gab: C 12) Gab: C 13) Gab: D 14) Gab: E 15) Gab: A 16) Gab: 02-04-08 17) Gab: 10 18) Gab: 46 19) Gab: C 20) Gab: 29 21) Gab: E 22) Gab: B 23) Gab: C 24) Gab: C 25) Gab: A 26) Gab: E 27) Gab: 02 28) Gab: 51 29) Gab: A 30) Gab: D 31) 3 1  B A I I 32) Gab: E 33) Gab: B 34) Gab: B 35) Gab: 29 36) Gab: E 37) Gab: a) f = 3,3 rad/s b) tf = 3s c) c = 3 rad/s d)  = 4,1 rad 38) Gab: 14 39) Gab: C 40) Gab: FFVF 41) Gab: A
  • 21. 21 LISTA 5 - VETORES E CINEMÁTICA VETORIAL RESUMO TEÓRICO Vetores Determinado por um segmento orientado AB, é o conjunto de todos os segmentos orientados equipolentes a AB. Se indicarmos com este conjunto, simbolicamente poderemos escrever: onde XY é um segmento qualquer do conjunto. O vetor determinado por AB é indicado por ou B - A ou . Um mesmo vetor é determinado por uma infinidade de segmentos orientados, chamados representantes desse vetor, os quais são todos equipolentes entre si. Assim, um segmento determina um conjunto que é o vetor, e qualquer um destes representantes determina o mesmo vetor. Usando um pouco mais nossa capacidade de abstração, se considerarmos todos os infinitos segmentos orientados de origem comum, estaremos caracterizando, através de representantes, a totalidade dos vetores do espaço. Ora, cada um destes segmentos é um representante de um só vetor. Consequentemente, todos os vetores se acham representados naquele conjunto que imaginamos. As características de um vetor são as mesmas de qualquer um de seus representantes, isto é: o módulo, a direção e o sentido do vetor são o módulo, a direção e o sentido de qualquer um de seus representantes. O módulo de se indica por | | . Soma de vetores Se v=(a,b) e w=(c,d), definimos a soma de v e w, por: v + w = (a+c,b+d) Propriedades da Soma de vetores Diferença de vetores Se v=(a,b) e w=(c,d), definimos a diferença entre v e w, por: v - w = (a-c,b-d) Produto de um número escalar por um vetor Se v=(a,b) é um vetor e c é um número real, definimos a multiplicação de c por v como: c.v = (ca,cb) Propriedades do produto de escalar por vetor Quaisquer que sejam k e c escalares, v e w vetores: Módulo de um vetor O módulo ou comprimento do vetor v=(a,b) é um número real não negativo, definido por: Vetor unitário Vetor unitário é o que tem o módulo igual a 1. Existem dois vetores unitários que formam a base canônica para o espaço R², que são dados por: i = (1,0) j = (0,1) Para construir um vetor unitário u que tenha a mesma direção e sentido que um outro vetor v, basta dividir o vetor v pelo seu módulo, isto é: Observação: Para construir um vetor u paralelo a um vetor v, basta tomar u=cv, onde c é um escalar não nulo. Nesse caso, u e v serão paralelos: Se c = 0, então u será o vetor nulo. Se 0 < c < 1, então u terá comprimento menor do que v. Se c > 1, então u terá comprimento maior do que v. Se c < 0, então u terá sentido oposto ao de v. Decomposição de vetores em Vetores Unitários Para fazer cálculos de vetores em apenas um dos planos em que ele se apresenta, pode-se decompor este vetor em vetores unitários em cada um dos planos apresentados. Sendo simbolizados, por convenção, î como vetor unitário do plano x e como vetor unitário do plano y. Caso o problema a ser resolvido seja dado em três dimensões, o vetor utilizado para o plano z é o vetor unitário .
  • 22. 22 Então, a projeção do vetor no eixo x do plano cartesiano será dado por , e sua projeção no eixo ydo plano será: . Este vetor pode ser escrito como: =( , ), respeitando que sempre o primeiro componente entre parênteses é a projeção em x e o segundo é a projeção no eixo y. Caso apareça um terceiro componente, será o componente do eixo z. No caso onde o vetor não se encontra na origem, é possível redesenhá-lo, para que esteja na origem, ou então descontar a parte do plano onde o vetor não é projetado. Fonte: http://www.sofisica.com.br/conteudos/Mecanica/Cinematica /Vetores.php EXERCÍCOS Questão 01 - (UDESC/2012) Observando o movimento de um carrossel no parque de diversões, conclui-se que seu movimento é do tipo circular uniforme. Assinale a alternativa correta em relação ao movimento. a) Não é acelerado porque o módulo da velocidade permanece constante. b) É acelerado porque o vetor velocidade muda de direção, embora mantenha o mesmo módulo. c) É acelerado porque o módulo da velocidade varia. d) Não é acelerado porque a trajetória não é retilínea. e) Não é acelerado porque a direção da velocidade não varia. Questão 02 - (MACK SP/2012) Um avião, após deslocar-se 120 km para nordeste (NE), desloca-se 160 km para sudeste (SE). Sendo um quarto de hora, o tempo total dessa viagem, o módulo da velocidade vetorial média do avião, nesse tempo, foi de a) 320 km/h b) 480 km/h c) 540 km/h d) 640 km/h e) 800 km/h Questão 03 - (FMTM MG/2006) A figura apresenta uma “árvore vetorial” cuja resultante da soma de todos os vetores representados tem módulo, em cm, igual a a) 8. b) 26. c) 34. d) 40. e) 52. Questão 04 - (UNIFICADO RJ) Considere um helicóptero movimentando-se no ar em três situações diferentes: I. Subindo verticalmente com velocidade constante; II. Descendo verticalmente com velocidade constante; III. Deslocando-se horizontalmente para a direita, com velocidade constante. A resultante das forças exercidas pelo ar sobre o helicóptero, em cada uma dessas situações, é corretamente representada por: a. b. c. d. e.        I II III Questão 05 - (MACK SP/2007) O movimento de uma partícula é caracterizado por ter vetor velocidade e vetor aceleração não nulo de mesma direção. Nessas condições, podemos afirmar que esse movimento é a) uniforme. b) uniformemente variado. c) harmônico simples. d) circular uniforme. e) retilíneo. Questão 06 - (PUC MG) Marque a alternativa INCORRETA. a) Quando o módulo da velocidade é constante, a aceleração pode não ser nula. b) Quando a aceleração é nula, o módulo da velocidade é constante. c) A componente de um vetor é um vetor. d) Quando um vetor é nulo, ele pode ter componentes não nulas. Questão 07 - (UNIUBE MG) Em uma pista de Fórmula Indy, um piloto faz o trajeto CD e FA com movimentos retardado, os trajetos AB e DE com movimento acelerado e os trechos BC e EF com movimento uniforme. A alternativa que mostra o vetor aceleração resultante corretamente, supondo que o movimento se realize na ordem alfabética, é:
  • 23. 23 F E B C A D a. F E B C A D b. F E B C A D c. F E B C A D d. A F E B C D e. Questão 08 - (PUC MG) Você e um amigo resolvem ir ao último andar de um edifício. Vocês partem juntos do primeiro andar. Entretanto, você vai pelas escadas e seu amigo, pelo elevador. Depois de se encontrarem na porta do elevador, descem juntos pelo elevador até o primeiro andar. É CORRETO afirmar que: a) o seu deslocamento foi maior que o de seu amigo. b) o deslocamento foi igual para você e seu amigo. c) o deslocamento de seu amigo foi maior que o seu. d) a distância que seu amigo percorreu foi maior que a sua. Questão 09 - (UFSC) Assinale a(s) afirmativa correta, some os valores respectivos e marque o resultado no cartão-resposta. A representação correta da velocidade tangencial “ v  ”, da quantidade de movimento “ q  ”, da aceleração centrípeta “ a  ” e da força normal “ F  ”, atuantes sobre um ponto material de massa m que descreve um movimento circular uniforme (MCU), está(ão) corretamente representada(s) no(s) esquema(s) 01. F a 0 q v 02. F a 0 q v 04. F a 0 q v 08. Fa 0 q v 16. F a 0 q v 32. F a 0 q v 64. Fa 0 q v Questão 10 - (CESGRANRIO RJ) A figura a seguir mostra a fotografia estroboscópica do movimento de uma partícula. V P IV III II I A aceleração da partícula, no ponto P da trajetória, é melhor representada pelo vetor: a) I; b) II; c) III; d) IV; e) V. Questão 11 - (UEL PR/2008) Um ciclista percorre as rotas 1 e 2 para se deslocar do ponto A ao ponto B, como mostrado no mapa a seguir, e registra em cada uma a distância percorrida. Assinale a alternativa que apresenta os valores aproximados da distância percorrida na rota 1 e na rota 2. Considere como aproximação todos os quarteirões quadrados com 100 m de lado. As rotas 1 e 2 encontram-se pontilhadas. a) rota 1  800 m; rota 2  800 m. b) rota 1  700 m; rota 2  700 m. c) rota 1  800 m; rota 2  900 m. d) rota 1  900 m; rota 2  700 m. e) rota 1  900 m;
  • 24. 24 rota 2  600 m Questão 12 - (UEL PR/2013) Uma família viaja para Belém (PA) em seu automóvel. Em um dado instante, o GPS do veículo indica que ele se localiza nas seguintes coordenadas: latitude 21°20’ Sul e longitude 48°30’ Oeste. O motorista solicita a um dos passageiros que acesse a Internet em seu celular e obtenha o raio médio da Terra, que é de 6730 km, e as coordenadas geográficas de Belém, que são latitude 1°20’ Sul e longitude 48°30’ Oeste. A partir desses dados, supondo que a superfície da Terra é esférica, o motorista calcula a distância D, do veículo a Belém, sobre o meridiano 48°30’Oeste. Assinale a alternativa que apresenta, corretamente, o valor da distância D, emkm. a) 6730 9 D   b) 2 )6730( 18 D   c) 6730 9 D   d) 6730 36 D   e) 6730 3 D 2         Questão 13 - (PUCCAMP SP/2011) Analise o esquema abaixo. O vetor resultante ou soma vetorial das três medidas acima representadas tem módulo a) 11 b) 13 c) 15 d) 17 e) 19 Questão 14 - (UNESP/2012) No dia 11 de março de 2011, o Japão foi sacudido por terremoto com intensidade de 8,9 na Escala Richter, com o epicentro no Oceano Pacífico, a 360 km de Tóquio, seguido de tsunami. A cidade de Sendai, a 320 km a nordeste de Tóquio, foi atingida pela primeira onda do tsunami após 13 minutos. (O Estado de S.Paulo, 13.03.2011. Adaptado.) Baseando-se nos dados fornecidos e sabendo que cos   0,934, onde  é o ângulo Epicentro-Tóquio-Sendai, e que 28  32  93,4  215 100, a velocidade média, em km/h, com que a 1.ª onda do tsunami atingiu até a cidade de Sendai foi de: a) 10. b) 50. c) 100. d) 250. e) 600. Questão 15 - (UFSCar SP) Nos esquemas estão representadas a velocidade V e a aceleração a do ponto material P. Assinale a alternativa em que o módulo da velocidade desse ponto material permanece constante. a. P v a   b. P a v  c. P a v  . d. P a v . e. P v a   Questão 16 - (UNIFESP SP/2002) Na figura, são dados os vetores a , b e c Sendo u a unidade de medida do módulo desses vetores, pode-se afirmar que o vetor cbad  :tem módulo a) 2u, e sua orientação é vertical, para cima. b) 2u, e sua orientação é vertical, para baixo. c) 4u, e sua orientação é horizontal, para a direita. d) 2 u, e sua orientação forma 45º com a horizontal, no sentido horário. e) 2 u, e sua orientação forma 45º com a horizontal, no sentido anti-horário. Questão 17 - (UNIFOR CE) Numa sala cúbica, de aresta a, uma mosca voa numa diagonal (segmento que une dois vértices, passando
  • 25. 25 pelo centro a sala). O deslocamento da mosca tem módulo a) a b) 2a c) 3a d) a 4 9 e) 3a Questão 18 - (FUVEST SP/2002) Em uma estrada, dois carros, A e B, entram simultaneamente em curvas paralelas, com raios RA e RB. Os velocímetros de ambos os carros indicam, ao longo de todo o trecho curvo, valores constantes VA e VB. Se os carros saem das curvas ao mesmo tempo, a relação entreVA e VB é: a) VA = VB b) VA/VB = RA/ RB c) VA/VB = (RA/ RB )2 d) VA/VB = RB/ RA e) VA/VB = (RB/ RA)2 Questão 19 - (UFOP MG) A velocidade do centro de massa de um disco que rola sem deslizar, em um plano horizontal é v. As velocidades dos pontos A e B, mostrados na figura são, respectivamente. O V A B X Y  a) – v e v b) 0 e v c) 0 e 2v d) v e v e) v e 2v Questão 20 - (MACK SP) Um corpo é atirado verticalmente para cima a partir do solo com velocidade inicial de módulo 50 m/s. O módulo de sua velocidade vetorial média entre o instante de lançamento e o instante em que retorna ao solo é: a) 50 m/s; b) 25 m/s; c) 5,0 m/s; d) 2,5 m/s; e) zero. GABARITO: 1) Gab: B 2) Gab: E 3) Gab: C 4) Gab: A 5) Gab: E 6) Gab: D 7) Gab: B 8) Gab: B 9) Gab: 04 10) Gab: B 11) Gab: D 12) Gab: A 13) Gab: B 14) Gab: E 15) Gab: C 16) Gab: B 17) Gab: C 18) Gab: B 19) Gab: C 20) Gab: E 21) Gab: C 22) Gab: D 23) Gab: B 24) Gab: B 25) Gab: E 26) Gab: D 27) Gab: E 28) Gab: E 29) Gab: E 30) Gab: A 31) Gab: C 32) Gab: D 33) Gab: E 34) Gab: D 35) Gab: E 36) Gab: 25 37) Gab: A 38) Gab: 07 39) Gab: 03 40) Gab: 25
  • 26. 26 LISTA 6 - LEIS DE NEWTON RESUMO TEÓRICO DINÂMICA Quando se fala em dinâmica de corpos, a imagem que vem à cabeça é a clássica e mitológica de Isaac Newton, lendo seu livro sob uma macieira. Repentinamente, uma maçã cai sobre a sua cabeça. Segundo consta, este foi o primeiro passo para o entendimento da gravidade, que atraia a maçã. Com o entendimento da gravidade, vieram o entendimento de Força, e as três Leis de Newton. Na cinemática, estuda-se o movimento sem compreender sua causa. Na dinâmica, estudamos a relação entre a força e movimento. Força: É uma interação entre dois corpos. O conceito de força é algo intuitivo, mas para compreendê- lo, pode-se basear em efeitos causados por ela, como: Aceleração: faz com que o corpo altere a sua velocidade, quando uma força é aplicada. Deformação: faz com que o corpo mude seu formato, quando sofre a ação de uma força. Força Resultante: É a força que produz o mesmo efeito que todas as outras aplicadas a um corpo. Dadas várias forças aplicadas a um corpo qualquer: A força resultante será igual a soma vetorial de todas as forças aplicadas: Leis de Newton As leis de Newton constituem os três pilares fundamentais do que chamamos Mecânica Clássica, que justamente por isso também é conhecida por Mecânica Newtoniana. 1ª Lei de Newton - Princípio da Inércia  Quando estamos dentro de um carro, e este contorna uma curva, nosso corpo tende a permanecer com a mesma velocidade vetorial a que estava submetido antes da curva, isto dá a impressão que se está sendo "jogado" para o lado contrário à curva. Isso porque a velocidade vetorial é tangente a trajetória.  Quando estamos em um carro em movimento e este freia repentinamente, nos sentimos como se fôssemos atirados para frente, pois nosso corpo tende a continuar em movimento. estes e vários outros efeitos semelhantes são explicados pelo princípio da inércia, cujo enunciado é: "Um corpo em repouso tende a permanecer em repouso, e um corpo em movimento tende a permanecer em movimento." Então, conclui-se que um corpo só altera seu estado de inércia, se alguém, ou alguma coisa aplicar nele uma força resultante diferente se zero. 2ª Lei de Newton - Princípio Fundamental da Dinâmica Quando aplicamos uma mesma força em dois corpos de massas diferentes observamos que elas não produzem aceleração igual. A 2ª lei de Newton diz que a Força é sempre diretamente proporcional ao produto da aceleração de um corpo pela sua massa, ou seja: ou em módulo: F=ma Onde: F é a resultante de todas as forças que agem sobre o corpo (em N); m é a massa do corpo a qual as forças atuam (em kg); a é a aceleração adquirida (em m/s²). A unidade de força, no sistema internacional, é o N (Newton), que equivale a kg m/s² (quilograma metro por segundo ao quadrado). 3ª Lei de Newton - Princípio da Ação e Reação Quando uma pessoa empurra um caixa com um força F, podemos dizer que esta é uma força de ação. mas conforme a 3ª lei de Newton, sempre que isso ocorre, há uma outra força com módulo e direção iguais, e sentido oposto a força de ação, esta é chamada força de reação. Esta é o princípio da ação e reação, cujo enunciado é: "As forças atuam sempre em pares, para toda força de ação, existe uma força de reação." EXERCÍCIOS TEXTO: 1 - Comum à questão: 1 Considere as Leis de Newton e as informações a seguir. Uma pessoa empurra uma caixa sobre o piso de uma sala. As forças aplicadas sobre a caixa na direção do movimento são: - Fp: força paralela ao solo exercida pela pessoa; - Fa: força de atrito exercida pelo piso. A caixa se desloca na mesma direção e sentido de Fp . A força que a caixa exerce sobre a pessoa é Fc . Questão 01 - (UERJ/2012) Se o deslocamento da caixa ocorre com velocidade constante, as magnitudes das forças citadas apresentam a seguinte relação: a) Fp = Fc = Fa b) Fp > Fc = Fa c) Fp = Fc > Fa d) Fp = Fc < Fa Questão 02 - (UPE/2012) Um corpo de massa m está suspenso por duas molas ideais, paralelas, com constantes elásticas k e
  • 27. 27 deformadas de d. Sabendo que o sistema se encontra em equilíbrio, assinale a alternativa que expressa k. Dado: Considere a aceleração da gravidade g. a) d mg2 b) d mg c) d mg 2 d) mg d2 e) mg d Questão 03 - (UFJF MG) Colocando-se sobre as bordas de uma mesa dois lápis L1 e L2 de modo que uma parte deles sobressaia da mesa, tal que possam sustentar um palito de churrasco c1 como mostra a figura, golpeando-se rapidamente o palito de churrasco, ele se quebra e os lápis não. Este fenômeno está relacionado com: L1 L2 C1 a) O princípio da inércia. b) A conservação do momento. c) A conservação da energia. d) A conservação do momento e energia. e) A lei de conservação dos choques. Questão 04 - (UFOP MG) Quando um ônibus é freado bruscamente, todos os passageiros são lançados” para a frente. Explique fisicamente este fenômeno. Questão 05 - (UFSC) Uma mola comprimida no interior de um tubo cilíndrico impulsiona uma bola, projetando–a horizontalmente para fora do tubo. Desprezando–se a resistência do ar, o esquema que representa corretamente a(s) força(s) atuante(s) sobre a bola fora do tubo é: 01. 02. 04. 08. 16. Questão 06 - (UFU MG/2011) Um objeto é lançado verticalmente na atmosfera terrestre. A velocidade do objeto, a aceleração gravitacional e a resistência do ar estão representadas pelos vetores v  , g  e atritoF  , respectivamente. Considerando apenas estas três grandezas físicas no movimento vertical do objeto, assinale a alternativa correta. a) b) c) d) Questão 07 - (UFTM/2007) As confusões entre os conceitos de peso e massa são muitas, mas sabemos que em um corpo qualquer, essas grandezas são distintas. Analise as afirmações: I. A massa de um corpo é uma medida da inércia desse corpo. II. Peso está relacionado à força com que a Terra atrai um corpo. III. O peso é uma grandeza vetorial. Já a massa é uma grandeza escalar. É correto o contido em a) I, apenas. b) I e II, apenas. c) I e III, apenas. d) II e III, apenas. e) I, II e III. Questão 08 - (MACK SP/2008) No sistema a seguir, o fio e a polia são considerados ideais e o atrito entre as superfícies em contato é desprezível. Abandonando-se o corpo B a partir do repouso, no ponto M, verifica-se que, após 2 s, ele passa pelo ponto N com velocidade de 8 m/s. Sabendo-se que a massa do corpo A é de 5 kg, a massa do corpo B é
  • 28. 28 a) 1 kg b) 2 kg c) 3 kg d) 4 kg e) 5 kg Questão 09 - (UFLA MG) Dois corpos têm massas m1 e m2. Aplicando-se as forças F1  e F2  em m1 e m2, estas passam a ter acelerações a1  e a2  , respectivamente. Como resultados possíveis para as acelerações, teremos, EXCETO: a) Se a2a1, entãom2m1eF2F1   b) Se aa, entãommeFF 212121   c) Se a2a1, entãom2m1eF2F1   d) Se aa, entãommeFF 212121   e) Se aa, entãommeFF 212121   Questão 10 - (UNIFOR) Num anel atuam simultaneamente três forças coplanares, 1F , 2F e 3F , representadas abaixo. A resultante 321 FFFR  tem módulo, em newtons, a) 11 b) 9,0 c) 8,0 d) 7,0 e) 5,0 Questão 11 - (UNIOESTE PR) O equilíbrio é uma situação física comum no nosso cotidiano. Os engenheiros, por exemplo, ao elaborarem muitos de seus projetos, estão constantemente atentos para atender adequadamente às condições necessárias e suficientes para que o equilíbrio ocorra. Assinale, entre as alternativas a seguir, aquela que apresenta um corpo em equilíbrio. a) Um brinquedo em movimento circular uniforme, preso a uma corda. b) Um satélite em órbita em torno da Terra. c) Um livro no ponto mais alto da trajetória, quando lançado verticalmente para cima por um aluno. d) Uma bola que se movimenta em uma trajetória parabólica, após ter sido chutada pelo goleiro em um jogo de futebol. e) Um elevador em movimento vertical com velocidade constante. Questão 12 - (UNIOESTE PR) São fornecidas abaixo diversas afirmativas, envolvendo conceitos de mecânica. Assinale a(s) alternativa(s) correta(s). 01. Um objeto, inicialmente em repouso, recebe um impulso e adquire velocidade. Após um curto intervalo de tempo, o impulso deixa de atuar. Pode-se afirmar que, a partir deste momento, como consequência do impulso, o objeto passa a ter uma força, a qual, aos poucos, vai cessando, até que o objeto volte ao repouso. Considere que o objeto não sofre nenhum outro tipo de interação. 02. Um objeto não pode realizar uma trajetória curva com velocidade escalar constante, caso a soma vetorial de todas as forças que sobre ele atuam seja nula. 04. A velocidade de um projétil, sujeito apenas ao campo gravitacional da Terra, lançado obliquamente, aumenta até atingir um valor máximo, após o que o projétil se desloca verticalmente, de volta para o chão. 08. É possível exercer uma força sobre um objeto sem que haja realização de trabalho. 16. Um objeto que está em repouso não pode estar sendo submetido à ação da gravidade. 32. A atração gravitacional entre dois objetos leva um certo tempo para ocorrer, não sendo instantânea. 64. Para que um objeto esteja em movimento, não é necessário haver uma força aplicada sobre ele. Questão 13 - (UNIOESTE PR) No teto de um elevador, está pendurado um dinamômetro que tem, na sua outra extremidade, um pequeno corpo de peso 1,6 N. O dinamômetro, no entanto, acusa 2,0 N. O elevador está a) subindo com velocidade constante. b) em repouso. c) descendo com velocidade constante. d) subindo com velocidade crescente. e) descendo com velocidade crescente. Questão 14 - (UNIOESTE PR) O bloco A sobre uma mesa tem massa 15,0kg, e o bloco B suspenso tem massa 5,0kg. O coeficiente de atrito entre o bloco A e a mesa vale 0,20. Considere que o fio utilizado para unir os blocos é ideal (sem peso e inextensível) e que a aceleração gravitacional vale 10,0m/s2 . Analise as afirmações e assinale-as devidamente.
  • 29. 29 00. A aceleração adquirida pelos corpos, durante seus movimentos, é de 1,0 m/s2 . 01. No fio ideal, a tração é de 45,0N. 02. A velocidade do bloco A, após 3,0s de iniciado o movimento, é de 30,0 m/s. 03. A distância percorrida pelo bloco A, após 3,0 s de iniciado o movimento, é de 4,5 m. 04. Os blocos A e B têm acelerações diferentes. Questão 15 - (UEL PR) “Em média, as grandes usinas geradoras brasileiras precisam de 2180 litros de água para gerar 1MW. A usina hidrelétrica de Três Marias tem 396 MW de potência instalada. Para gerar 1MW ela precisa que 2325 litros passem por suas turbinas. Quando 920 mil litros passam pelas suas turbinas, ela gera 396 MW, que é sua potência instalada. Qualquer quantidade superior de água não gerará mais energia, pois o máximo que a hidrelétrica pode produzir é 396MW.” Matéria publicada na Folha de São Paulo do dia 22 de abril de 2001, página B4. A água que desce da represa para a usina é usada com o objetivo de acionar as turbinas dos geradores de eletricidade. Na usina de Três Marias, quando 920 mil litros de água chegam na entrada das turbinas, é correto afirmar: a) A velocidade será muito maior que a velocidade de 2325 litros de água. b) A velocidade será igual à velocidade de 2325 litros de água. c) A velocidade será menor que a velocidade de 2325 litros de água. d) Não se pode comparar 920 mil litros de água com 2325 litros de água, porque as suas acelerações serão diferentes. e) É necessário calcular a aceleração de cada uma das quantidades de água envolvidas, para se proceder à comparação. Questão 16 - (UEL PR) Uma pessoa apoia-se em um bastão sobre uma balança, conforme a figura abaixo. A balança assinala 70kg. Se a pessoa pressiona a bengala, progressivamente, contra a balança, a nova leitura: a) Indicará um valor maior que 70kg. b) Indicará um valor menor que 70kg. c) Indicará os mesmos 70kg. d) Dependerá da força exercida sobre o bastão. e) Dependerá do ponto em que o bastão é apoiado na balança. Questão 17 - (UEL PR) O cabo de um reboque arrebenta se nele for aplicada uma força que exceda 1800N. Suponha que o cabo seja usado para rebocar um carro de 900kg ao longo de uma rua plana e retilínea. Nesse caso, que aceleração máxima o cabo suporta? a) 0,5m/s2 . b) 1,0m/s2 . c) 2,0m/s2 . d) 4,0m/s2 . e) 9,0m/s2 . Questão 18 - (UEL PR) Um jogador de tênis, ao acertar a bola com a raquete, devolve-a para o campo do adversário. Sobre isso, é correto afirmar: a) De acordo com a Segunda Lei de Newton, a força que a bola exerce sobre a raquete é igual, em módulo, à força que a raquete exerce sobre a bola. b) De acordo com a Primeira Lei de Newton, após o impacto com a raquete, a aceleração da bola é grande porque a sua massa é pequena. c) A força que a raquete exerce sobre a bola é maior que a força que a bola exerce sobre a raquete, porque a massa da bola é menor que a massa da raquete. d) A bola teve o seu movimento alterado pela raquete. A Primeira Lei de Newton explica esse comportamento. e) Conforme a Segunda Lei de Newton, a raquete adquire, em módulo, a mesma aceleração que a bola. Questão 19 - (UEM PR) Uma massa m é presa a uma associação de duas molas ideais 1 e 2, de constantes elásticas, respectivamente, k1 e k2, como ilustradas na figura abaixo. Considerando-se x1 e x2, respectivamente, os deslocamentos das posições de equilíbrio das molas 1 e 2, e kE a constante elástica resultante da associação das molas, podemos afirmar que: 01. k2 x2 = k1 x1. 02. k2 x2 = mg. 04. kE = k1 + k2. 08. kE x = mg, sendo x = x1 + x2. 16. – kE x = resultante das forças que atua na massa m, sendo x = x1 + x2. Questão 20 - (UEM PR) Através de uma polia ideal, passa uma corda C1, que sustenta duas massas, m1 e m2. Outra corda, C2, presa a m2, sustenta uma massa m3, conforme a figura. Considere as cordas idênticas e ideais. Considere também que a tração em C1 é T1, e em C2 é T2. Se m1 m2  m3, pode-se afirmar corretamente que:
  • 30. 30 01. quando m3 + m2 = m1, a aceleração do sistema é nula. 02. quando m3 + m2 = m1, T1 = T2. 04. quando m3 + m2 = 2m1, a aceleração do sistema é g/2. 08. invertendo–se a posição das massas m2 e m3, a aceleração do sistema não se altera. 16. quando m3 > m2, T2 > T1. GABARITO: 1) Gab: A 2) Gab: C 3) Gab: A 4) Gab: inércia 5) Gab: 08 6) Gab: A 7) Gab: E 8) Gab: C 9) Gab: D 10) Gab: E 11) Gab: E 12) Gab: 02+08+64 13) Gab: D 14) Gab: VVFVF 15) Gab: B 16) Gab: C 17) Gab: C 18) Gab: D 19) Gab: 01-02-08 20) Gab: 01-08 21) Gab: E 22) Gab: 17 23) Gab: A 24) Gab: 02+04+08+16 25) Gab: 01-02-04-08 26) Gab: 4,5 27) Gab: C 28) Gab: A 29) Gab: C 30) Gab: B 31) Gab: B 32) Gab: 17 33) Gab: A 34) Gab: B 35) Gab: A 36) Gab: C 37) Gab: A 38) Gab: D 39) Gab: C 40) Gab: B
  • 31. 31 LISTA 7 - APLICAÇÕES DAS LEIS DE NEWTON RESUMO TEÓRICO A maior dificuldade de estudantes de física do ensino médio é com relação à solução de problemas, nas mais diversas frentes da disciplina. Apresentarei aqui a solução de um exercício sobre interação entre blocos, relacionado a aplicação das Leis de Newton. Exemplo 1) (UF-PE) A figura abaixo mostra três blocos de massas mA = 1,0 kg, mB = 2,0 kg e mc = 3,0 kg. Os blocos se movem em conjunto, sob a ação de uma força F constante e horizontal, de módulo 4,2 N. Desprezando o atrito, qual o módulo da força resultante sobre o bloco B? a) 1,0 N b) 1,4 N c) 1,8 N d) 2,2 N e) 2,6 N Solução. Primeiramente devemos saber que F = m.a (Segunda lei de Newton). Devemos saber também, que os três corpos se movem com a mesma aceleração, e que essa aceleração tem a mesma direção e sentido da força F. Assim podemos calcular a aceleração dos blocos pelo Sistema de Corpos Isolados (SCI) Na figura abaixo representamos as forças que agem em A, B e C Em que F é a força aplicada. Fa no primeiro bloco é a reação de b em a devido a F. Fa no segundo bloco é a ação de a em b devido a F Fc no segundo bloco é a reação de c em b devido a F Fc no terceiro bloco é a ação de de b em c devido a F n é a força normal e p é a força peso nos três casos Simplificando-se os pesos com as forças de reação normal em cada caso temos que: Como F = 4,2 N, temos: 4,2 = (1,0 + 2,0 + 3,0).a a = 4,2/6 a = 0,7 m/s2 Encontrada a aceleração devemos encontrar o valor da resultante em B. A resultante em B é: FB = FA – FC Subtraído as equações B e C temos: FA = (mB + mC) . a FA = (2,0 + 3,0). 0,7 FA = 5,0 . 0,7 FA = 3,5 N Encontrando o valor de FC FC = mC . a FC = 3,0 . 0,7 FC = 2,1 N Assim: FB = 3,5 – 2,1 FB = 1,4 N Concluímos então que a força resultante em B é igual a 1,4 N. EXERCÍCIOS Questão 01 - (UFT TO/2011) Um piloto de teste pisa no acelerador de uma Ferrari, para aumentar sua velocidade escalar, em uma pista plana horizontal. Considere que o coeficiente de atrito estático entre os pneus da Ferrari e a pista vale 0,5. Nesta situação, qual valor melhor representa o módulo da aceleração máxima que esta Ferrari pode atingir nesta pista? Desconsidere o atrito com o ar, e considere o módulo da aceleração da gravidade igual a 9,8 m/s2 . a) 3,8 m/s2 b) 4,9 m/s2 c) 16,2 m/s2 d) 11,1 m/s2 e) 9,8 m/s2 Questão 02 - (ACAFE SC/2011) Um garoto obeso tem um peso de módulo 1200 N (120kgf) e tenta mover uma caixa pesada conforme a figura. O coeficiente de atrito estático entre os sapatos do garoto e o piso é 0,5. Assinale a alternativa correta que apresenta o módulo da máxima força horizontal, em newtons, que o garoto pode aplicar na caixa. a) 1200 b) 600 c) 1800 d) 300 Questão 03 - (FEI SP) Um caminhão, partindo do repouso, carrega uma bobina de aço de massa m sobre a sua carroceria sem que haja escorregamento. Quanto à força de atrito de
  • 32. 32 escorregamento na bobina, quando o caminhão estiver acelerando podemos afirmar que: a) não há força de atrito entre a bobina e a carroceria b) tem direção normal à carroceria e sentido igual ao de deslocamento do caminhão c) tem direção paralela à carroceria e sentido igual ao de deslocamento do caminhão d) tem direção paralela à carroceria e sentido contrário ao de deslocamento do caminhão e) tem direção normal à carroceria e sentido contrário ao de deslocamento do caminhão Questão 04 - (UDESC/2011) A figura abaixo mostra uma caixa de madeira que desliza para baixo com velocidade constante sobre o plano inclinado, sob a ação das seguintes forças: peso, normal e de atrito. Assinale a alternativa que representa corretamente o esquema das forças exercidas sobre a caixa de madeira. a) b) c) d) e) Questão 05 - (UFOP MG) Em um bloco em movimento sobre uma superfície horizontal com velocidade constante atuam as quatro forças mostradas na figura abaixo. Das forças indicadas, assinar quais representam um par ação-reação. Fa FV N P a) (N, P) b) (F, Fa) c) (N, Fa) d) (N, P) e (F, Fa) e) nenhum par construído com as forças da figura constitui um par ação-reação. Questão 06 - (Unifacs BA/2011) Considerando-se um pequeno bloco, preso na extremidade de um fio ideal, que descreve um movimento circular uniforme sobre uma superfície horizontal, sem atrito, pode-se afirmar: 01. A força resultante que atua sobre o bloco é zero. 02. O período do movimento independe do comprimento do fio. 03. A tração no fio produz variação na direção da velocidade do bloco. 04. O bloco afastará radialmente, sob a ação exclusiva da força centrífuga, se o fio sofrer rompimento. 05. O módulo da aceleração centrípeta do bloco é diretamente proporcional à sua velocidade angular. Questão 07 - (UERJ) Um balão, de peso igual a 0,1 N, está preso a um fio. Além da força de empuxo E, o ar exerce uma forço horizontal F que empurro e inclina o fio em relação à vertical, conforme mostra afigura. A tração no fio tem módulo igual o 0,2 N. Calcule, em newtons, os módulos de: a) E; b) F. Questão 08 - (FATEC SP) Um corpo em movimento, num plano horizontal, descreve uma trajetória curva. É correto afirmar que: a) o movimento é necessariamente circular uniforme. b) a força resultante é necessariamente centrípeta. c) a trajetória é necessariamente parabólica. d) a força centrípeta existe apenas quando a trajetória é circular. Questão 09 - (UFF RJ) A figura mostra, em vista lateral, o exato instante em que uma pipa paira no ar, em equilíbrio, sob a ação do vento que sopra com uma velocidade horizontal constante.
  • 33. 33 A força que o vento faz sobre a pipa nesta situação está mais bem representada, na figura, pelo vetor: 1Fa. 2Fb. 3Fc. 4Fd. 5Fe. Questão 10 - (FURG RS) Um carro faz uma curva de 80 m de raio, com velocidade de módulo constante igual a 72 km/h. Podemos afirmar que sua aceleração é: a) Zero m/s2 b) 0,5 m/s2 c) 0,9 m/s2 d) 4 m/s2 e) 5 m/s2 Questão 11 - (UEL PR/2008) Com relação a um corpo em movimento circular uniforme e sem atrito, considere as afirmativas seguintes: I. O vetor velocidade linear é constante. II. A aceleração centrípeta é nula. III. O módulo do vetor velocidade é constante. IV. A força atua sempre perpendicularmente ao deslocamento. Assinale a alternativa que contém todas as afirmativas corretas. a) I e IV. b) II e III. c) III e IV. d) I, II e III. e) I, II e IV. Questão 12 - (UEM PR) O goleiro de um time de futebol bate um tiro de meta e a bola percorre a trajetória esquematizada abaixo. Despreze a resistência do ar e assinale o que for correto (o ponto B corresponde ao instante em que a bola atinge o solo). 01. No ponto A, a resultante das forças que atua sobre a bola é para a direita e para cima. 02. No ponto B, a resultante das forças que atua sobre a bola é nula. 04. No ponto A, a velocidade resultante da bola é para a direita e para cima. 08. No ponto B, a velocidade resultante da bola é nula. 16. No ponto A, a energia total da bola é maior que no ponto B. Questão 13 - (UEM PR) Um motociclista descreve uma circunferência num “globo da morte" de raio 4 m, em movimento circular uniforme, no sentido indicado pela seta curva, na figura abaixo. A massa total (motorista + moto) é de 150 kg. Considere g = 10 m/s e assinale o que for correto. 01.A velocidade do motociclista em B é tangente à circunferência e dirigida para baixo (). 02.A aceleração do motociclista no ponto C é dirigida para o centro da circunferência. 04.A força resultante sobre o motociclista no ponto A é dirigida para fora da circunferência e perpendicular à mesma (). 08.Se a velocidade do motociclista no ponto mais alto da circunferência for 12 m/s, a força exercida sobre o globo nesse ponto será 3900 N. 16.No ponto mais baixo da circunferência, a força exercida sobre o globo é a mesma que a da parte mais alta. 32.A velocidade mínima que o motociclista deve ter no ponto mais alto da circunferência para que ele consiga fazer a volta completa sem cair é 6,3 m/s. Questão 14 - (UEM PR/2008) Sobre aviões de acrobacia que executam loopings (movimento circular cuja trajetória ocorre em um plano vertical em relação ao horizonte) no céu, é correto afirmar que, durante a execução do looping, a) a aceleração centrífuga é duas vezes maior que a centrípeta. b) a energia potencial gravitacional se anula durante o looping. c) existe somente energia rotacional na execução da manobra. d) a aceleração centrípeta torna-se mais intensa que a aceleração da gravidade. e) a aceleração centrífuga reduz-se a um quarto da centrípeta. Questão 15 - (UNIOESTE PR) Um carro de massa 1800 kg está em movimento circular sobre uma rodovia circular plana de raio igual a 200 m. O módulo de sua velocidade é constante. O coeficiente de atrito estático entre os pneus do carro e a rodovia é de 20,0e  . Considere a aceleração da gravidade g = 10 m/s2 e desconsidere o atrito com o ar. Para as condições de movimento descritas, assinale a alternativa correta. a) O módulo da força de atrito que atua sobre o carro, resultante da interação dos pneus do carro e a rodovia, tem valor constante de 3600 N,
  • 34. 34 independentemente do módulo da velocidade de movimento do carro. b) A força de atrito sobre o carro, resultante da interação entre os pneus do carro e a rodovia, é a força centrípeta que permite o movimento do carro em círculo. c) Os vetores força peso e força centrípeta sobre o carro possuem o mesmo sentido. d) A força de atrito sobre o carro, resultante da interação entre os pneus do carro e a rodovia, atua em sentido contrário à força centrípeta sobre o carro, que o mantém em movimento sobre a rodovia. e) A força resultante sobre o carro é nula. Questão 16 - (UNIOESTE PR/2008) Um carrinho de brinquedo é solto a partir do repouso para percorrer uma pista sinuosa como mostra a figura abaixo. Depois de descer a rampa de altura h, o carrinho encontra uma lombada, cuja elevação acompanha a forma de um semicírculo de raio r. Supondo que não exista nenhum atrito agindo no brinquedo, qual o valor máximo da razão h/r, para que o carrinho permaneça em contato com a pista na parte superior da lombada? a) 1/2. b) 3/2. c) 4/3. d) 5/3. e) 5/2. Questão 17 - (UNIOESTE PR/2010) Uma criança pendura-se na extremidade livre de uma corda que tem a outra extremidade presa ao teto de uma sala de ginástica. Ela, então, impulsiona-se e faz uma trajetória circular cujo diâmetro é 0,8 m. Se a velocidade tangencial da criança, cuja massa é 40 kg, for 2,0 m/s, qual será o ângulo que a corda faz com uma linha vertical perpendicular ao solo? Considere a criança como uma partícula, a massa da corda desprezível e a aceleração gravitacional igual 10 m/s2 . a) 30º. b) 35º. c) 45º. d) 20º. e) 60º. Questão 18 - (UEL PR) A força de atrito é sempre dissipativa e resiste ao movimento, mas há situações em que, embora esta força seja resistente ao movimento, ela possibilita que o movimento seja favorecido. Assinale a alternativa que apresenta a situação física em que a força de atrito com a superfície, ou de resistência de um fluido, favorece o movimento. a) Força de resistência do ar que atua em um automóvel em movimento. b) Força de resistência do ar que atua em um paraquedas. c) Força de atrito entre duas placas de vidro bem polido molhadas. d) Força de atrito que o chão aplica nos pneus de um carro em movimento. e) Força de atrito entre o pistão e o cilindro no motor do automóvel. Questão 19 - (UEL PR/2012) Uma pessoa, de massa 80,0 kg, consegue aplicar uma força de tração máxima de 800,0 N. Um corpo de massa M necessita ser levantado como indicado na figura a seguir. O coeficiente de atrito estático entre a sola do sapato da pessoa e o chão de concreto é e = 1,0. Faça um esboço de todas as forças que atuam em todo o sistema e determine qual a maior massa M que pode ser levantada pela pessoa sem que esta deslize, para um ângulo  = 45º. Questão 20 - (UEM PR/2006) Uma caixa contendo ferramentas está em repouso sobre uma superfície horizontal áspera. Uma pessoa está tentando colocá-la em movimento, empurrandoa com uma força paralela à superfície, mas não está conseguindo. Qual a razão para isso? a) A força que a mão da pessoa faz sobre a caixa é a mesma que a caixa faz sobre a mão. b) A força que o solo faz sobre a caixa devido ao atrito cinético é muito maior do que a força que a pessoa faz sobre a caixa. c) A força de atrito estático que o solo exerce sobre a caixa é de mesma intensidade da força que a mão faz sobre a caixa. d) A quantidade de momento que a caixa possui. e) O fato de o torque que a mão imprime à caixa ser menor do que a força de atrito estático que o solo exerce sobre a caixa. GABARITO: 1) Gab: B 2) Gab: B 3) Gab: C 4) Gab: E 5) Gab: E 6) Gab: 03 7) Gab: a) 0,27N b) 0,1N 8) Gab: C 9) Gab: B 10) Gab: E 11) Gab: C 12) Gab: 04 13) Gab: 43 14) Gab: D 15) Gab: B 16) Gab: B 17) Gab: C 18) Gab: D 19) Gab:
  • 35. 35 M = 40 2 Kg. 20) Gab: C 21) Gab: 11 22) Gab: 03 23) Gab: C 24) Gab: B 25) Gab: C 26) Gab: D 27) Gab: a)v = 4m/s b) aR = 0,8m/s2 c) NN = 552N e NJ = 756N 28) Gab: 04 29) Gab: E 30) Gab: 42 31) Gab: C 32) Gab: D 33) Gab: 29 34) Gab: 23 35) Gab: C 36) Gab: 09 37) Gab: C 38) Gab: C 39) Gab: D 40) Gab: 50